204
G.R. No. 137705 August 22, 2000 SERG'S PRODUCTS, INC., and SERGIO T. GOQUIOLAY, petitioners, vs. PCI LEASING AND FINANCE, INC., respondent. FACTS: PCI Leasing filed a complaint for a sum of money with an application for a writ of replevin. Writ of Replevin was granted directing its sheriff to seize and deliver the machineries and equipment to PCI Leasing after 5 days and upon the payment of the necessary expenses. Serg's Products Inc. filed a motion for special protective order but the motion was opposed by PCI Leasing, on the ground that the properties were still personal and therefore still subject to seizure and a writ of replevin. Serg's Products Inc. asserted that the properties sought to be seized were immovables. They argued that to give effect to the agreement would be prejudicial to innocent third parties. Appellate court held that the subject machines were personal property, and that they had only been leased, not owned, by petitioners. ISSUE: Whether or not the machineries purchased and imported by SERG’S became real property by virtue of immobilization. RULING: 1

PROPERTY (Civil Law)

Embed Size (px)

DESCRIPTION

Digested cases in Property

Citation preview

G.R. No. 137705 August 22, 2000

SERG'S PRODUCTS, INC., and SERGIO T. GOQUIOLAY, petitioners, vs. PCI LEASING AND FINANCE, INC., respondent.

FACTS:

PCI Leasing filed a complaint for a sum of money with an application for a writ of replevin. Writ of Replevin was granted directing its sheriff to seize and deliver the machineries and equipment to PCI Leasing after 5 days and upon the payment of the necessary expenses.

Serg's Products Inc. filed a motion for special protective order but the motion was opposed by PCI Leasing, on the ground that the properties were still personal and therefore still subject to seizure and a writ of replevin.

Serg's Products Inc. asserted that the properties sought to be seized were immovables. They argued that to give effect to the agreement would be prejudicial to innocent third parties.

Appellate court held that the subject machines were personal property, and that they had only been leased, not owned, by petitioners.

ISSUE:

Whether or not the machineries purchased and imported by SERG’S became real property by virtue of immobilization.

RULING:

Court held that the machines were placed by petitioners in the factory built on their own land. Indisputably, they were essential and principal elements of their chocolate-making industry. Although each of them was movable or personal property on its own, all of them have become "immobilized by destination" because they are essential and principal elements in the industry. Court

1

agreed with petitioners that said machines are real, not personal pursuant to Article 415 (5) of the Civil Code.

However, the Court held that contracting parties may validly stipulate that a real property be considered as personal. After agreeing to such stipulation, they are consequently estopped from claiming otherwise.

In this case, the Lease Agreement clearly provides that the machines in question are to be considered as personal property. Hence, the petitioners are estopped from denying the characterization of the subject machines as personal property. Under the circumstances, they are proper subjects of the Writ of Seizure.

In treating the machines as personal property pursuant to the Lease Agreement is good only insofar as the contracting parties are concerned. Third persons acting in good faith are not affected by its stipulation characterizing the subject machinery as personal.

2

G.R. No. L-18456 November 30, 1963

CONRADO P. NAVARRO, plaintiff-appellee, vs.

RUFINO G. PINEDA, RAMONA REYES, ET AL., defendants-appellants.

FACTS:

To secure a loan, Pineda and his mother executed real estate and chattel mortgages in favor of Navarro. The real estate mortgage covered a parcel of land owned by the mother while the chattel mortgage covered a residential house. The defendants failed to pay when the mortgage debt became due and payable. They asked and granted extensions to pay for the loan.

On the second extension, Pineda executed a PROMISE wherein in case of default in payment, he wouldn’t ask for any additional extension and there would be no need for any formal demand. In spite of this, they still failed to pay.

Navarro then filed a complaint for foreclosure of the mortgage and for damages. The trial court decided in his favor.

ISSUE:

Whether or not the deed of real estate mortgage and chattel mortgage appended to the complaint is valid notwithstanding the fact that the house was made subject of chattel mortgage for the reason that it is erected on a land that belongs to a third person.

RULING:

Where a house stands on a rented land belonging to another person, it may be the subject matter of a chattel mortgage as personal property if so stipulated in the

3

document of mortgage, and in an action by the mortgagee for the foreclosure, the validity of the chattel mortgage cannot be assailed by one of the parties to the contract of mortgage.

Furthermore, although in some instances, a house of mixed materials has been considered as a chattel between the parties and that the validity of the contract between them, has been recognized, it has been a constant criterion that with respect to third persons, who are not parties to the contract, and specially in execution proceedings, the house is considered as immovable property.

G.R. Nos. L-10837-38 May 30, 1958

ASSOCIATED INSURANCE and SURETY COMPANY, INC., plaintiff, vs. ISABEL IYA, ADRIANO VALINO and LUCIA VALINO, defendants.

FACTS:

Adriano Valino and Lucia A. Valino, husband and wife, purchased a house on installment basis from the Philippine Realty Corporation. To enable her to purchase on credit rice from NARIC, Valino filed a bond (P11,000) subscribed by Associated Insurance and Surety Co Inc, and as a counter-guaranty, Valino executed an alleged chattel mortgage on the aforementioned house in favour of the surety company.

To secure payment of an indebtedness in the amount of P12,000.00, the Valinos executed a real estate mortgage over the lot and the house in favor of Isabel Iya.

As Valino failed to satisfy her obligation to the NARIC, the company foreclosed the chattel mortgage over the house. A public sale was conducted wherein the property was awarded to the surety company, and then caused the said house to be declared in its name for tax purposes.

The surety company learned of the existence of the real estate mortgage over the lot and the improvements thereon; thus, they prayed for the exclusion of the residential house from the real estate mortgage and the declaration of its ownership in virtue of the award given during bidding.

4

Isabel Iya filed her answer alleging among other things, that in virtue of the real estate mortgage executed by her co-defendants, she acquired a real right over the lot and the house constructed thereon; that the auction sale as a result of the foreclosure of the chattel mortgage on the house was null and void for non-compliance with the form required by law. She prayed for the dismissal of the complaint and annulment of the sale.

Surety Company argued that as the lot on which the house was constructed did not belong to the spouses at the time the chattel mortgage was executed, the house might be considered as personal property, and they prayed that the said building be excluded from the real estate mortgage.

ISSUE:

As the building constructed thereon has been the subject of 2 mortgages; controversy arise as to which of these encumbrances should receive preference over the other.

RULING:

A building certainly cannot be divested of its character of realty by the fact that the land on which it is constructed belongs to another. In the case at bar, as personal properties could only be the subject of a chattel mortgage and as obviously the structure in question is not one, the execution of the chattel mortgage covering said building is clearly invalid and a nullity.

While it is true that said document was correspondingly registered in Chattel Mortgage Registry of Rizal, this act produced no effect whatsoever, for where the interest conveyed is in the nature of real property, the registration of the document in the registry of chattels is merely a futile act. Thus, the registration of the chattel mortgage of a building of strong materials produced no effect as far as the building is concerned. The building is subject to the real estate mortgage, in favour of Iya. Iya’s right to foreclose not only the land but also the building erected thereon is recognized.

5

G.R. No. L-40411 August 7, 1935

DAVAO SAW MILL CO., INC., plaintiff-appellant, vs. APRONIANO G. CASTILLO and DAVAO LIGHT & POWER CO., INC., defendants-appellees.

FACTS:

The Davao Sawmill Co. has operated a sawmill in the Province of Davao. However, the land upon which the business was conducted belonged to another person. On the land the sawmill company erected a building which housed the machinery used by it. Some of the implements thus used were clearly personal property, the conflict concerning machines which were placed and mounted on foundations of cement.

In the contract of lease, Davo Sawmill agreed to turn over free of charge all improvements and buildings erected by it on the premises with the exception of machineries, which shall remain with the Davao Sawmill. In an action brought by the Davao Light and Power Co., judgment was rendered against Davao Sawmill. A writ of execution was issued and the machineries placed on the sawmill were levied upon as personalty by the sheriff. Davao Light and

6

Power Co. proceeded to purchase the machinery and other properties auctioned by the sheriff.

ISSUE:

Whether or not the machineries are personal in nature.

RULING:

YES. It is machinery which is involved; moreover, machinery not intended by the owner of any building or land for use in connection therewith, but intended by a lessee for use in a building erected on the land by the latter to be returned to the lessee on the expiration or abandonment of the lease.

It was held that machinery which is movable in its nature only becomes immobilized when placed in a plant by the owner of the property or plant, but not when so placed by a tenant, a usufructuary, or any person having only a temporary right, unless such person acted as the agent of the owner.

In this case, the machinery placed in the sawmill is a movable property.

G.R. No. 168557 February 16, 2007

FELS ENERGY, INC., Petitioner, vs. THE PROVINCE OF BATANGAS and THE OFFICE OF THE PROVINCIAL ASSESSOR OF BATANGAS, Respondents.

FACTS:

NPC entered into a lease contract with Polar Energy, Inc. over 3x30 MW diesel engine power barges moored at Balayan Bay in Calaca, Batangas. The contract was for a period of five years.

In the agreement, NPC was made to shoulder any tax expenses. Subsequently, Polar Energy, Inc. assigned its rights under the Agreement to FELS. FELS received an assessment of real property taxes on the power barges. FELS referred the matter to NPC, reminding it of its obligation under the Agreement to pay all real estate taxes. It then gave NPC the full power and authority to represent

7

it in any conference regarding the real property assessment of the Provincial Assessor. NPC sought reconsideration to Provincial Assessor but was denied. LBAA affirmed provincial assessor while CBAA found the power barges exempt from real property tax, consequently reversed its own ruling. FELS & NPC separately filed a petition for review before CA.

ISSUE:

Whether or not power barges, which are floating and movable, are personal properties and therefore, not subject to real property tax.

RULING:

Power barges are real property and are thus subject to real property tax. Tax assessments by tax examiners are presumed correct and made in good faith, with the taxpayer having the burden of proving otherwise.

Article 415 (9) of the New Civil Code provides that "docks and structures which, though floating, are intended by their nature and object to remain at a fixed place on a river, lake, or coast" are considered immovable property. Thus, power barges are categorized as immovable property by destination, being in the nature of machinery and other implements intended by the owner for an industry or work which may be carried on in a building or on a piece of land and which tend directly to meet the needs of said industry or work.

G.R. No. 179987 September 3, 2013

HEIRS OF MARIO MALABANAN, (Represented by Sally A. Malabanan), Petitioners, vs. REPUBLIC OF THE PHILIPPINES, Respondent.

FACTS:

The property subject of the application for registration is a parcel of land situated in Barangay Tibig, Silang Cavite. Mario Malabanan filed an application for land registration covering the property in the RTC. Malabanan claimed that he had purchased the property from Eduardo Velazco, and that he and his predecessors-in-interest had been in open,

8

continuous, uninterrupted, public and adverse possession and occupation of the land for more than 30 years.

Velazco testified that the property was originally belonged to a twenty-two hectare property owned by his great-grandfather, Lino Velazco. Upon Lino’s death, his four sons inherited the property and divided it among themselves. But by 1966, Esteban’s wife, Magdalena, had become the administrator of all the properties inherited by the Velazco sons from their father, Lino. After the death of Esteban and Magdalena, their son Virgilio succeeded them in administering the properties, including Lot 9864-A, which originally belonged to his uncle, Eduardo Velazco. It was this property that was sold by Eduardo Velazco to Malabanan.

Among the evidence presented by Malabanan during trial was a Certification dated 11 June 2001, issued by the CENRO-DENR, which stated that the subject property was “verified to be within the Alienable or Disposable land per Land Classification Map No. 3013 established under Project No. 20-A and approved as such under FAO 4-1656 on March 15, 1982.” On 3 December 2002, the RTC approved the application for registration.

The Republic interposed an appeal to the Court of Appeals, arguing that Malabanan had failed to prove that the property belonged to the alienable and disposable land of the public domain, and that the RTC had erred in finding that he had been in possession of the property in the manner and for the length of time required by law for confirmation of imperfect title. On 23 February 2007, the Court of Appeals reversed the RTC ruling and dismissed the application of Malabanan.

ISSUE:

Whether or not a parcel of land classified as alienable and disposable be deemed private land and therefore susceptible to acquisition by prescription in accordance with the Civil Code.

RULING:

9

As a general rule and pursuant to the Regalian Doctrine, all lands of the public domain belong to the State and are inalienable. Lands that are not clearly under private ownership are also presumed to belong to the State and, therefore, may not be alienated or disposed.

In complying with Section 14(2) of the Property Registration Decree, consider that under the Civil Code, prescription is recognized as a mode of acquiring ownership of patrimonial property. However, public domain lands become only patrimonial property not only with a declaration that these are alienable or disposable. There must also be an express government manifestation that the property is already patrimonial or no longer retained for public service or the development of national wealth, under Article 422 of the Civil Code. And only when the property has become patrimonial can the prescriptive period for the acquisition of property of the public dominion begin to run.

The petitioners failed to present sufficient evidence to establish that they and their predecessors-in-interest had been in possession of the land since June 12, 1945. Without satisfying the requisite character and period of possession - possession and occupation that is open, continuous, exclusive, and notorious since June 12, 1945, or earlier - the land cannot be considered ipso jure converted to private property even upon the subsequent declaration of it as alienable and disposable. Prescription never began to run against the State, such that the land has remained ineligible for registration under Section 14(1) of the Property Registration Decree. Likewise, the land continues to be ineligible for land registration under Section 14(2) of the Property Registration Decree unless Congress enacts a law or the President issues a proclamation declaring the land as no longer intended for public service or for the development of the national wealth.

G.R. No. 186961 February 20, 2012

REPUBLIC OF THE PHILIPPINES, Petitioner, vs. EAST SILVERLANE REALTY DEVELOPMENT CORPORATION, Respondent.

10

FACTS:

The respondent filed with the RTC an application for land registration, covering a parcel of land identified as Lot 9039 of Cagayan Cadastre, situated in El Salvador, Misamis Oriental and with an area of 9,794 square meters.

The respondent purchased the portion of the subject property consisting of 4,708 square meters (Area A) from Francisca Oco and the remaining portion consisting of 5,086 square meters (Area B) from Rosario U. Tan Lim, Nemesia Tan and Mariano U. Tan.

It was claimed that the respondent’s predecessors-in-interest had been in open, notorious, continuous and exclusive possession of the subject property since June 12, 1945.

The RTC granted the application and ordered the Land Registration Authority to issue a decree in the name of the applicant East Silverlane Realty Development Corporation. The CA likewise affirmed the RTC's decision.

The petitioner assails the foregoing, alleging that the respondent failed to prove that its predecessors-in-interest possessed the subject property in the manner and for the length of time required under Section 48 (b) of Commonwealth Act No. 141, otherwise known as the "Public Land Act" (PLA), and Section 14 of Presidential Decree No. 1529, otherwise known as the "Property Registration Decree" (P.D. No. 1529). According to the petitioner, the respondent did not present a credible and competent witness to testify on the specific acts of ownership performed by its predecessors-in-interest on the subject property.

ISSUE:

Whether the respondent has proven itself entitled to the benefits of the PLA and P.D. No. 1529 on confirmation of imperfect or incomplete titles.

11

RULING:

The Court held that the evidence submitted by the respondent fell short of proving that it has acquired an imperfect title over the subject property under Section 48 (b) of the PLA.

Court ruled that possession and occupation of an alienable and disposable public land for the periods provided under the Civil Code do not automatically convert said property into private property or release it from the public domain. There must be an express declaration that the property is no longer intended for public service or development of national wealth.

Without such express declaration, the property, even if classified as alienable or disposable, remains property of the State, and thus, may not be acquired by prescription.

The respondent cannot register the subject property in its name since it was not established by the required quantum of evidence that the respondent and its predecessors-in-interest had been in open, continuous, exclusive and notorious possession of the subject property for the prescribed statutory period.

12

G.R. No. 147266 September 30, 2005

LUDO & LUYM DEVELOPMENT CORPORATION AND/OR CPC DEVELOPMENT CORPORATION, Petitioners, vs. VICENTE C. BARRETO as substituted by his heirs, namely: MAXIMA L. BARRETO, PEREGRINA B. UY, ROGELIO L. BARRETO, VIOLETA L. BARRETO, FLORENDA B. TEMPLANZA, EDUARDO L. BARRETO, EVELYN B. BERSAMIN, CECILIA B. AQUINO and NELSON NILO L. BARRETO, Respondent.

FACTS:

Vicente C. Barreto, as tenant of landowner Antonio Bartolome, worked on and cultivated two hectares of land devoted to sugarcane plantation. Antonio Bartolome sold the entire estate to LUDO with the latter absorbing all the farmworkers of the former. Vicente C. Barreto was designated as a co-overseer with Bartolome on the six-hectare coco land portion of the estate, pending the development of the entire estate into a residential-commercial complex. . It was agreed that the new owner, LUDO, Antonio Bartolome and complainant Vicente C. Barreto will share in the harvests.

Ludo decided to convert the entire estate into a residential-commercial complex and disturbance compensation was given to some of the farmworkers; some who refused to accept the same were eventually settled by compromise agreements. DAR then issued a conversion permit authorizing the conversion of the entire state into a residential/commercial lot.

Ten years later, CPC, the developer of the subject property, wrote the Secretary of the DAR to ask for the renewal of the conversion permit earlier issued to the owner. Barreto fervently opposed the move. CPC then formally informed Vicente C. Barreto of the termination of his employment as a co-overseer of the subject landholding.

13

In its decision, DARAB Regional Office found that there was no tenancy relationship existing between respondent LUDO and complainant Vicente C. Barreto, thus, no disturbance compensation was due the latter for having been dispossessed of the six-hectare landholding he had been tilling.

Barreto appealed the decision to the DARAB, who thereafter dismissed the appeal. During the pendency of the case, Barreto passed away and was substituted by his wife and children in the appeal. They subsequently filed a petition for review on certiorari before the Court of Appeals. The appellate court ruled in favor of petitioners-appellants heirs of Vicente C. Barreto and annulled and set aside the DARAB’s decision. CPC were ordered to pay the Barreto’s disturbance compensation.

ISSUE:

Whether or not there existed a tenancy relationship between petitioner LUDO and Vicente C. Barreto.

RULING:

YES. The issue of whether or not there exists a tenancy relationship between parties is best answered by law, specifically, The Agricultural Tenancy Act of the Philippines. Court held that the essential requisites of tenancy relationship are: the parties are the landholder and the tenant; the subject is agricultural land; there is consent; the purpose is agricultural production; and there is consideration. All of the requisites are indispensable in order to create or establish tenancy relationship between the parties. The intention of a tenant to surrender the landholding cannot be presumed by implication alone. Tenancy relations cannot be bargained away except for the strong reasons provided by law which must be convincingly shown by evidence.

In the case at bar, no one has denied the existence of the tenancy status of deceased Vicente C. Barreto over the subject thirty-six-hectare landholding with respect to its former owner, Antonio Bartolome. There being no waiver executed by deceased tenant Barreto, no less than the law clarifies that the existence of an agricultural tenancy relationship is not terminated by mere changes of

14

ownership, in cases of sale or transfer of legal possession as in lease.

When petitioner LUDO became the owner of the subject landholding, it became subrogated to the rights and obligations of its predecessor-in-interest, Antonio Bartolome, his obligation under the law to the deceased tenant, Vicente C. Barreto, continues and subsists until terminated as provided for by law.

The Court also held that the subject landholding was just merely reclassified and not converted. Reclassification is very much different from conversion. Conversion is the act of changing the current use of a piece of agricultural land into some other use as approved by the DAR. Reclassification, in contrast, is the act of specifying how agricultural lands shall be utilized for non-agricultural uses such as residential, industrial or commercial, as embodied in the land use plan, subject to the requirements and procedure for land use conversion.

Accordingly, a mere reclassification of agricultural land does not automatically allow a landowner to change its use and thus cause the ejectment of the tenants. Parties can still continue with their tenurial relationship even after such reclassification. He has to undergo the process of conversion before he is permitted to use the agricultural land for other purposes.

15

G.R. No. 157285 February 16, 2007

WOODRIDGE SCHOOL, INC., and MIGUELA JIMENEZ-JAVIER, Petitioners, vs. ARB CONSTRUCTION CO., INC., Respondent.

FACTS:

Woodridge is the usufructuary of a parcel of land in the name of spouses Ernesto T. Matugas and Filomena U. Matugas. Its co-petitioner, Miguela Jimenez-Javier, is the registered owner of the adjacent lot to that of Woodridge.

On the other hand, ARB is the owner and developer of Soldiers Hills Subdivision in Bacoor, Cavite, which is composed of four phases. Phase I of the subdivision was already accessible from the Marcos Alvarez Avenue. To provide the same accessibility to the residents of Phase II of the subdivision, ARB constructed the disputed road to link the two phases.

Petitioners' properties sit right in the middle of several estates. Initially, petitioners offered to pay ARB P50,000 as indemnity for the use of the road. ARB refused the offer and fenced the perimeter of the road fronting the properties of petitioners. By doing so, ARB effectively cut off petitioners'

16

access to and from the public highway. After failing to settle the matter amicably, Woodridge filed a complaint to enjoin ARB from depriving them of the use of the disputed subdivision road and to seek a compulsory right of way after payment of proper indemnity.

Trial court rendered its decision in favor of Woodridge. The appellate court reversed the decision of the lower court.

ISSUE:

Whether or not the road lots in a private subdivision are public property.

RULING:

The road lots in a private subdivision are private property, hence, the local government should first acquire them by donation, purchase, or expropriation, if they are to be utilized as a public road. Otherwise, they remain to be private properties of the owner-developer.

The use of the subdivision roads by the general public does not strip it of its private character. The road is not converted into public property by mere tolerance of the subdivision owner of the public's passage through it. The law is clear. The transfer of ownership from the subdivision owner-developer to the local government is not automatic but requires a positive act from the owner-developer before the city or municipality can acquire dominion over the subdivision roads. Therefore, until and unless the roads are donated, ownership remains with the owner-developer.

17

G.R. No. 152115 January 26, 2005

NIMFA USERO, petitioner, vs. COURT OF APPEALS and SPS. HERMINIGILDO & CECILIA POLINAR, respondents.

FACTS:

Petitioners and the private respondent are registered owners of neighboring parcels of land wherein between the lots is a low-level strip of land with stagnant body of water. Whenever there is a storm or heavy rain, the water therein would flood thereby causing damage to houses of the Polinars prompting them to build a concrete wall on the bank of the strip of land about 3meters from their house and riprapped the soil in that portion.

The Useros claimed ownership of the strip, demanded the halt of the construction but the Polinars never heeded believing that the strip is part of a creek. However, the Polinars offered to pay for the land. As the parties still failed to settle, both filed separate complaints for forcible entry.

18

The Municipal Trial Court ruled in favor of the petitioner, while the regional trial court reversed and ordered the dismissal of the complaint and confirmed the existence of the creek between the lots.

ISSUE:

Whether or not the disputed strip of land is part of the creek hence part of public domain.

RULING:

Yes. Art. 420 of the Philippine New Civil Code (NCC) provides for properties which are part of public domain. A creek is included in the phrase "and others of similar character". A creek, which refers to a recess or arm of a river, is a property belonging to the public domain, therefore not susceptible of private ownership. Being public water, it cannot be registered under the Torrens system under the name of any individual.

The phrase "others of similar character" includes a creek which is a recess or an arm of a river. It is property belonging to the public domain which is not susceptible to private ownership. Being public water, a creek cannot be registered under the Torrens System in the name of any individual.

G.R. No. 155051 May 29, 2007

RURAL BANK OF ANDA, INC., Petitioner, vs. ROMAN CATHOLIC ARCHBISHOP OF LINGAYEN- DAGUPAN, Respondent.

FACTS:

The lot in dispute is Cadastral Lot 736 (Lot 736) located in the Poblacion of Binmaley, Pangasinan.

The Sangguniang Bayan of Binmaley, Pangasinan, passed and approved Resolution Nos. 1045 and 105. Resolution No. 104 converted Lot 736 from an institutional lot to a commercial lot. Resolution No. 105 authorized the municipal mayor to enter into a contract of lease for 25

19

years with the Rural Bank of Anda over a portion of Lot 736 with an area of 252 square meters.

Fr. Arenos, the director of the seminary, discovered that a sawali fence was being constructed enclosing a portion of Lot 736. Mayor Domalanta and Fr. Arenos agreed that the construction of the building for the Rural Bank of Anda should be stopped. Respondent requested Mayor Domalanta to remove the sawali fence and restore the concrete fence. Later, Mayor Domalanta informed respondent that the construction of the building of the Rural Bank of Anda would resume but that he was willing to discuss with respondent to resolve the problem concerning Lot 736.

Respondent filed a complaint for Abatement of Illegal Constructions, Injunction and Damages with Writ of Preliminary Injunction in the Regional Trial Court of Lingayen, Pangasinan.

The trial court rendered a decision in favor of the plaintiff. The trial court found that Lot 736 is not covered by any Torrens title either in the name of respondent or in the name of the Municipality of Binmaley. The trial court held that Lot 736 is public in nature. Since Lot 736 is property of public dominion, it is outside the commerce of man. The CA affirmed the decision of the trial court.

ISSUE:

The issue in this case is whether Resolution Nos. 104 and 105 of the Sangguniang Bayan of Binmaley are valid.

RULING:

The records show that Lot 736 is used as a pathway going to the school, the seminary, or the church, which are all located on lots adjoined to Lot 736.14 Lot 736 was also used for parking and playground. In other words, Lot 736 was used by the public in general.

20

Both respondent and the Municipality of Binmaley failed to prove their right over Lot 736. Since Lot 736 has never been acquired by anyone through purchase or grant or any other mode of acquisition, Lot 736 remains part of the public domain and is owned by the state.

It is well settled "that no public land can be acquired by private persons without any grant, express or implied, from the government."

Municipal corporations cannot appropriate to themselves public or government lands without prior grant from the government. Since Lot 736 is owned by the state, the Sangguniang Bayan of Binmaley exceeded its authority in passing Resolution Nos. 104 and 105. Thus, Resolution Nos. 104 and 105 are void and consequently, the contract of lease between the Municipality of Binmaley and the Rural Bank of Anda over a portion of Lot 736 is also void.

G.R. No. 164584 June 22, 2009

PHILIP MATTHEWS, Petitioner, vs. BENJAMIN A. TAYLOR and JOSELYN C. TAYLOR, Respondents.

21

FACTS:

Benjamin Taylor, a British, was married to Joselyn Taylor, a Filipina. While their marriage was subsisting, Joselyn bought a Boracay property in consideration of P129,000 in 1989.The sale was allegedly financed by Benjamin. They constructed improvements thereon and eventually converted the property to a vacation and tourist resort known as the Admiral Ben Bow Inn.

However, Benjamin and Joselyn had a falling out. Joselyn executed a Special Power of Attorney (SPA) in favor of Benjamin, authorizing the latter to maintain, sell, lease, and sub-lease and otherwise enter into contract with third parties with respect to their Boracay property.

In 1992, Joselyn as lessor and petitioner Philip Matthews as lessee, entered into an Agreement of Lease involving the Boracay property for a period of 25 years. The agreement was signed by the parties and executed before a Notary Public. Petitioner thereafter took possession of the property and renamed the resort as Music Garden Resort.

Benjamin opposed the agreement and claims that it was null and void since it was entered into by Joselyn without his consent. Benjamin claimed that his funds were used in the acquisition and improvement of the Boracay property, and coupled with the fact that he was Joselyn’s husband, any transaction involving said property required his consent.

Matthews claimed good faith in transacting with Joselyn. Since Joselyn appeared to be the owner of the Boracay property, he found it unnecessary to obtain the consent of Benjamin. Moreover, as appearing in the Agreement, Benjamin signed as a witness to the contract, indicating his knowledge of the transaction and, impliedly, his conformity to the agreement entered into by his wife. Benjamin was, therefore, estopped from questioning the validity of the Agreement.

RTC rendered a judgment in favor of Benjamin. The RTC considered the Boracay property as community property of Benjamin and Joselyn; thus, the consent of the

22

spouses was necessary to validate any contract involving the property. On appeal to the CA, Matthews still failed to obtain a favorable decision. Hence, the petition for review on certiorari before the SC.

ISSUE:

Whether or not the consent of Benjamin is required in the Agreement of Lease.

RULING:

No. Sec. 7, Art. XII of the 1987 Constitution states that "aliens, whether individuals or corporation, have been disqualified from acquiring lands of the public domain."

The rule is clear and inflexible: aliens are absolutely not allowed to acquire public or private lands in the Philippines. The Court held that Benjamin has no right to nullify the Agreement of Lease between Joselyn and Matthews. Benjamin, being an alien, is absolutely prohibited from acquiring private and public lands in the Philippines even if he provided the funds for such acquisition. Considering that Joselyn appeared to be the designated "vendee" in the Deed of Sale of said property, she acquired sole ownership thereto. By entering into such contract knowing that it was illegal, no implied trust was created in his favor; no reimbursement for his expenses can be allowed; and no declaration can be made that the subject property was part of the conjugal/community property of the spouses.

23

G.R. No. 161107 March 12, 2013

HON. MA. LOURDES C. FERNANDO, in her capacity as City Mayor of Marikina City, JOSEPHINE C. EVANGELIST A, in her capacity as Chief, Permit Division, Office of the City Engineer, and ALFONSO ESPIRITU, in his capacity as City Engineer of Marikina City, Petitioners, vs. ST. SCHOLASTICA'S COLLEGE and ST. SCHOLASTICA'S ACADEMY-MARIKINA, INC., Respondents.

FACTS:

St. Scholastica’s College (SSC) and St. Scholastica’s Academy-Marikina, Inc. (SSA-Marikina) are educational institutions. SSC is the owner of four (4) parcels of land. The property is enclosed by a tall concrete perimeter fence built some thirty (30) years ago. Abutting the fence along the West Drive are buildings, facilities, and other improvements. The petitioners are the officials of the City Government of Marikina. The city enacted an ordinance regulating the construction of fences and walls. Section 3 of such ordinance limits the height of fences or walls on the front yard which shall be no more than one (1) meter in height. Fences in excess of one (1) meter shall be of an open fence type, at least eighty percent (80%) see-thru.

The SSC argued that the ordinance contravenes Section 1, Article III of the 1987 Constitution. That the implementation of the ordinance on their property would be tantamount to an appropriation of property without due process of law; and that the petitioners could only appropriate a portion of their property through eminent domain. They also pointed out that the goal of the provisions to deter lawless elements and criminality did not exist as the solid concrete walls of the school had served as sufficient protection for many years.

RTC rendered a decision in favor of SSC. RTC agreed with SSC that the order of the city to demolish the fence and to move it back six (6) meters would amount to an appropriation of property which could only be done through the exercise of eminent domain. It further found that the

24

80% see-thru fence requirement could run counter to the respondents’ right to privacy, considering that respondents were entitled to some sense of privacy in their affairs.

The CA affirmed the RTC decision. The CA reasoned out that the objectives stated in Ordinance No. 192 did not justify the exercise of police power, as it did not only seek to regulate, but also involved the taking of the respondents’ property without due process of law. Hence, the petitioners appealed before the SC.

ISSUE:

Whether or not the city ordinance is a valid exercise of police power.

RULING:

No. As with the State, local governments may be considered as having properly exercised their police power only if the following requisites are met: (1) the interests of the public generally, as distinguished from those of a particular class, require its exercise and (2) the means employed are reasonably necessary for the accomplishment of the purpose and not unduly oppressive upon individuals. In short, there must be a concurrence of a lawful subject and lawful method.

The Court joins the CA in finding that the real intent of the setback requirement was to make the parking space free for use by the public, considering that it would no longer be for the exclusive use of the respondents as it would also be available for use by the general public. Section 9 of Article III of the 1987 Constitution, a provision on eminent domain, provides that private property shall not be taken for public use without just compensation. Also, the implementation of the setback requirement would be tantamount to a taking of a total of 3,762.36 square meters of the respondents’ private property for public use without just compensation, in contravention to the Constitution.

The ultimate goal of this objective is clearly the prevention of crime to ensure public safety and security. The means employed by the petitioners, however, is not reasonably necessary for the accomplishment of this

25

purpose and is unduly oppressive to private rights. The petitioners have not adequately shown that an 80% see-thru fence would provide better protection and a higher level of security, or serve as a more satisfactory criminal deterrent, than a tall solid concrete wall.

The State may not, under the guise of police power, infringe on private rights solely for the sake of the aesthetic appearance of the community. Similarly, the Court cannot perceive how a see-thru fence will foster "neighborliness" between members of a community. Compelling the respondents to construct their fence in accordance with the assailed ordinance is, thus, a clear encroachment on their right to property, which necessarily includes their right to decide how best to protect their property.

G.R. No. 158687 January 27, 2006

FRISCO F. DOMALSIN, Petitioner, vs.

SPOUSES JUANITO VALENCIANO and AMALIA VALENCIANO, Respondents.

FACTS:

The property subject of this action for forcible entry is a parcel of land located at sitio Riverside, Camp 3, Tuba, Benguet. Frisco B. Domalsin claims to be the lawful owner and possessor of said parcel of land since 1979 up to the present. He declared it for taxation purposes and allegedly introduced improvements thereon. He was in continuous, adverse possession and in the concept of an owner for the past nineteen (19) years.

On August 1, 1998, Sps. Valenciano allegedly entered the premises to construct a building made of cement and strong materials without the authority and consent of Domalsin by means of force and strategy, and without a building permit from the DPWH.

Respondents claimed that the ongoing construction was with the consent and conformity of the DPWH and in fact the improvements found in the property were introduced by the residents thereof, including its first residents, William and Gloria Banuca, and not by Domalsin.

26

The premises on which petitioners Sps. Valenciano are constructing their house were leveled after the earthquake in 1990 by the Banuca spouses. Petitioners Sps. Valenciano are just starting the construction because the permission was only given now by Gloria Banuca.

What is being contested is the possession of a portion of the road-right-of way of Kennon Road which is located in front of a parcel of land that Domalsin bought by way of Deed of Waiver and Quitclaim from Castillo Binay-an.

MCTC and RTC ruled that Frisco F. Domalsin is the actual possessor of the lot in dispute and ordered the Sps. Valenciano to vacate and deliver the physical possession thereof to the petitioner. It held that petitioner had prior material possession over the subject land.

CA, however, reversed the decision. Hence, this appeal.

ISSUE:

Whether or not petitioner is the rightful owner of the subject property.

RULING:

Neither the petitioner nor the respondents can own nor possess the subject property the same being part of the public dominion. Properties of public dominion are owned by the general public. Public use is "use that is not confined to privileged individuals, but is open to the indefinite public." As the land in controversy is a portion of Kennon Road which is for the use of the people, there can be no dispute that same is part of public dominion. This being the case, the parties cannot appropriate the land for themselves. Thus, they cannot claim any right of possession over it.

The fact that the parties do not and cannot own the property under litigation does not mean that the issue to be resolved is no longer priority of possession. The determining factor for one to be entitled to possession will be prior physical possession and not actual physical

27

possession. Since title is never in issue in a forcible entry case, the Court of Appeals should have based its decision on who had prior physical possession. The main thing to be proven in an action for forcible entry is prior possession and that same was lost through force, intimidation, threat, strategy and stealth, so that it behooves the court to restore possession regardless of title or ownership.

G.R. No. 140798 September 19, 2006

MARCELITO D. QUEVADA, petitioner, vs. COURT OF APPEALS and JUANITO N. VILLAVERDE, respondents.

FACTS:

Juanito is the lessor of a parcel of land with a residential house in Sampaloc, Manila. Sometime in 1994, he and Quevada entered into a Contract of Lease of a portion of the residential house (consisting of 96 square meters) which is located on the subject property for the period from August 15, 1994 to August 15, 1995, at a monthly rental of P2,500. After expiration of the lease, they entered into another Contract of Lease, which was an extension of the previous date. After the expiration of the extended Lease, Marcelito continued possessing the premises, but without payment of any reasonable compensation.

28

Juanito made several demands to the petitioner to vacate the premises but was refused. Because of the Marcelito’s refusal to vacate the premises, Juanito referred the matter to the barangay court for conciliation, only for the former to repudiate the "agreement to vacate as of December 31, 1997." A notice to vacate the leased property was served upon Quevada. Quevada answered that he started building the house on the lot which was finished in 1986 at which time he occupied the house as his residence. Juanito in turn "advised" him that he would go ahead and buy the lot but with an assurance that as soon as Marcelito would be in a financial position to do payment, the former will transfer the title to the latter. Thus, a Lease Contract, in the meantime, was executed, for him to pay the rentals at P2,500 a month, but only with respect to the land, since the house belonged to him. Juanito did not give him a chance to pay the purchase price by setting a deadline to do the payment; similarly, he respondent refused to accept the monthly rental of the lot for P2,500.

Metropolitan Trial Court (MeTC) ruled in favor of Juanito. RTC and CA affirmed the decision of MeTC.

ISSUE:

1) Whether the action for ejectment is proper; 2) whether such action can be brought by private respondent who is not the titled owner of the property; 3) whether petitioner can be reimbursed for the value of the house on the property; and 4) whether there is an implied trust.

RULING:

As a lessor, Juanito was unlawfully deprived possession of the residential house after Marcelito's right to its possession as lessee had expired on April 15, 1996. Despite several demands given by the former to vacate the premises, the latter refused and even repudiated the agreement to vacate, which was entered into before the barangay court.

Petitioner's continued use and occupancy of the premises without any contract between him and private

29

respondent was by mere tolerance or permission of the latter. Possessory acts, no matter how long so continued, do not start the running of the period of prescription. Possession by tolerance is lawful, but such possession becomes unlawful when the possessor by tolerance refuses to vacate upon demand made by the owner. A person who occupies the land of another at the latter's tolerance or permission, without any contract between them, is necessarily bound by an implied promise to vacate upon demand, failing which, a summary action for ejectment is the proper remedy.

Its filing was within the one-year period after Juanito had been unlawfully deprived or withheld of its possession. The unlawful deprivation or withholding of possession started not from the date the lease contract expired, but from the date the written notice to vacate was served. Juanito may bring the action for unlawful detainer, even though he is not the titled owner of the leased property. The only issue to be resolved in unlawful detainer or desahucio is the actual physical or material possession of the property involved, independent of any claim of ownership by any of the party litigants.

Marcelito is necessarily in prior lawful possession of the property, but his possession eventually becomes unlawful upon termination or expiration of his right to possess. His prior physical possession of the leased property does not automatically entitle him to continue in its possession and does not give him a better right to the property.

Court held that petitioner should be paid for the value of the portion of the house covered by the lease, to be offset against rentals due. The fundamental doctrine of unjust enrichment is the transfer of value without just cause or consideration. Therefore, to have a just transfer of the leased portion of the house, its value should be offset against the reasonable rent due for its continued use and occupancy until the former vacates and surrenders it to the latter.

G.R. No. 131726 May 7, 2002

30

YOLANDA PALATTAO, petitioner, vs. THE COURT OF APPEALS, HON. ANTONIO J. FINEZA, as Presiding Judge of the Regional Trial Court of Caloocan City, Branch 131 and MARCELO CO, respondents.

FACTS:

Petitioner Yolanda Palattao entered into a lease contract whereby she leased to private respondent a house and a 490-square-meter lot located in 101 Caimito Road, Caloocan City. The duration of the lease contract was for three years. The contract gave Marcelo, respondent lessee, the first option to purchase the leased property.

During the last year of the contract, the parties began negotiations for the sale of the leased premises. Yolanda offered to sell to private respondent 413.28 square meters of the leased lot at P7,800.00 per square meter. Marcelo manifested his desire to buy the whole 490-square-meter leased premises and inquired from Yolanda the reason why only 413.28 square meters of the leased lot were being offered for sale.

Yolanda made a final offer to sell the lot at P7,500.00 per square meter with a down payment of 50% upon the signing of the contract of conditional sale. Marcelo accepted petitioner’s offer and reiterated his request for clarification as to the size of the lot for sale. Petitioner acknowledged private respondent’s acceptance of the offer in his letter dated November 10, 1993. Petitioner gave private respondent on or before November 24, 1993, within which to pay the 50% down payment in cash or manager’s check. Petitioner stressed that failure to pay the down payment on the stipulated period will enable petitioner to freely sell her property to others. Petitioner likewise notified private respondent that she is no longer renewing the lease agreement upon its expiration on December 31, 1993.

Marcelo did not accept the terms proposed by petitioner and wrote a letter to petitioner manifesting his intention to exercise his option to renew their lease contract for another three years. Yolanda declined to renew the lease and demanded that Marcelo vacate the premises but the latter refused.

31

Metropolitan Trial Court rendered a decision in favor of Yolanda. RTC, on the other hand, reversed the decision of MeTC. Petitioner filed a petition for review with the Court of Appeals, which dismissed the petition.

ISSUE:

Whether or not the ejectment case will prosper.

RULING:

Yes. Court held that the preservation of the status quo agreed upon by the parties applied only during the period of negotiations for an amicable settlement and cannot be construed to be effective for the duration of the pendency of the specific performance case.

An "accion publiciana" does not suspend an ejectment suit. An action for reconveyance of property or "accion reivindicatoria" also has no effect on ejectment suits regarding the same property.

In the case at bar, the continued occupation by private respondent of the leased premises is conditioned upon his right to acquire ownership over said property.

Contracts that are consensual in nature, like a contract of sale, are perfected upon mere meeting of the minds. Once there is concurrence between the offer and the acceptance upon the subject matter, consideration, and terms of payment, a contract is produced. The offer must be certain. While it is true that Marcelo informed Yolanda that he is accepting the latter’s offer to sell the leased property, it appears that they did not reach an agreement as to the extent of the lot subject of the proposed sale. Marcelo did not give his consent to buy only 413.28 square meters of the leased lot, as he desired to purchase the whole 490 square-meter-leased premises which, however, was not what was exactly proposed in petitioner’s offer. Therefore, private respondent’s acceptance of petitioner’s offer was not absolute, and will consequently not generate consent that would perfect a contract.

Considering that the lease contract was not renewed after its expiration on December 31, 1991, private

32

respondent has no more right to continue occupying the leased premises. Consequently, his ejectment therefrom must be sustained.

G.R. No. 157806 November 22, 2007

SPOUSES SHEIKDING BOOC and BILY BOOC, petitioners, vs. FIVE STAR MARKETING CO., INC., respondent.

FACTS:

Five Star Marketing Co., Inc. filed with the MTCC of Iligan City a Complaint for unlawful detainer against the spouses Sheikding and Bily Booc. Five Star is the owner of the land and building situated in Quezon Avenue, Iligan City. That Sps. Booc are the present occupants of the 3rd floor premises of the building, who were allowed to live temporarily in the premises for free.

Five Star notified all building occupants that it had withdrawn the privilege granted (rental free) to them coupled with a notice of rental rates in each premises concerned, and further required to any interested occupants to negotiate and sign a lease agreement or vacate the property otherwise. The petitioners were notified but ignored the demand and a letter of demand to vacate the premises was sent to the petitioners.

Sps. Booc contended that Five Star has no cause of action against them as they are actually the owners of the portion of the building that they are occupying; that the said property is owned in common by petitioner Sheikding and his brother, Rufino Booc.

MTCC rendered a judgment in favor of the Sps. Booc and against Five Star. RTC affirmed with modification the assailed decision of the MTCC. Aggrieved by the judgment

33

of the RTC, respondent filed a petition for review with the CA and the petition was granted.

ISSUE:

Whether or not the ejectment suit will prosper.

RULING:

Yes. Five Star has proved, by preponderance of evidence, its claim that it is the owner of the disputed properties and, therefore, has the right of material possession over the same.

Petitioners' claim of co-ownership is anchored on their assertion that it was petitioner Sheikding together with Rufino who actually purchased the subject lot; that they were also the ones who financed the construction of the subject building; and that they paid the taxes due on the subject properties. In claiming that the subject lot and building were bought and constructed with the money of petitioner Sheikding and Rufino, petitioners, in effect, aver that respondent is merely holding the property in trust for them.

As a rule, the burden of proving the existence of a trust is on the party asserting its existence and such proof must be clear and satisfactorily show the existence of the trust and its elements. Aside from the Joint Affidavit, no other competent evidence was presented to support petitioners' allegation of ownership of the lot in question.

It is settled that a certificate of title is a conclusive evidence of ownership; it does not even matter if the title is questionable, the instant action being an ejectment suit. In addition, the age-old rule is that the person who has a Torrens Title over a land is entitled to possession thereof.

34

G.R. No. 175561 October 20, 2010

SPOUSES IDA aka "MILAGROS" NIEVES BELTRAN and JOSE BELTRAN, Petitioners, vs. ANITA R. NIEVES, represented by NELIA G. MORAN, Respondent.

FACTS:

Nieves is the registered owner of the subject parcel of land as well as the house thereon. Milagros Beltran is Nieves’ niece, being the daughter of Gaston, Nieves’ brother. In asserting their ownership and rightful occupation against Nieves, petitioners spouses Beltran claim that Nieves sold the land and house to Gaston. The deed of sale, which Nieves disclaims having signed, remains unregistered.

Nieves filed an ejectment suit against Sps. Beltran which seeks to eject the latter from a 474 square meter parcel of land covered by Transfer Certificate of Title (TCT) No. T-10963 and the house erected thereon both registered in the name of petitioner.

35

Nieves claimed that for a long period of time, she tolerated the occupation of the subject property by her sister Julieta (Tita) Nieves and her brother Gaston Nieves and the latter’s children, respondent Milagros and Talin, among others. However, despite repeated demands, respondent Milagros and her husband refused to vacate the subject premises and surrender its possession to petitioner.

MCTC promulgated its decision in favor of the spouses Beltran and dismissed Nieves’ complaint. The MCTC made a provisional ruling that Gaston Nieves, Milagros Nieves’ father, owned the subject lot by virtue of the unregistered deed of sale by Nieves to Gaston. RTC affirmed the MCTC’s decision. The RTC held that the MCTC had no jurisdiction to entertain the question of just title. Moreover, an action for unlawful detainer should be filed within one year from the unlawful withholding of possession. Therefore, the complaint’s failure to allege with clarity the specific date that the withholding of possession became unlawful was a fatal jurisdictional lapse. Nieves then filed a Petition for Review before the CA.

CA ruled in favor of Nieves. CA ruled that the MCTC and the RTC erred in declaring that the Spouses Beltran are entitled to possess the subject property on the basis of the unregistered deed of sale. The tax declarations presented by the spouses Beltran are not conclusive evidences of ownership, but are good indicators of possession in the concept of an owner. The CA further ruled that a certificate of title is conclusive evidence of ownership. Nieves, who holds title to the land, is thus entitled to possession of the land.

ISSUES:

Won Spouses Beltran have no right over the property based on an unregistered deed of sale.

RULING:

Court agreed with the CA. Registered owners such as Nieves are entitled to the possession of the property covered by the title from the time such title was issued in their favor. The only issue in an ejectment case is the physical possession of real property ‒ possession de facto and not possession de jure. Court rules upon the issue of

36

ownership only to determine who between the parties has the better right of possession.

A person who occupies the land of another at the latter’s tolerance or permission, without any contract between them, is necessarily bound by an implied promise that he will vacate upon demand, failing which a summary action for ejectment is the proper remedy against them. Whatever right of possession that the spouses Beltran may have over the subject property cannot prevail over that of Nieves for the simple reason that Nieves is the registered owner of the subject property and the alleged deed of sale, which Nieves disputes, remains unregistered. Although it is true that the spouses Beltran, and not Nieves, were in prior physical possession of the subject property, this argument cannot hold water as prior physical possession is material only in forcible entry cases.

G.R. No. 105760 July 7, 1997

PHILIPPINE NATIONAL BANK, petitioner, vs. COURT OF APPEALS, HON. JUDGE OF THE REGIONAL TRIAL COURT OF GAPAN, NUEVA ECIJA, BR. 34, and NILDEFONSO MONTANO, respondents.

FACTS:

Sps. Crisanto de la Cruz and Pepita Montano mortgaged two parcels of land to petitioner PNB for a loan of P24,000.00. PNB extrajudicially foreclosed the mortgage

37

and was the only bidder at the public auction sale; thus, a Certificate of Sale over said lots was issued in favor of PNB.

PNB filed before the RTC a Petition for the Issuance of a Writ of Possession, alleging therein that by virtue of a foreclosure sale wherein it purchased the subject properties and due to the mortgagors' (spouses Crisanto de la Cruz and Pepita Montano) failure to redeem the property within a period of one year, it had become the absolute owner of the same and is entitled to a Writ of Possession. The petition was granted by the RTC.

Before implementation of the writ, Montano filed a Motion for the Dissolution of the Writ of Possession alleging that: (1) he was instituted as tenant on the subject property even before 1972 by the former owners of the land; (2) the two lots are the subject matters of CAR Case; (3) after the foreclosure of the subject land, Montano's counsel wrote PNB of the pending case between the mortgagors and private respondent as tenant on the land; (4) the issuance of said Writ in PNB's favor would work grave injustice to him and violate his rights under P.D. 27, P.D. 36, P.D. 583, and other laws and legal issuances on land reform; (5) he was issued a certification by the Cabiao-San Isidro Agrarian Reform Team that he is an agricultural lessee in the subject landholding and another certification that he is an active member of the Samahang Nayon; and (6) in line with the ruling in "Clapano vs. Gapultos" (132 SCRA 429) that possession of property is given to a purchaser in Extra-Judicial foreclosure unless a third-party is actually holding the property adversely to the judgment debtor, he is to be considered a "third person".

RTC granted Montano's motion to dissolve the writ of possession. The case was then referred to the CA. CA rendered judgment in favor of petitioner PNB then reversed its decision when Montano filed a motion thereby dissolving the Writ of Possession.

ISSUE:

WON petitioner PNB is entitled to a Writ of Possession of the land in question

38

RULING:

PNB is not entitled to a writ of possession, as the same may be issued in extrajudicial foreclosure of real estate mortgage only if the debtor is in possession and no third person had intervened. Such requisite is evidently lacking in the case at bar, as it has been established that Montano has been in possession and finally adjudged as the tenant on the landholding in question.

The decision in CAR proves that Montano is indeed a tenant of the landholding. The judgment in the agrarian suit is conclusive upon petitioner PNB.

Petitioner PNB further insists that as absolute owner of the properties, under Art. 428 and 429 of the New Civil Code, it has the right to possess and dispose of the same. These very provisions cited, however, show that the exercise of the rights of ownership is subject to limitations that may be imposed by law. Under Art. 428 of the Civil Code, the owner has the right to dispose of a thing without other limitations than those established by law. As an incident of ownership, therefore, there is nothing to prevent a landowner form donating his naked title to the land. However, the new owner must respect the rights of the tenant.

The Court further held that the agricultural lessee's rights are enforceable against the transferee or the landowner's successor-in-interest. Therefore, Montano may enforce his right of possession against petitioner PNB. Petitioner PNB may not, by way of defense, argue that its right over the land is superior to Montano's claim on the subject properties since the agricultural lease was not annotated on the Transfer Certificate of Title and, therefore, it dealt with the properties in good faith.

Even if the fact of tenancy had not been reflected on the title, PNB admitted that before they consented to the mortgage, an ocular inspection was conducted on the landholding on the occasion of which, PNB's Credit Investigator already found Montano staying on the land and even interviewed the latter. Upon the interview, Montano allegedly said that he had been allowed to stay on the

39

property in question because he was ejected from the adjacent parcel of land which he used to till.

The land being an agricultural one, and considering the ocular inspection conducted sometime in 1978 when P.D. 27 had been in effect for some time, petitioner PNB's suspicion that the land was tenanted should have been aroused by the existence of a farmer on the land other than the mortgagors themselves. It cannot be denied that PNB had been put on notice by its actual knowledge of another person possessing the land, no matter what the given reason may have been for private respondent Montano's occupancy of the properties in question.

Furthermore, as purchaser at a public auction, petitioner PNB was only substituted to and acquired the right, title, interest and claim of the judgment debtor or mortgagor to the property as of the time of the levy. In this case, the only remaining right of the mortgagors (spouses Crisanto de la Cruz and Pepita Montano) at the time of levy is the right to be paid a reasonable price for the land they owned as mandated by P.D. 27. That is the only right which petitioner PNB acquired as the new absolute owner of the land.

40

G.R. No. 175025 February 15, 2012

ROGELIO J. JAKOSALEM and GODOFREDO B. DULFO, Petitioners, vs. ROBERTO S. BARANGAN, Respondent.

FACTS:

Col. Roberto S. Barangan purchased a 300 square meter parcel of land; the old title was cancelled and a new one was issued in his name. Since then, he has been dutifully paying real property taxes for the said property. He was not, however, able to physically occupy the subject property because as a member of the Philippine Air Force, he was often assigned to various stations in the Philippines.

When he was about to retire from the government service, Barangan went to visit his property and he discovered that it was being occupied by petitioner Godofredo Dulfo and his family.

Barangan sent a letter to Dulfo demanding that he and his family vacate the subject property within 30 days. In reply, petitioner Atty. Rogelio J. Jakosalem, the son-in-law of petitioner Dulfo, sent a letter claiming ownership over the subject property.

Barangan commissioned a geodetic engineer to conduct a relocation survey of the subject property based on the technical description appearing on respondent Barangan’s TCT. The relocation survey revealed that the property occupied by Dulfo and his family is the same property covered by Barangan’s title.

Barangan filed a Complaint for Recovery of Possession with the RTC against Jakosalem. In their answer, Dulfo and Jakosalem claimed that the subject property was assigned to Jakosalem by Mr. Nicanor Samson; that they have been in possession of the subject property since May 8, 1979; and

41

that the property covered by respondent Barangan’s title is not the property occupied by petitioner Dulfo and his family.

RTC issued an order directing the engineer of DENR to conduct a survey of land; however, it did not push through because the defense its request for an ocular inspection claiming that it was no longer necessary.

RTC rendered a decision against Barangan for failure to present sufficient evidence to prove his claim. The RTC further said that even if the subject property is owned by Barangan, prescription and laches have already set in; thus, Barangan may no longer recover the same.

CA reversed the findings of the RTC. Hence, the appeal.

ISSUE:

Whether or not the property occupied by Dulfo is the same property claimed by Barangan;

WON laches and prescription barred the filing of the case.

RULING:

Barangan is entitled to recover the subject property. Article 434 of the Civil Code provides that "in an action to recover, the property must be identified, and the plaintiff must rely on the strength of his title and not on the weakness of the defendant’s claim." In other words, in order to recover possession, a person must prove (1) the identity of the land claimed, and (2) his title.

In this case, Barangan was able to prove the identity of the property and his title. To prove his title to the property, he presented in evidence the following documents: (1) Land Purchase Agreement; (2) Deed of Absolute Sale; (3) and a Torrens title registered under his name, TCT No. N-10772.

Records also show that during the trial, the RTC ordered the DENR to conduct a resurvey of the subject property; but petitioners moved that the same be

42

abandoned claiming that the resurvey would only delay the proceedings. The persistent refusal of petitioners to participate in the relocation survey does not speak well of their claim that they are not occupying respondent Barangan’s property.

The Court held that prescription and laches cannot apply to registered land covered by the Torrens system" because "under the Property Registration Decree, no title to registered land in derogation to that of the registered owner shall be acquired by prescription or adverse possession."

G. R. No. 185124 January 25, 2012

REPUBLIC OF THE PHILIPPINES, represented by the NATIONAL IRRIGATION ADMINISTRATION (NIA),Petitioner, vs. RURAL BANK OF KABACAN, INC., LITTIE SARAH A. AGDEPPA, LEOSA NANETTE AGDEPPA and MARCELINO VIERNES, MARGARITA TABOADA, PORTIA CHARISMA RUTH ORTIZ, represented by LINA ERLINDA A. ORTIZ and MARIO ORTIZ, JUAN MAMAC and GLORIA MATAS, Respondents.

FACTS:

National Irrigation Authority (NIA) is a government-owned-and-controlled corporation and is primarily responsible for irrigation development and management in the country. NIA was specifically authorized under P.D. 552 to exercise the power of eminent domain. NIA needed some parcels of land for the purpose of constructing the Malitubog-Marigadao Irrigation Project. It filed with the RTC of Kabacan, Cotabato a complaint for the expropriation of a portion of three (3) parcels of land.

After filing their amended complaints, NIA prayed that it be authorized to take immediate possession of the properties after depositing with the Philippine National

43

Bank the amount of P 19,246.58 representing the provisional value of the properties.

Respondents responded and alleged that NIA had no authority to expropriate portions of their land because it was not a sovereign political entity; that it was not necessary to expropriate their properties, because there was an abandoned government property adjacent to theirs, where the project could pass through. That NIA’s valuation of their expropriated properties was inaccurate because of the improvements on the land that should have placed its value at P 5 million; and that NIA never negotiated with the landowners before taking their properties for the project, causing permanent and irreparable damages to their properties valued at P 250,000.

RTC issued an order forming a committee tasked to determine the fair market value of the expropriated properties to establish the just compensation to be paid to the owners. Lower court issued a Writ of Possession in favor of NIA. The committee submitted its computation adding the value of the earth fill excavated from portions of Lot Nos. 3039 and 3080. Petitioner objected to the inclusion of the value of the excavated soil in the computation of the value of the land. RTC adopted the findings of the committee despite the objections of NIA to the inclusion of the value of the excavated soil in the computation of the value of the land.

NIA, through the Office of the Solicitor General, appealed to the Court of Appeals (CA) which affirmed with modification the RTC’s decision. CA deleted the value of the soil in determination of compensation but affirmed RTC’s valuation of the improvements made on the properties.

ISSUE:

Whether or not the value of the excavated soil should be included in the computation of just compensation.

RULING:

There is no legal basis to separate the value of the excavated soil from that of the expropriated properties, contrary to what the trial court did. In the context of expropriation proceedings, the soil has no value separate

44

from that of the expropriated land. Just compensation ordinarily refers to the value of the land to compensate for what the owner actually loses. Such value could only be that which prevailed at the time of the taking.

In National Power Corporation v. Ibrahim, et al. The SC held that rights over lands are indivisible. This conclusion is drawn from Article 437 of the Civil Code which provides: “The owner of a parcel of land is the owner of its surface and of everything under it, and he can construct thereon any works or make any plantations and excavations which he may deem proper, without detriment to servitudes and subject to special laws and ordinances. He cannot complain of the reasonable requirements of aerial navigation.” Thus, the ownership of land extends to the surface as well as to the subsoil under it.

Hence, the CA correctly modified the trial court’s Decision when it ruled it is preposterous that NIA will be made to pay not only for the value of the land but also for the soil excavated from such land when such excavation is a necessary phase in the building of irrigation projects. As pointed out by the OSG, the law does not limit the use of the expropriated land to the surface area only. To sanction the payment of the excavated soil is to allow the landowners to recover more than the value of the land at the time when it was taken, which is the true measure of the damages, or just compensation, and would discourage the construction of important public improvements.

G.R. No. 146259 September 13, 2007

FLORENTINO, TROADIO and PEDRO, all surnamed OCHOA, petitioners, vs. MAURO APETA and APOLONIA ALMAZAN, respondents.

FACTS:

Ochoa and their predecessors-in-interest have been occupying Lot No. 1580 consisting of 886 square meters situated in Malaban, Biñan, Laguna since 1910. The lot is

45

covered by a TCT No. T-40624 of the Registry of Deeds of that province. They built their houses and apartment building thereon.

Sometime in May 10, 1982, Mauro Apeta and Apolonia Almazan found that they are the true owners of Lot No. 1580 being occupied by Ochoas. Apeta filed with the RTC a complaint for recovery of possession and damages against Ochoa. Respondents alleged in that they are the lawful owners of Lot No. 1580 covered by Certificate of Title No. RT-599 (10731) issued by the Registry of Deeds of Laguna.

During the proceedings before the RTC, upon agreement of the parties, the trial judge commissioned Engr. Romulo Unciano of the Bureau of Lands of Region IV to conduct a resurvey of the disputed property. The result of the resurvey shows that Lot No. 1580, occupied by Ochoas, was registered in the name of Margarita Almada, Almazan’s predecessor-in-interest; and that the lot covered by TCT No. T-40624 is not Lot No. 1580, but Lot No. 1581 registered in the name of Servillano Ochoa, Ochoas’ predecessor-in-interest. This lot has been occupied by Isidro Jasmin.

Trial court rendered a decision in favor of respondents. CA affirmed the judgment of the RTC. Ochoa filed a Petition for Review on Certiorari.

ISSUE:

Whether or not petitioners were builders in good faith and be allowed to take possession of their improvements.

RULING:

Good faith is an intangible and abstract quality with no technical meaning or statutory definition, and it encompasses, among other things, an honest belief, the absence of malice and the absence of design to defraud or to seek an unconscionable advantage. It implies honesty of intention, and freedom from knowledge of circumstances which ought to put the holder upon inquiry. The essence of good faith lies in an honest belief in the validity of one’s right, ignorance of a superior claim and absence of intention to overreach another.

46

Applied to possession, one is considered in good faith if he is not aware that there exists in his title or mode of acquisition any flaw which invalidates it.

SC is convinced that Ochoa and their predecessors-in-interest were in good faith when they built their houses and apartment building on Lot No. 1580 since they were convinced it was covered by their TCT No. T-40624.

SC applied art. 448, 546, and 548 of the Civil Code. SC held that the landowner can make a choice - either by appropriating the building by paying the proper indemnity or obliging the builder to pay the price of the land. The choice belongs to the owner of the land, a rule that accords with the principle of accession that the accessory follows the principal and not the other way around. He must choose only one.

G.R. No. 177703 January 28, 2008

VILMA G. ARRIOLA and ANTHONY RONALD G. ARRIOLA, petitioners, vs. JOHN NABOR C. ARRIOLA, respondent.

47

FACTS:

John Nabor C. Arriola filed Special Civil Action with the RTC of Las Piñas City against Vilma G. Arriola and Anthony Ronald G. Arriola for judicial partition of the properties of decedent Fidel Arriola. John is the son of decedent Fidel with his first wife Victoria C. Calabia, while petitioner Anthony is the son of decedent Fidel with his second wife, petitioner Vilma.

RTC rendered a decision ordering the partition of the parcel of land left by the decedent Fidel S. Arriola by and among his heirs in equal shares of one-third (1/3) each without prejudice to the rights of creditors or mortgagees thereon.

The parties failed to agree on how to partition among them the land covered by subject land, John sought its sale through public auction, and petitioners acceded to it. Accordingly, the RTC ordered the public auction of the subject land. The public auction sale was scheduled on May 31, 2003 but it had to be reset when petitioners refused to include in the auction the house (subject house) standing on the subject land. This prompted respondent to file with the RTC an Urgent Manifestation and Motion for Contempt of Court, praying that petitioners be declared in contempt. RTC denied the motion.

John filed with the CA a Petition for Certiorari where he sought to have the RTC Orders set aside, and prayed that he be allowed to proceed with the auction of the subject land including the subject house. CA granted the petition. Hence, petitioners filed a Petition for Review on Certiorari before the SC.

ISSUE:

Whether or not the public auction should include the subject house.

RULING:

Yes. SC agrees with the CA that the subject house is covered by the judgment of partition for reasons that as the

48

deceased owned the subject land, he also owned the subject house which is a mere accessory to the land.

The right to accession is automatic (ipso jure), requiring no prior act on the part of the owner or the principal. So that even if the improvements including the house were not alleged in the complaint for partition, they are deemed included in the lot on which they stand, following the principle of accession. Consequently, the lot subject of judicial partition in this case includes the house which is permanently attached thereto; otherwise, it would be absurd to divide the principal, i.e., the lot, without dividing the house which is permanently attached thereto. While SC treats the subject house as part of the co-ownership of the parties, it stops short of authorizing its actual partition by public auction.

Following restrictions on its partition: first, that the heirs cannot extra-judicially partition it for a period of 10 years from the death of one or both spouses or of the unmarried head of the family, or for a longer period, if there is still a minor beneficiary residing therein; and second, that the heirs cannot judicially partition it during the aforesaid periods unless the court finds compelling reasons therefor. No compelling reason has been alleged by the parties; nor has the RTC found any compelling reason to order the partition of the family home, either by physical segregation or assignment to any of the heirs or through auction sale as suggested by the parties.

49

G.R. No. 170923 January 20, 2009

SULO SA NAYON, INC. and/or PHILIPPINE VILLAGE HOTEL, INC. and JOSE MARCEL E. PANLILIO,Petitioners, vs. NAYONG PILIPINO FOUNDATION, Respondent.

FACTS:

In 1975, respondent leased a portion of the Nayong Pilipino Complex, to petitioner Sulo sa Nayon, Inc. for the construction and operation of a hotel building, to be known as the Philippine Village Hotel. The lease was for an initial period of 21 years, or until May 1996. It is renewable for a period of 25 years under the same terms and conditions upon due notice in writing to respondent of the intention to renew.

In 1995, petitioners sent respondent a letter notifying the latter of their intention to renew the contract. July of the same year, parties agreed to the renewal of the contract for another 25 years, or until 2021. Under the new agreement, petitioner PVHI was bound to pay the monthly rentals.

Beginning January 2001, petitioners defaulted in the payment of their monthly rental. Respondent repeatedly demanded petitioners to pay the arrears and vacate the premises. MeTC rendered its decision in favor of respondent. RTC modified the ruling of the MeTC. RTC ordered Nayong Pilipino to submit a written manifestation of the option or choice it selected, i.e., to appropriate the improvements upon payment of proper indemnity or compulsory sale of the land whereon the hotel building of PVHI and related improvements or facilities were erected.

CA held that the RTC erroneously applied the rules on accession, as found in Articles 448 and 546 of the Civil Code. CA held that, “By and large, respondents are admittedly mere lessees of the subject premises and as such, cannot validly claim that they are builders in good

50

faith in order to solicit the application of Articles 448 and 546 of the Civil Code in their favor.”

ISSUE:

Whether or not Sulo sa Nayon as builders have acted in good faith in order for Art. 448 in relation to Art.546 of the Civil Code may apply with respect to their rights over improvements.

RULING:

Article 448 is manifestly intended to apply only to a case where one builds, plants, or sows on land in which he believes himself to have a claim of title and not to lands where the only interest of the builder, planter or sower is that of a holder, such as a tenant. In the case at bar, petitioners have no adverse claim or title to the land. In fact, as lessees, they recognize that the respondent is the owner of the land. What petitioners insist is that because of the improvements, which are of substantial value that they have introduced on the leased premises with the permission of respondent they should be considered builders in good faith who have the right to retain possession of the property until reimbursement by respondent.

SC affirms the ruling of the CA that introduction of valuable improvements on the leased premises does not give the petitioners the right of retention and reimbursement which rightfully belongs to a builder in good faith. Otherwise, such a situation would allow the lessee to easily "improve" the lessor out of its property. We reiterate the doctrine that a lessee is neither a builder in good faith nor in bad faith that would call for the application of Articles 448and 546 of the Civil Code. His rights are governed by Article 1678 of the Civil Code.

51

G.R. No. 157044 October 5, 2005

RODOLFO V. ROSALES, (represented by his heirs, Rodolfo, Jr., Romeo Allan, Lillian Rhodora, Roy Victor, Roger Lyle and Alexander Nicolai, all surnamed Rosales) and LILY ROSQUETA-ROSALES, Petitioners vs. MIGUEL CASTELLTORT, JUDITH CASTELLTORT, and LINA LOPEZ-VILLEGAS, assisted by her Attorney-in-Fact, Rene Villegas, Respondents.

FACTS:

Spouses Rodolfo V. Rosales and Lily Rosqueta-Rosales are the registered owners of a parcel of land designated as Lot 17, Block 1 of Subdivision Plan LRC Psd-55244 situated in Los Baños, Laguna.

On August 16, 1995, petitioners discovered that a house was being constructed on their lot, without their knowledge and consent, by respondent Miguel Castelltort. It turned out that respondents Castelltort and his wife Judith had purchased a lot, Lot 16 of the same Subdivision Plan, from respondent Lina Lopez-Villegas (Lina) through her son-attorney-in-fact Rene Villegas (Villegas) but that after a survey thereof by geodetic engineer Augusto Rivera, he pointed to Lot 17 as the Lot 16 the Castelltorts purchased.

52

Negotiations began, with Villegas offering a larger lot near petitioners’ lot in the same subdivision as a replacement thereof. In the alternative, Villegas proposed to pay the purchase price of petitioners’ lot with legal interest. Both proposals were, however, rejected by petitioners and directed Castelltort to stop the construction of and demolish his house and any other structure he may have built thereon, and desist from entering the lot.

Petitioners subsequently filed a complaint for recovery of possession and damages with prayer for the issuance of a restraining order and preliminary injunction against spouses-respondents Miguel and Judith Castelltort before the RTC of Calamba, Laguna. RTC held that there is no well-founded belief of ownership by the defendants of the land upon which they built their house. That they acted in bad faith and equally guilty of negligence which led to the construction of the defendants’ house on plaintiffs’ property and therefore jointly and severally liable for all the damages suffered by the plaintiffs.

CA reversed the decision of the trial court. CA held that RTC ignored the pivotal issue to be resolved in this case, i.e. whether appellant Miguel is a builder in good faith. Miguel relied on the title which the intervenor showed to him which, significantly, has no annotation that would otherwise show a prior adverse claim. Thus, as far as Miguel is concerned, his title over the subject lot, as well as the title of the intervenor thereto, is clean and untainted by an adverse claim or other irregularities. Hence, the petition.

ISSUE:

Whether or not the respondent acted in good faith and who has the right of option under Art. 448?

HELD:

Under the foregoing provision (Art 448), the landowner can choose between appropriating the building by paying the proper indemnity or obliging the builder to pay the price of the land, unless its value is considerably more than that of the structures, in which case the builder in good faith shall pay reasonable rent. If the parties cannot come to terms over the conditions of the lease, the court must fix the terms thereof.

53

The choice belongs to the owner of the land, a rule that accords with the principle of accession, i.e., that the accessory follows the principal and not the other way around. Even as the option lies with the landowner, the grant to him, nevertheless, is preclusive. The landowner cannot refuse to exercise either option and compel instead the owner of the building to remove it from the land.

In the case at bar, Castelltort’s good faith ceased on August 21, 1995 when petitioners personally apprised him of their title over the questioned lot. As held by the CA, should petitioners then opt to appropriate the house, they should only be made to pay for that part of the improvement built by Castelltort on the questioned property at the time good faith still existed on his part or until August 21, 1995.

The commencement of Castelltort’s payment of reasonable rent should start on August 21, 1995 as well, to be paid until such time that the possession of the property is delivered to petitioners, subject to the reimbursement of expenses, that is, if such option is for petitioners to appropriate the house.

G.R. No. 175399 October 27, 2009

OPHELIA L. TUATIS, Petitioner, vs. SPOUSES ELISEO ESCOL and VISMINDA ESCOL; HONORABLE COURT OF APPEALS, 22nd DIVISION, CAGAYAN DE ORO CITY; REGIONAL TRIAL COURT, BRANCH 11, SINDANGAN, ZAMBOANGA DEL NORTE; and THE SHERIFF OF RTC, BRANCH 11, SINDANGAN, ZAMBOANGA DEL NORTE, Respondents.

FACTS:

Sometime in November 1989, Visminda, as seller, and Tuatis, as buyer, entered into a Deed of Sale of a Part of a Registered Land by Installment. The subject matter of said Deed was a piece of real property situated in Poblacion,

54

Sindangan, Zamboanga del Norte being known as Lot No. 251.

Tuatis claimed that she had paid the entire purchase price of P10,000.00 and took possession of the subject property and constructed a residential building thereon. In 1996, Tuatis requested Visminda to sign a prepared absolute deed of sale covering the subject property, but the latter refused, contending that the purchase price had not yet been fully paid.

The parties tried to amicably settle the case before the Lupon Barangay, to no avail. Thus, Tuatis filed a Complaint for Specific Performance with Damages before the RTC. In her answer, Visminda countered that, except for the P3,000.00 downpayment and P1,000.00 installment paid by Tuatis on 19 December 1989 and 17 February 1990, respectively, Tuatis made no other payment to Visminda. Despite repeated verbal demands, Tuatis failed to comply with the conditions that she and Visminda agreed upon in the Deed of Sale by Installment for the payment of the balance of the purchase price for the subject property.

The RTC rendered a decision in favor of Visminda and likewise granted a motion for issuance of a Writ of Execution. Tuatis moved that the RTC issue an order allowing her to buy the subject property from Visminda. While Tuatis indeed had the obligation to pay the price of the subject property, she opined that such should not be imposed if the value of the said property was considerably more than the value of the building constructed thereon by Tuatis. Tuatis alleged that the value of the building she constructed was considerably more than that of the subject property. Tuatis maintained that she then had the right to choose between being indemnified for the value of her residential building or buying from Visminda the parcel of land subject of the case. Tuatis stated that she was opting to exercise the second option.

When the Sheriff enforced the Writ of Execution, Tuatis immediately filed with the CA a petition for the annulment of the RTC Order. Hence, Tuatis filed a petition to the SC principally arguing that Article 448 of the Civil

55

Code must be applied to the situation between her and Visminda.

ISSUE:

Whether or not Art. 448 of the Civil Code must be applied to this case.

RULING:

Taking into consideration the provisions of the Deed of Sale by Installment and Article 448 of the Civil Code, Visminda has the following options:

Under the first option, Visminda may appropriate for herself the building on the subject property after indemnifying Tuatis for the necessary and useful expenses the latter incurred for said building, as provided in Article 546 of the Civil Code. Until Visminda appropriately indemnifies Tuatis for the building constructed by the latter, Tuatis may retain possession of the building and the subject property.

Under the second option, Visminda may choose not to appropriate the building and, instead, oblige Tuatis to pay the present or current fair value of the land. The P10,000.00 price of the subject property, as stated in the Deed of Sale on Installment executed in November 1989, shall no longer apply, since Visminda will be obliging Tuatis to pay for the price of the land in the exercise of Visminda’s rights under Article 448 of the Civil Code, and not under the said Deed. Tuatis’ obligation will then be statutory, and not contractual, arising only when Visminda has chosen her option under Article 448 of the Civil Code.

If the present or current value of the land, the subject property herein, turns out to be considerably more than that of the building built thereon, Tuatis cannot be obliged to pay for the subject property, but she must pay Visminda reasonable rent for the same. Visminda and Tuatis must agree on the terms of the lease; otherwise, the court will fix the terms.

56

The Court highlights that the options under Article 448 are available to Visminda, as the owner of the subject property. There is no basis for Tuatis’ demand that, since the value of the building she constructed is considerably higher than the subject property, she may choose between buying the subject property from Visminda and selling the building to Visminda for P502,073.00. Again, the choice of options is for Visminda, not Tuatis, to make. And, depending on Visminda’s choice, Tuatis’ rights as a builder under Article 448 are limited to the following: (a) under the first option, a right to retain the building and subject property until Visminda pays proper indemnity; and (b) under the second option, a right not to be obliged to pay for the price of the subject property, if it is considerably higher than the value of the building, in which case, she can only be obliged to pay reasonable rent for the same.

57

G.R. No. 151815 February 23, 2005

SPOUSES JUAN NUGUID AND ERLINDA T. NUGUID, petitioners, vs. HON. COURT OF APPEALS AND PEDRO P. PECSON, respondents.

FACTS:

Pedro P. Pecson owned a commercial lot located at 27 Kamias Road, Quezon City, on which he built a four-door two-storey apartment building. For failure to pay realty taxes, the lot was sold at public auction by the City Treasurer of Quezon City to Mamerto Nepomuceno, who in turn sold it for P103,000 to the spouses Juan and Erlinda Nuguid.

Pecson challenged the validity of the auction sale before the RTC. RTC upheld the spouses’ title but declared that the four-door two-storey apartment building was not included in the auction sale. This was affirmed by the Court of Appeals and thereafter by the Supreme Court. As a result, the Nuguid spouses moved for delivery of possession of the lot and the apartment building.

The trial court ruled that the Spouses Nuguid were to reimburse Pecson for his construction cost of P53,000, following which, the spouses Nuguid were entitled to immediate issuance of a writ of possession over the lot and improvements. RTC also directed Pecson to pay the same amount of monthly rentals to the Nuguids as paid by the tenants occupying the apartment units or P21,000 per month and allowed the offset of the amount of P53,000 due from the Nuguids against the amount of rents collected by Pecson from the tenants of the apartment.

Pecson duly moved for reconsideration but the RTC issued a Writ of Possession directing the deputy sheriff to put the spouses Nuguid in possession of the subject property with all the improvements thereon and to eject all the occupants therein. Pecson then filed a special civil action for certiorari and prohibition with the Court of Appeals. The CA affirmed the order of payment of construction costs but rendered the issue of possession moot on appeal.

58

Pecson filed a petition for review before the Supreme Court. SC remanded the case to the trial court for it to determine the current market value of the apartment building on the lot. The determined value shall then be paid by the Sps. Nuguid to Pecson otherwise the latter shall be restored to the possession of the apartment building until payment of the required indemnity.

The parties have arrived at a compromise agreement that the value of the said improvement/building is P400,000. The plaintiff has already received P300,000.00. However, when Sps. Nuguid was ready to pay the balance of P100,000.00, the Pecson now insists that there should be a rental to be paid by Sps. Nuguid. The trial court scheduled for another hearing but directed Nuguid to pay Pecson the balance of P100,000.

After conducting a hearing, the lower court issued an order directing the spouses to pay the sum of P1,344,000 as reimbursement of the unrealized income of Pecson for the period beginning November 22, 1993 up to December 1997. The sum was based on the computation of P28,000/month rentals of the four-door apartment.

The Nuguid spouses filed a motion for reconsideration but this was denied for lack of merit. The Nuguid couple then appealed the trial court’s ruling to the Court of Appeals.

The CA reduced the rentals from P1,344,000 to P280,000 in favor of the Nuguid couple. The said amount represents accrued rentals from the determination of the current market value on January 31, 1997 until its full payment on December 12, 1997. Hence, Sps. Nuguid filed a petition for review on certiorari with the SC.

Sps. Nuguid argued that after reaching an agreed price of P400,000 for the improvements, they only made a partial payment of P300,000. Thus, they contend that their failure to pay the full price for the improvements will, at most, entitle Pecson to be restored to possession, but not to collect any rentals.

59

Pecson, on the other hand, points out that all he asked was that the value of the fruits of the property during the period he was dispossessed be accounted for since the prior decision of SC recognized that he was entitled to the property. In other words, he was entitled to rental income from the property.

ISSUE:

WON the Sps. Nuguid should reimburse Pecson for the benefits derived from the apartment building.

RULING:

YES. Under Article 448, the landowner is given the option, either to appropriate the improvement as his own upon payment of the proper amount of indemnity or to sell the land to the possessor in good faith. Relatedly, Article 546 provides that a builder in good faith is entitled to full reimbursement for all the necessary and useful expenses incurred; it also gives him right of retention until full reimbursement is made.

The right of retention is considered as one of the measures devised by the law for the protection of builders in good faith. Its object is to guarantee full and prompt reimbursement as it permits the actual possessor to remain in possession while he has not been reimbursed (by the person who defeated him in the case for possession of the property) for those necessary expenses and useful improvements made by him on the thing possessed.

The right of retention, which entitles the builder in good faith to the possession as well as the income derived therefrom, is already provided for under Article 546 of the Civil Code.

Sps. Nuguid committed a violation of Pecson's right of retention when they insisted on dispossessing him by filing

60

for a Writ of Possession to cover both the lot and the building despite his right of ownership over the apartment building. They also took advantage of the situation to benefit from the four-unit apartment building by collecting rentals thereon, before they paid for the cost of the apartment building. It was only four years later that they finally paid its full value to the respondent. Pecson is clearly entitled to payment by virtue of his right of retention over the said improvement.

Given the circumstances of the instant case where the builder in good faith has been clearly denied his right of retention for almost half a decade, SC find that the increased award of rentals by the RTC was reasonable and equitable. The Nuguid couple had reaped all the benefits from the improvement introduced by Pecson during said period, without paying any amount to the latter as reimbursement for his construction costs and expenses. They should account and pay for such benefits.

G.R. No. 140798 September 19, 2006

MARCELITO D. QUEVADA, petitioner, vs. COURT OF APPEALS and JUANITO N. VILLAVERDE, respondents.

FACTS:

Juanito is the lessor of a parcel of land with a residential house in Sampaloc, Manila. Sometime in 1994, he (as a lessor) and Quevada entered into a Contract of Lease of a portion of the residential house (consisting of 96 square meters) which is located on the subject property for the period from August 15, 1994 to August 15, 1995, at a monthly rental of P2,500.

After expiration of the lease, they entered into another Contract of Lease, which was an extension of the previous date. After the expiration of the extended Lease, Marcelito continued possessing the premises, but without payment of any reasonable compensation.

Juanito made several demands to the petitioner to vacate the premises but was refused; as a matter of fact, the petitioner refused to vacate without justifiable reason. Because of the Marcelito’s refusal to vacate the premises, Juanito referred the matter to the barangay court for

61

conciliation, only for the former to repudiate the "agreement to vacate as of December 31, 1997."

A notice to vacate the leased property was served upon Quevada. Quevada answered that he started building the house on the lot which was finished in 1986 at which time he occupied the house as his residence.

Juanito in turn "advised" him that he would go ahead and buy the lot but with an assurance that as soon as Marcelito would be in a financial position to do payment, the former will transfer the title to the latter. Thus, a Lease Contract, in the meantime, was executed, for him to pay the rentals at P2,500 a month, but only with respect to the land, since the house belonged to him. Juanito did not give him a chance to pay the purchase price by setting a deadline to do the payment; similarly, he respondent refused to accept the monthly rental of the lot for P2,500.

Metropolitan Trial Court (MeTC) ruled in favor of Juanito. RTC and CA affirmed the decision of MeTC.

ISSUE:

Whether or not petitioner ought to be reimbursed for the value of the house standing on the lot.

RULING:

YES. Marcelito should be paid for the value of the portion of the house covered by the lease, to be offset against rentals due.

Petitioner is not the owner of or claiming title to the land, but a mere tenant occupying only a portion of the house on it under the lease contract between him and private respondent. No supporting evidence was presented showing that petitioner's construction of the house was with the consent of the land's previous owner, but good faith should be presumed, particularly since the lease relationship was open and in plain view.

62

Neither is there a showing of bad faith in petitioner's refusal to vacate the land. On the other hand, there is no indication that private respondent will oblige petitioner to pay the price of the land. In fact, the former refuses to sell it to the latter. As a mere tenant, however, petitioner must pay reasonable rent for the continued use and occupancy of the leased premises from the time the lease contract expired until he finally vacates and surrenders it to private respondent.

It would not be fair for private respondent to receive both the rent and the portion of the house covered by the lease. As to its valuation, there is only petitioner's assertion that it cost P500,000. In order to satisfy the demands of substantial justice, morality, conscience, and fair dealing, and pursuant to equity and the principle proscribing unjust enrichment, the value of the portion of the house covered by the lease should be determined so that compensation of its value against the rentals due can take effect.

G.R. No. 185020 October 6, 2010

FILOMENA R. BENEDICTO, Petitioner, vs. ANTONIO VILLAFLORES, Respondent.

FACTS:

In 1980, Antonio Villaflores bought a portion of a property owned by her aunt Maria Villaflores and the thereafter took possession of said portion and thereon, constructed a house. By 1992, the remaining portion of Maria’s property was sold to Antonio. Both the 1980 and the 1992 sale was covered by a Deed of Absolute Sale but there was failure to register the same with Registry of Deeds (ROD).

63

By 1994, Maria resold said property previously sold to Antonio to Filomena Benedicto of which Filomena registered with the ROD, thus properly transferring the title of Maria Villaflores in the name of the Filomena. Thereafter, Filomena proceeded against Antonio for Accion Publiciana with Cancellation of Notice of Adverse Claim, Damages and Attorney’s Fees against Antonio. She alleged that she acquired Lot 2-A in 1994 from her grandaunt Maria by virtue of the Kasulatan ng Bilihang Tuluyan and she was not aware that Antonio had any claim or interest over the subject property.

Antonio traversed the complaint, asserting absolute ownership over Lot 2-A. He alleged that he purchased the subject property from Maria in 1980; and that he took possession of the same and constructed his house thereon. He came to know of the sale in favor of Filomena only in 2000 when the latter demanded that he vacate the property. He averred that Filomena was aware of the sale; hence, the subsequent sale in favor of Filomena was rescissible, fraudulent, fictitious, or simulated.

After trial, the RTC rendered a decision sustaining Filomena’s ownership. According to the RTC, Filomena was the one who registered the sale in good faith; as such, she has better right than Antonio. It rejected Antonio’s allegation of bad faith on the part of Filomena because no sufficient evidence was adduced to prove it.

The RTC found Antonio’s evidence of ownership questionable but declared him a builder in good faith. The CA affirmed the RTC for upholding Filomena’s ownership of Lot 2-A and for declaring Antonio a builder in good faith. However, it remanded the case to the RTC for further proceedings to determine the respective rights of the parties under Articles 448 and 546 of the Civil Code, and the amount due Antonio. Hence, a petition for review on certiorari before the SC. Filomena asserts that Antonio is not entitled to any reimbursement because he possessed the property by mere tolerance. Maria merely allowed Antonio to construct his house on a portion of Lot 2-A; hence, he is not entitled to any reimbursement or retention.

ISSUE:

64

Whether or not Antonio is a builder in good faith, thus entitled to reimbursements for the improvements he introduced to the property?

RULING:

Antonio is a builder in good faith and, as provided under Article 448, a landowner is given the option to either appropriate the improvement as his own upon payment of the proper amount of indemnity, or sell the land to the possessor in good faith. Relatedly, Article 546 provides that a builder in good faith is entitled to full reimbursement for all the necessary and useful expenses incurred; it also gives him right of retention until full reimbursement is made.

The objective of Article 546 of the Civil Code is to administer justice between the parties involved. The said provision was formulated in trying to adjust the rights of the owner and possessor in good faith of a piece of land, to administer complete justice to both of them in such a way as neither one nor the other may enrich himself of that which does not belong to him. Guided by this precept, it is therefore the current market value of the improvements which should be made the basis of reimbursement. A contrary ruling would unjustly enrich the private respondents who would otherwise be allowed to acquire a highly valued income-yielding four-unit apartment building for a measly amount. Consequently, the parties should therefore be allowed to adduce evidence on the present market value of the apartment building upon which the trial court should base its finding as to the amount of reimbursement to be paid by the landowner.

Thus, the SC denied Filomena’s petition and affirmed the CA ruling.

G.R. No. 150666 August 3, 2010

65

LUCIANO BRIONES and NELLY BRIONES, Petitioners, vs. JOSE MACABAGDAL, FE D. MACABAGDAL and VERGON REALTY INVESTMENTS CORPORATION,Respondents.

FACTS:

Respondent-spouses purchased from Vergon Realty Investments Corporation (Vergon) Lot No. 2-R, a 325-square-meter land located in Vergonville Subdivision No. 10 at Las Piñas City, Metro Manila. On the other hand, petitioners are the owners of Lot No. 2-S, which is adjacent to Lot No. 2-R.

Sometime in 1984, after obtaining the necessary building permit and the approval of Vergon, petitioners constructed a house on Lot No. 2-R which they thought was Lot No. 2-S. After being informed of the mix up by Vergon’s manager, respondent-spouses immediately demanded petitioners to demolish the house and vacate the property. Petitioners, however, refused to heed their demand. Thus, respondent-spouses filed an action to recover ownership and possession of the said parcel of land with the RTC of Makati City.

Petitioners insisted that the lot on which they constructed their house was the lot which was consistently pointed to them as theirs by Vergon’s agents. They interposed the defense of being buyers in good faith and impleaded Vergon as third-party defendant claiming that because of the warranty against eviction, they were entitled to indemnity from Vergon in case the suit is decided against them. The RTC ruled in favor of respondent-spouses and found that petitioners’ house was undoubtedly built on Lot No. 2-R and ordered the latter to immediately vacate the property.

On appeal, the CA affirmed the RTC’s finding that the lot upon which petitioners built their house was not the one which Vergon sold to them. The CA further ruled that petitioners cannot use the defense of allegedly being a purchaser in good faith for wrongful occupation of land.

ISSUE:

Whether or not petitioners were in good faith.

66

RULING:

YES. Article 527 of the Civil Code presumes good faith, and since no proof exists to show that the mistake was done by petitioners in bad faith, the latter should be presumed to have built the house in good faith.

When a person builds in good faith on the land of another, Article 448 of the Civil Code governs. Said article provides, that, the owner of the land on which anything has been built, sown or planted in good faith, shall have the right to appropriate as his own the works, sowing or planting, after payment of the indemnity provided for in Articles 546 and 548, or to oblige the one who built or planted to pay the price of the land, and the one who sowed, the proper rent. However, the builder or planter cannot be obliged to buy the land if its value is considerably more than that of the building or trees. In such case, he shall pay reasonable rent, if the owner of the land does not choose to appropriate the building or trees after proper indemnity. The parties shall agree upon the terms of the lease and in case of disagreement, the court shall fix the terms thereof.

The builder in good faith can compel the landowner to make a choice between appropriating the building by paying the proper indemnity or obliging the builder to pay the price of the land. The choice belongs to the owner of the land, a rule that accords with the principle of accession, i.e., that the accessory follows the principal and not the other way around. However, even as the option lies with the landowner, the grant to him, nevertheless, is preclusive. He must choose one. He cannot, for instance, compel the owner of the building to remove the building from the land without first exercising either option. It is only if the owner chooses to sell his land, and the builder or planter fails to purchase it where its value is not more than the value of the improvements, that the owner may remove the improvements from the land. The owner is entitled to such remotion only when, after having chosen to sell his land, the other party fails to pay for the same.

67

Moreover, petitioners have the right to be indemnified for the necessary and useful expenses they may have made on the subject property.

G.R. No. 153652 January 16, 2004

ALFREDO YASAY DEL ROSARIO, petitioner, vs. SPS. JOSE E. MANUEL and CONCORDIA MANUEL, represented by Attorney-in-fact, PATRICIA ARIOLA,respondents.

FACTS:

On August 12, 1999, spouses Jose and Concordia Manuel, respondents, filed with the MTC, San Mateo, Rizal a complaint for unlawful detainer against Alfredo Yasay del Rosario, petitioner. They alleged that they are the true and lawful owners of a 251 square meter lot located at Sta. Ana, San Mateo, Rizal. Because of their compassion, they allowed petitioner, whose house was destroyed by a strong typhoon, to occupy their lot. They agreed that he could build thereon a temporary shelter of light materials. But without their consent, what he constructed was a house of concrete materials.

In 1992, respondents asked petitioner to vacate the lot. This was followed by repeated verbal demands but to no avail, prompting them to bring the matter to the barangay. But the parties failed to reach an amicable settlement. On June 25, 1999, the barangay chairman issued a Certification to File Action.

In his answer to the complaint, petitioner claimed that sometime in 1968, respondents allowed him to build his house on the lot, provided he would guard the premises to prevent land grabbers and squatters from occupying the area. In 1995, when respondents visited this country, they agreed verbally to sell the portion on which his house was constructed. A year later, he made an offer to buy the 60 square meter portion occupied by him and to spend for its survey. But what respondents wanted to sell was the whole

68

area containing 251 square meters. He then informed them that he would first consult his children and they said they will wait. Instead, they filed the instant complaint.

ISSUE:

WON petitioner is a builder in good faith.

RULING:

NO. Petitioner is not a builder in good faith. Considering that he occupies the land by mere tolerance, he is aware that his occupation of the same may be terminated by respondents any time.

G.R. No. L-57348 May 16, 1985

FRANCISCO DEPRA, plaintiff-appellee, vs. AGUSTIN DUMLAO, defendant-appellant.

FACTS:

Francisco Depra is the owner of a parcel of land situated in the municipality of Dumangas, Iloilo, with an area of approximately 8,870 square meters. Agustin Dumlao, owns an adjoining lot with an approximate area of 231 sq. ms.

When Dumlao constructed his house on his lot, the kitchen thereof had encroached on an area of thirty four (34) square meters of Depra’s property, After the encroachment was discovered in a relocation survey of Depra’s lot made on November 2,1972, his mother, Beatriz Depra after writing a demand letter asking Dumlao to move back from his encroachment, filed an action for Unlawful Detainer. After trial, the Municipal Court found that Dumlao was a builder in good faith, and applying Article 448 of the Civil Code.

Ordering that a forced lease is created between the parties with the plaintiffs, as lessors, and the defendants as lessees, over the disputed portion with an area of thirty four (34) square meters, the rent to be paid is five (P5.00) pesos a month, payable by the lessee to the lessors within the first five (5) days of the month the rent is due; and the lease shall

69

commence on the day that this decision shall have become final.

Neither party appealed but Depra did not accept payment of rentals so that Dumlao deposited such rentals with the Municipal Court. Depra filed a Complaint for Quieting of Title against Dumlao before the then Court of First Instance of Iloilo involving the very same 34 square meters, which was the bone of contention in the Municipal Court. Dumlao, in his Answer, admitted the encroachment but alleged, in the main, that the present suit is barred by res judicata by virtue of the Decision of the Municipal Court, which had become final and executory.

Depra claims that the Decision of the Municipal Court was null and void ab initio because its jurisdiction is limited to the sole issue of possession, whereas decisions affecting lease, which is an encumbrance on real property, may only be rendered by Courts of First Instance.

ISSUE:

Whether or not the factual situations of DUMLAO and DEPRA conform to the juridical positions respectively defined by law, for a "builder in good faith" under Article 448, a "possessor in good faith" under Article 526 and a "landowner in good faith' under Article 448.

RULING:

ART. 448. The owner of the land on which anything has been built sown or planted in good faith, shall have the right to appropriate as his own the works, sowing or planting, after payment of the indemnity provided for in articles 546 and 548, or to oblige the one who built or planted to pay the price of the land, and the one who sowed, the proper rent. However, the builder or planter cannot be obliged to buy the land if its value is considerably more than that of the building or trees. In such case, he shall pay reasonable rent, if the owner of the land does not choose to appropriate the building or trees after proper indemnity. The parties shall agree upon the terms of the lease and in case of disagreement, the court shall fix the terms thereof.

70

In the case at bar, Depra has the option either to pay for the encroaching part of Dumlao's kitchen, or to sell the encroached 34 square meters of his lot to Dumlao. He cannot refuse to pay for the encroaching part of the building, and to sell the encroached part of his land, as he had manifested before the Municipal Court. But that manifestation is not binding because it was made in a void proceeding.

However, the good faith of Dumlao is part of the Stipulation of Facts in the Court of First Instance. It was thus error for the Trial Court to have ruled that Depra is "entitled to possession," without more, of the disputed portion implying thereby that he is entitled to have the kitchen removed. He is entitled to such removal only when, after having chosen to sell his encroached land, Dumlao fails to pay for the same. In this case, Dumlao had expressed his willingness to pay for the land, but Depra refused to sell.

G.R. No. 160453 November 12, 2012

REPUBLIC OF THE PHILIPPINES, Petitioner, vs. ARCADIO IVAN A. SANTOS III, and ARCADIO C. SANTOS, JR., Respondents.

FACTS:

Alleging continuous and adverse possession of more than ten years, Arcadio Ivan A. Santos III, with Arcadio C. Santos, Jr as co-applicant, applied for the registration of property in the RTC. The property, was bounded in the Northeast by Lot belonging to Arcadio Jr., in the Southeast by the Parañaque River, in the Southwest by an abandoned road, and in the Northwest by Lot owned by Arcadio Ivan.

71

He alleged that the property had been formed through accretion and had been in their joint open, notorious, public, continuous and adverse possession for more than 30 years.

The City of Parañaque (the City) opposed the application for land registration, stating that it needed the property for its flood control program; that the property was within the legal easement of 20 meters from the river bank; and that assuming that the property was not covered by the legal easement, title to the property could not be registered in favor of the applicants for the reason that the property was an orchard that had dried up and had not resulted from accretion.

The RTC granted the application for land registration. On appeal, CA affirmed the RTC.

ISSUE:

Whether or not Article 457 of the Civil Code was applicable herein.

RULING:

SC declared the disputed lot as exclusively belonging to the State for being part of the dried-up bed of the Paranaque River.

Article 457 of the Civil Code provides that "to the owners of lands adjoining the banks of rivers belong the accretion which they gradually receive from the effects of the currents of the waters." The application by both lower courts of Article 457 of the Civil Code was erroneous in the face of the fact that respondents’ evidence did not establish accretion, but instead the drying up of the Parañaque River.

Applicants for land registration carried the burden of proof to establish the merits of their application by a preponderance of evidence, by which is meant such evidence that is of greater weight, or more convincing than that offered in opposition to it. They would be held entitled to claim the property as their own and apply for its registration under the Torrens system only if they

72

established that, indeed, the property was an accretion to their land.

Accretion is the process whereby the soil is deposited along the banks of rivers. The deposit of soil, to be considered accretion, must be: (a) gradual and imperceptible; (b) made through the effects of the current of the water; and (c) taking place on land adjacent to the banks of rivers.

The RTC and the CA grossly erred in treating the dried-up river bed as an accretion that became respondents’ property pursuant to Article 457 of the Civil Code. That land was definitely not an accretion. The process of drying up of a river to form dry land involved the recession of the water level from the river banks, and the dried-up land did not equate to accretion.

The State exclusively owned the disputed Lot and may not be divested of its right of ownership. Article 502 of the Civil Code expressly declares that rivers and their natural beds are public dominion of the State. It follows that the river beds that dry up, continue to belong to the State as its property of public dominion, unless there is an express law that provides that the dried-up river beds should belong to some other person.

Since property of public dominion is outside the commerce of man and not susceptible to private appropriation and acquisitive prescription, the adverse possession which may be the basis of a grant of title in the confirmation of an imperfect title refers only to alienable or disposable portions of the public domain. It is only after the Government has declared the land to be alienable and disposable agricultural land that the year of entry, cultivation and exclusive and adverse possession can be counted for purposes of an imperfect title.

73

G.R. No. 68166 February 12, 1997

HEIRS OF EMILIANO NAVARRO, petitioner, vs. INTERMEDIATE APPELLATE COURT & HEIRS OF SINFOROSO PASCUAL, respondents.

FACTS:

Sinforoso Pascual owns a tract of land which lies between Talisay River on the eastern side and Bulacan River on the western side. In front of his land, on the northern side, is the disputed land where there use to lay the Manila Bay. Pascual filed an application to register and confirm his title to that parcel of land claiming that the land is an accretion to his property.

Emiliano Navarro, a lessee and in possession of a part of the subject property, opposed Pascual's application. Navarro claimed that the land sought to be registered has always been part of the public domain, it being a part of the foreshore of Manila Bay.

During the pendency of the trial, Emiliano Navarro died and was substituted by his heirs. Subsequently, Pascual died and was substituted by his heirs.

On 1975, the court a quo rendered judgment finding the subject property to be foreshore land and, being a part of the public domain, it cannot be the subject of land registration proceedings. On appeal, the CA reversed the findings of the court a quo and granted the petition for registration of the subject property.

The appellate court explained that the subject land is an accretion. That the Manila Bay could not have been the cause of the deposit thereat for in the natural course of things, the waves of the sea eat the land on the shore. That it is more logical that the two rivers flowing towards the bay emptied their cargo of sand, silt and clay at their mouths, thus causing land to accumulate therein.

ISSUE

74

Whether or not the disputed land is foreshore land.

RULING:

SC sided with the conclusion formed by the trial court that the disputed land is part of the foreshore of Manila Bay and therefore, part of the public domain.

First, the disputed land used to adjoin, border or front the Manila Bay and not any of the two rivers whose torrential action is to account for the accretion on the land. Sulpicio Pascual testified in open court that the waves of Manila Bay used to hit the disputed land being part of the bay's foreshore but, after he had planted palapat and bakawan trees thereon in 1948, the land began to rise. If the accretion were to be attributed to the action of either or both of the Talisay and Bulacan Rivers, the alluvium should have been deposited on either or both of the eastern and western boundaries of Pascual's own tract of land, not on the northern portion thereof which is adjacent to the Manila Bay. Clearly lacking, is the third requisite of accretion, which is, that the alluvium is deposited on the portion of claimant's land which is adjacent to the river bank.

Accretion as a mode of acquiring property under said Article 457, requires the concurrence of the following requisites:

1. That the accumulation of soil or sediment be gradual and imperceptible;

2. That it be the result of the action of the waters of the river; and

3. That the land where the accretion takes place is adjacent to the bank of the river.

Accretion is the process whereby the soil is deposited, while alluvium is the soil deposited on the estate fronting the river bank; the owner of such estate is called the riparian owner. Riparian owners are, strictly speaking, distinct from littoral owners, the latter being owners of lands bordering the shore of the sea or lake or other tidal

75

waters. The alluvium, by mandate of Article 457 of the Civil Code, is automatically owned by the riparian owner from the moment the soil deposit can be seen but is not automatically registered property, hence, subject to acquisition through prescription by third persons.

Second, there is no dispute as to the fact that the disputed land adjoins the Manila Bay. Manila Bay is obviously not a river. The disputed property is an accretion on a sea bank, Manila Bay being an inlet or an arm of the sea.

G.R. No. 146616 August 31, 2006

SIAIN ENTERPRISES, INC., Petitioner,vs. F.F. CRUZ & CO., INC., Respondent.

FACTS:

On September 18, 1973, Western Visayas Industrial Corporation (WESVICO) filed a foreshore lease application over the foreshore land adjacent to Lot 3308, registered in its name located at Loboc, Lapuz, Lapaz, Iloilo City. The petition for registration was archived by Court of first instance now RTC because WESVICO could no longer be contacted. Development Bank of the Philippines (DBP) foreclosed the lease application where WESVICO ceased to hold its operation. On July 7, 1983, F.F. Cruz & Co filed with the Bureau of Lands to lease the application over the foreshore land, which was inspected by its District Land Officer, that F.F. Cruz occupied as a sanctuary of its marine equipment used and utilized in the repair and maintenance of its equipment for loading and unloading in the construction of Iloilo City Port.

On September 29, 1986, SIAIN Enterprises Inc. purchased from DBP the properties previously owned by WESVICO including Lot 3309 over to the adjacent foreshore land. Upon knowing that F.F. Cruz overlapped 130 linear meters, SIAIN filed a protest, Sangguniang Panglunsod of Iloilo City and Land Management Bureau favor the “mutual interest ” settling SIAIN to have 70 linear meters and F.F. Cruz to have 60 linear meters of the disputed area. SIAIN appealed to DENR, which they ruled that the area is a

76

natural foreshore, SIAIN is the riparian / littoral owner that should enjoy the preferential right to lease the entire 130 linear meters adjoining the property.

ISSUE:

Whether or not the disputed area is reclaimed land or foreshore land?

Whether or not SIAIN has preferential right to lease the foreshore area?

RULING:

The Supreme Court held in favor of SIAIN as ruled in par 32 of Lands Administrative Order No. 7-1 which states that: Preference of Riparian Owner – the owner of the property adjoining foreshore lands or lands covered with water bordering upon shores or banks of navigable lakes or rivers, shall be given preference to apply for such lands as may not be needed for public service and in consonance with Art. 4 Spanish Law of Waters of 1866 which provide that, Lands added to the shores by accretions and alluvial deposits caused by the action of the sea form part of the public domain.

In the case at bar, the disputed area is classified as a foreshore area subject to the preferential right to lease of the littoral owner. The reason for the preferential right is the same justification for giving accretions to the riparian owner for the diminutions which his land suffers by reason of destructive force of the waters. So, he who loses by the encroachments of the sea should gain by its recession. Such foreshore area existed even before F.F. Cruz undertook its reclamation. It was formed by accretions or alluvial deposits due to the action of the sea, citing the case of Santulan vs. Executive Secretary. WESVICO does not waived or abandoned its preferential right to lease the disputed area filed by its application for registration that was archived due to financial reasons and denied by the trial court, WESVICO is not barred to file a new foreshore lease application.

Being part of the public domain, ownership of the area could not be acquired by WESVICO, but the preferential right remained. Therefore, the decision of Court of Appeals

77

was reversed and set aside and reinstated the DENR decision.

G.R. No. 116290 December 8, 2000

DIONISIA P. BAGAIPO, petitioner, vs. THE HON. COURT OF APPEALS and LEONOR LOZANO, respondents.

FACTS:

Petitioner Dionisia P. Bagaipo is the registered owner of Lot No. 415, a 146,900 square meter agricultural land situated in Ma-a, Davao City. Respondent Leonor Lozano is the owner of a registered parcel of land located across and opposite the southeast portion of petitioner’s lot facing the Davao River. Lozano acquired and occupied her property in 1962 when his wife inherited the land from her father who died that year.

Bagaipo filed a complaint for Recovery of Possession with Mandatory Writ of Preliminary Injunction and Damages against Lozano for: (1) the surrender of possession by Lozano of a certain portion of land measuring 29,162 square meters which is supposedly included in the area belonging to Bagaipo; and (2) the recovery of a land area measuring 37,901 square meters which Bagaipo

78

allegedly lost when the Davao River traversed her property. Bagaipo contended that as a result of a change in course of the said river, her property became divided into three lots, namely: Lots 415-A, 415-B and 415-C.

Bagaipo also presented Godofredo Corias, a former barangay captain and long-time resident of Ma-a to prove her claim that the Davao River had indeed changed its course. Corias testified that the occurrence was caused by a big flood in 1968 and a bamboo grove which used to indicate the position of the river was washed away. The river which flowed previously in front of a chapel located 15 meters away from the riverbank within Bagaipo’s property now flowed behind it. Corias was also present when Magno conducted the relocation survey in 1988.

For his part, Lozano insisted that the land claimed by Bagaipo is actually an accretion to their titled property. He asserted that the Davao River did not change its course and that the reduction in Bagaipo’s domain was caused by gradual erosion due to the current of the Davao River. Lozano added that it is also because of the river’s natural action that silt slowly deposited and added to his land over a long period of time. He further averred that this accretion continues up to the present and that registration proceedings instituted by him over the alluvial formation could not be concluded precisely because it continued to increase in size.

The trial court conducted an ocular inspection. It concluded that the applicable law is Article 457 and rendered a decision against Bagaipo. The CA affirmed the decision of the trial court. Hence, the appeal.

ISSUE:

Whether or not respondent own Lot 415-C in accordance with the principle of accretion under Article 457.

RULING:

YES. The trial court and the appellate court both found that the decrease in land area was brought about by erosion and not a change in the river’s course. These findings are

79

factual, thus conclusive on the Court, unless there are strong and exceptional reasons, or they are unsupported by the evidence on record, or the judgment itself is based on a misapprehension of facts.

The decrease in petitioner’s land area and the corresponding expansion of respondent’s property were the combined effect of erosion and accretion respectively. Art. 461 of the Civil Code is inapplicable. Petitioner cannot claim ownership over the old abandoned riverbed because the same is inexistent. The riverbed’s former location cannot even be pinpointed with particularity since the movement of the Davao River took place gradually over an unspecified period of time, up to the present.

The rule is well-settled that accretion benefits a riparian owner when the following requisites are present: 1) That the deposit be gradual and imperceptible; 2) That it resulted from the effects of the current of the water; and 3) That the land where accretion takes place is adjacent to the bank of the river. These requisites were sufficiently proven in favor of respondents. In the absence of evidence that the change in the course of the river was sudden or that it occurred through avulsion, the presumption is that the change was gradual and was caused by alluvium and erosion.

Petitioner did not demonstrate that Lot 415-C allegedly comprising 29,162 square meters was within the boundaries of her titled property. The survey plan commissioned by petitioner which was not approved by the Director of Lands was properly discounted by the appellate court.

G.R. No. 168970 January 15, 2010

CELESTINO BALUS, Petitioner, vs. SATURNINO BALUS and LEONARDA BALUS VDA. DE CALUNOD, Respondents.

FACTS:

The petitioner and respondents in this case are the children of Mr. Rufo (the owner of parcel land mortgaged in the bank).

80

Rufo mortgaged a parcel of land and all the improvement thereto as a security for a loan he obtained from the bank; unfortunately Rufo failed to pay his loan. The mortgaged property was foreclosed and subsequently sold to the bank as the sole bidder at public auction held for that purpose. The property was not redeemed within the period allowed by law (two years after the auction) the new titled issued in favor of the bank. Petitioner and respondents executed extrajudicial settlement of estate.

Three years after executing the extrajudicial settlement, the respondents bought the subject property from the bank and the deed of sale executed under the name of respondents. Meanwhile petitioner continued possession of the subject lot as a result respondents filed a complaint for recovery of possession and damages against petitioner contending that the respondents is the new owner of the subject property but petitioner still refused to surrender the possession.

The RTC held that the right of petitioner to purchase from respondents his share in the disputed property was recognized by the provision of extrajudicial settlement. Respondents as aggrieved by the decision of the RTC filed an appeal to the CA.

CA reversed and set aside the decision of the RTC and ordered petitioner to immediately surrender possession of the subject property to the respondents. The CA ruled that when petitioner and respondents did not redeem the subject property within the redemption period and allowed the consolidation of ownership and the issuance of a new title in the name of the Bank, their co-ownership was extinguished.

ISSUE:

Whether or not co- ownership among the petitioner and respondents over the property continued to exist even after the transfer of title to the bank.

HELD:

The main issue raised by petitioner is whether co-ownership by him and respondents over the subject property persisted even after the lot was purchased by the

81

Bank and title thereto transferred to its name, and even after it was eventually bought back by the respondents from the Bank.

Petitioner posits that the subject Extrajudicial Settlement is, in and by itself, a contract between him and respondents, because it contains a provision whereby the parties agreed to continue their co-ownership of the subject property by "redeeming" or "repurchasing" the same from the Bank. This agreement, petitioner contends, is the law between the parties and, as such, binds the respondents.

SC held that since Rufo lost ownership of the subject property during his lifetime, it only follows that at the time of his death, the disputed parcel of land no longer formed part of his estate to which his heirs may lay claim. Stated differently, petitioner and respondents never inherited the subject lot from their father.

For petitioner to claim that the Extrajudicial Settlement is an agreement between him and his siblings to continue what they thought was their ownership of the subject property, even after the same had been bought by the Bank, is stretching the interpretation of the said Extrajudicial Settlement too far.

In the first place, as earlier discussed, there is no co-ownership to talk about and no property to partition, as the disputed lot never formed part of the estate of their deceased father.

82

G.R. No. 170829 November 20, 2006

PERLA G. PATRICIO, Petitioner, vs. MARCELINO G. DARIO III and THE HONORABLE COURT OF APPEALS, Second Division, Respondents.

FACTS:

On July 5, 1987, Marcelino V. Dario died leaving his wife, petitioner Perla G. Patricio and their two sons, Marcelino Marc Dario and private respondent Marcelino G. Dario III as his legal heirs. Among the properties he left was a parcel of land with a residential house and a pre-school building built thereon. Petitioner and Marcelino Marc formally advised private respondent of their intention to partition the subject property and terminate the co-ownership. Private respondent refused to partition the property hence petitioner and Marcelino Marc instituted an action for partition before the Regional Trial Court of Quezon City. The trial court ordered the partition of the subject property in the following manner: Perla G. Patricio, 4/6; Marcelino Marc G. Dario, 1/6; and Marcelino G. Dario III, 1/6. The trial court also ordered the sale of the property by public auction wherein all parties concerned may put up their bids. In case of failure, the subject property should be distributed accordingly in the aforestated manner.

Private respondent filed a motion for reconsideration which was denied by the trial court hence he appealed before the Court of Appeals, which denied the same. However, upon a motion for reconsideration, the appellate court partially reconsidered. In the now assailed Resolution, the Court of Appeals dismissed the complaint for partition filed by petitioner and Marcelino Marc for lack of merit. It held that the family home should continue despite the death of one or both spouses as long as there is a minor beneficiary thereof. The heirs could not partition the property unless the court found compelling reasons to rule otherwise. The appellate court also held that the minor son of private respondent, who is a grandson of spouses Marcelino V. Dario and Perla G. Patricio, was a minor beneficiary of the family home.

ISSUE:

83

Whether partition of the family home is proper where one of the co-owners refuse to accede to such partition on the ground that a minor beneficiary still resides in the said home.

RULING:

The law explicitly provides that occupancy of the family home either by the owner thereof or by any of its beneficiaries must be actual.

Article 154 of the Family Code enumerates who are the beneficiaries of a family home: (1) The husband and wife, or an unmarried person who is the head of a family; and (2) Their parents, ascendants, descendants, brothers and sisters, whether the relationship be legitimate or illegitimate, who are living in the family home and who depend upon the head of the family for legal support. To be a beneficiary of the family home, three requisites must concur: (1) they must be among the relationships enumerated in Art. 154 of the Family Code; (2) they live in the family home; and (3) they are dependent for legal support upon the head of the family.

Moreover, Article 159 of the Family Code provides that the family home shall continue despite the death of one or both spouses or of the unmarried head of the family for a period of 10 years or for as long as there is a minor beneficiary, and the heirs cannot partition the same unless the court finds compelling reasons therefor. This rule shall apply regardless of whoever owns the property or constituted the family home.

In the case at bar, private respondents minor son, who is also the grandchild of deceased Marcelino V. Dario satisfies the first and second requisites. However, as to the third requisite, Marcelino Lorenzo R. Dario IV cannot demand support from his paternal grandmother if he has parents who are capable of supporting him.

84

There is no showing that private respondent is without means to support his son; neither is there any evidence to prove that petitioner, as the paternal grandmother, was willing to voluntarily provide for her grandsons legal support. On the contrary, herein petitioner filed for the partition of the property which shows an intention to dissolve the family home, since there is no more reason for its existence after the 10-year period ended in 1997. With this finding, there is no legal impediment to partition the subject property.

G.R. No. 176858 September 15, 2010

HEIRS OF JUANITA PADILLA, represented by CLAUDIO PADILLA, Petitioners, vs. DOMINADOR MAGDUA, Respondent.

FACTS:

Juanita Padilla (Juanita), the mother of petitioners, owned a piece of land located in San Roque, Tanauan, Leyte. After Juanita’s death on 23 March 1989, petitioners, as legal heirs of Juanita, sought to have the land partitioned. Petitioners sent word to their eldest brother Ricardo Bahia (Ricardo) regarding their plans for the partition of the land. In a letter dated 5 June 1998 written by Ricardo addressed to them, petitioners were surprised to find out that Ricardo had declared the land for himself, prejudicing their rights as co-heirs. It was then discovered that Juanita had allegedly executed a notarized Affidavit of Transfer of Real Property (Affidavit) in favor of Ricardo on 4 June 1966 making him the sole owner of the land. The records do not show that the land was registered under the Torrens system.

Petitioners filed an action with the RTC of Tacloban City for recovery of ownership, possession, partition and damages. Petitioners sought to declare void the sale of the land by Ricardo’s daughters, Josephine Bahia and Virginia Bahia-Abas, to respondent Dominador Magdua (Dominador). The sale was made during the lifetime of Ricardo. Petitioners alleged that Ricardo, through

85

misrepresentation, had the land transferred in his name without the consent and knowledge of his co-heirs.

RTC dismissed the case for lack of jurisdiction. Petitioners filed a motion; RTC denied the motion for reconsideration and dismissed the case on the ground of prescription.

ISSUE:

Whether or not the present action is already barred by prescription.

RULING:

Ricardo and petitioners are co-heirs or co-owners of the land. Co-heirs or co-owners cannot acquire by acquisitive prescription the share of the other co-heirs or co-owners absent a clear repudiation of the co-ownership, as expressed in Article 494 of the Civil Code which states:

Art. 494. No prescription shall run in favor of a co-owner or co-heir against his co-owners or co-heirs as long as he expressly or impliedly recognizes the co-ownership.

Since possession of co-owners is like that of a trustee, in order that a co-owner’s possession may be deemed adverse to the cestui que trust or other co-owners, the following requisites must concur: (1) that he has performed unequivocal acts of repudiation amounting to an ouster of the cestui que trust or other co-owners, (2) that such positive acts of repudiation have been made known to the cestui que trust or other co-owners, and (3) that the evidence thereon must be clear and convincing.

In the present case, all three requisites have been met. After Juanita’s death in 1989, petitioners sought for the partition of their mother’s land. The heirs, including Ricardo, were notified about the plan. Ricardo, through a letter dated 5 June 1998, notified petitioners, as his co-heirs, that he adjudicated the land solely for himself. Accordingly, Ricardo’s interest in the land had now become adverse to the claim of his co-heirs after repudiating their

86

claim of entitlement to the land. In order that title may prescribe in favor of one of the co-owners, it must be clearly shown that he had repudiated the claims of the others, and that they were apprised of his claim of adverse and exclusive ownership, before the prescriptive period begins to run.

However, in the present case, the prescriptive period began to run only from 5 June 1998, the date petitioners received notice of Ricardo’s repudiation of their claims to the land. Since petitioners filed an action for recovery of ownership and possession, partition and damages with the RTC on 26 October 2001, only a mere three years had lapsed. This three-year period falls short of the 10-year or 30-year acquisitive prescription period required by law in order to be entitled to claim legal ownership over the land. Thus, Dominador cannot invoke acquisitive prescription.

G.R. No. 122047 October 12, 2000

SPOUSES SERAFIN SI AND ANITA BONODE SI, petitioners, vs. COURT OF APPEALS, SPOUSES JOSE ARMADA and REMEDIOS ALMANZOR (deceased, and substituted by heirs: Cynthia Armada, Danilo Armada and Vicente Armada) respondents.

FACTS:

The 340 square meters of land, situated in San Jose District, Pasay City, the property in dispute, originally belonged to Escolastica, wife of Severo Armada, Sr. During the lifetime of the spouses, the property was transferred to their children and the Registry of Deeds, Pasay City, issued TCT No. 16007 in the names of the three sons, as follows: DR. CRISOSTOMO R. ARMADA, married to Cresenciana V. Alejo, 113.34 Square Meters; JOSE R. ARMADA, married to

87

Remedios Almanzor, 113.33 Square Meters; and DR. SEVERO R. ARMADA, Jr., single.

Annotated also in the title is the total cancellation of said title by virtue of the Deed of Sale conveying 113.34 square meters of the property herein, in favor of ANITA BONODE SI, married to Serafin D. Si, for the sum of P75,000.00, issuing in lieu thereof Transfer Certificate of Title.

On April 15, 1980, herein spouses Jose Armada and Remedios Almanzor, filed a complaint for Annulment of Deed of Sale and Reconveyance of Title with Damages, against herein petitioners Anita and Serafin Si and Conrado Isada, brother-in-law of Cresenciana. Isada brokered the sale. The complaint alleged that Conrado Isada sold Crisostomo’s share by making it appear that Cresenciana, the attorney-in-fact of her husband, is a Filipino citizen. By this time, Crisostomo and Cresenciana had migrated and were already citizens of the United States of America.

Petitioners contend that since the property was already three distinct parcels of land, there was no longer co-ownership among the brothers. Hence, Jose and Severo, Jr. had no right of redemption when Crisostomo sold his share to the spouses Si.

The Court of Appeals reversed the decision of the Regional Trial Court of Pasig City and nullified the sale of the subject lot by the spouses Crisostomo and Cresenciana Armada to spouses Serafin and Anita Si.

Petitioners filed their motion for new trial under Section 1, Rule 53 of the Revised Rules of Court. Petitioners presented new evidence, registered in the name of Escolastica de la Rosa, married to Severo Armada. CA ruling for the new trial, respondent court denied the motion, reasoning that when the motion was filed, the reglementary period had lapsed and the decision had become final and executory. Petitioners’ motion for reconsideration of said resolution was denied.

ISSUE:

88

Whether or not private respondents are co-owners who are legally entitled to redeem the lot under Article 1623 of the Civil Code.

RULING:

No, respondents are not co-owners since the brothers had a physical division of the lot, the community ownership terminated and right of redemption for each brother was no longer available.

Rightfully, as early as October 2, 1954, the lot in question had already been partitioned when their parents executed three (3) deed of sales in favor of Jose, Crisostomo and Severo, all surnamed Armada. Notably, every portion conveyed and transferred to the three sons was definitely described and segregated and with the corresponding technical description. In short, this is what we call extrajudicial partition. Moreover, every portion belonging to the three sons has been declared for taxation purposes.

The fact that the three portions are embraced in one certificate of title does not make said portions less determinable or identifiable or distinguishable, one from the other, nor that dominion over each portion less exclusive, in their respective owners. Hence, no right of redemption among co-owners exists. After the physical division of the lot among the brothers, the community ownership terminated, and the right of preemption or redemption for each brother was no longer available.

Under Art. 484 of the Civil Code, there is co-ownership whenever the ownership of an undivided thing or right belongs to different persons. There is no co-ownership when the different portions owned by different people are already concretely determined and separately identifiable, even if not yet technically described.

G.R. No. 125233 March 9, 2000

Spouses ALEXANDER CRUZ and ADELAIDA CRUZ, petitioners, vs. ELEUTERIO LEIS, RAYMUNDO LEIS,

89

ANASTACIO L. LAGDANO, LORETA L. CAYONDA and the HONORABLE COURT OF APPEALS, respondents.

FACTS:

Adriano and Gertrudes were married on 19 April 1923. On 27 April 1955, Gertrudes acquired from the then DANR a parcel of land with an area of one hundred (100) square meters, situated at Bo. Sto. Niño, Marikina, Rizal. The Deed of Sale described Gertrudes as a widow. On 2 March 1956, TCT No. 43100 was issued in the name of "Gertrudes Isidro," who was also referred to therein as a "widow." On 2 December 1973, Adriano died and it does not appear that he executed a will before his death.

On 5 February 1985, Gertrudes obtained a loan from petitioners, the spouses Alexander and Adelaida Cruz, in the amount of P15,000.00. The loan was secured by a mortgage over the property covered by TCT No. 43100. Gertrudes, however, failed to pay the loan on the due date.

Unable to pay her outstanding obligation after the debt became due and payable, Gertrudes executed two contracts in favor of petitioner Alexander Cruz. The first is denominated as "Kasunduan" which the parties concede is a pacto de retro sale, granting Gertrudes one year within which to repurchase the property. The second is a "Kasunduan ng Tuwirang Bilihan," a Deed of Absolute Sale covering the same property for the price of P39,083.00, the same amount stipulated in the "Kasunduan."

For failure of Gertrudes to repurchase the property, ownership thereof was consolidated in the name of Alexander Cruz and a new TCT title was issued in his name. When Gertrudes Isidro died, her heirs, herein private respondents, received demands to vacate the premises from petitioners, the new owners of the property. Private respondents responded by filing a complaint seeking the nullification of the contracts of sale over a lot executed by Gertrudes Isidro before the RTC.

The RTC rendered a decision in favor of private respondents. The RTC held that the land was conjugal property since the same was acquired during the marriage

90

of the spouses and that Adriano contributed money for the purchase of the property. Thus, the court concluded, Gertrudes could only sell to petitioner spouses her one-half share in the property. Petitioners appealed to the CA in vain. The Court of Appeals affirmed the decision of the RTC.

ISSUE:

Whether or not a co-owner may acquire exclusive ownership over the property held in common.

RULING:

No. The right of repurchase may be exercised by a co-owner with respect to his share alone. While the records show that petitioner redeemed the property in its entirety, shouldering the expenses therefor, that did not make him the owner of all of it. In other words, it did not put to end the existing state of co-ownership. There is no doubt that redemption of property entails a necessary expense. The result is that the property remains to be in a condition of co-ownership. It is conceded that, as a rule, a co-owner such as Gertrudes could only dispose of her share in the property owned in common. Article 493 of the Civil Code provides:

Art. 493. Each co-owner shall have the full ownership of his part of the fruits and benefits pertaining thereto, and he may therefore alienate, assign or mortgage it, and even substitute another person in its enjoyment, except when personal rights are involved. But the effect of the alienation or the mortgage, with respect to the co-owners, shall be limited to the portion which may be allotted to him in the division upon the termination of the co-ownership.

Unfortunately for private respondents, the property was registered solely in the name of "Gertrudes Isidro, widow." Where a parcel of land, forming past of the undistributed properties of the dissolved conjugal partnership of gains, is sold by a widow to a purchaser who merely relied on the face of the certificate of title thereto, issued solely in the name of the widow, the purchaser acquires a valid title to the land even as against the heirs of the deceased spouse. The rationale for this rule is that "a person dealing with registered land is not required to go behind the register to determine the condition of the

91

property. He is only charged with notice of the burdens on the property which are noted on the face of the register or the certificate of title. To require him to do more is to defeat one of the primary objects of the Torrens system.

G.R. No. 168943 October 27, 2006

IGLESIA NI CRISTO, petitioner, vs. HON. THELMA A. PONFERRADA, in her capacity as Presiding Judge, Regional Trial Court, Br. 104, Quezon City, and HEIRS OF ENRIQUE G. SANTOS, respondents.

FACTS:

Enrique Santos was the alleged owner with his TCT No. 57272 issued by the Register of Deeds on July 27, 1961. After his death, he was survived by his wife, Alicia Santos, and their children Alfredo, Roberto, Enrique and Susan. Thereafter, they took peaceful and adverse possession of the property.

In February 1996, Iglesia ni Cristo (INC) was claiming ownership over the property based on TCT No. 321744 issued on September 18, 1984. Santoses had the property fenced but INC deprived them of the final use and enjoyment of their property.

On 2001, the Santoses, represented by Enrique G. Santos, filed a complaint for Quieting of Title and/or Accion Reinvindicatoria before the RTC of Quezon City against the INC. INC moved to dismiss complaint on the grounds that the action (either Quieting of Title or Accion Reinvindicatoria) had prescribed, having been filed only on October 24, 2001 beyond the statutory ten-year period.

The case involved more than one plaintiff but the verification and certification against forum shopping incorporated in the complaint was signed only by Enrique Santos; there having no showing that Enrique Santos was, authorized to so represent the other plaintiffs to file the complaint and to sign the verification and certification of non-forum shopping.

92

The trial court denied INC’s motion to dismiss. Unsatisfied, INC filed a Petition for Certiorari and Prohibition before the CA. The CA dismissed the petition.

ISSUE:

Whether or not Enrique Santos can represent his co-owners with or without authority in an action for the recovery of possession of real property.

Whether or not an action to quiet title prescribe from dispossession.

RULING:

First issue: Yes. As heirs, the Santoses, they are considered co-owners pro indiviso of the whole property since no specific portion yet has been adjudicated to any of them. Consequently, as one of the heirs and principal party, the lone signature of Enrique G. Santos in the verification and certification is sufficient for the RTC to take cognizance of the case. The commonality of their interest gave Enrique G. Santos the authority to inform the RTC on behalf of the other plaintiffs therein that they have not commenced any action or claim involving the same issues in another court or tribunal, and that there is no other pending action or claim in another court or tribunal involving the same issues.

As co-owners, each of the heirs may properly bring an action for ejectment, forcible entry and detainer, or any kind of action for the recovery of possession of the subject properties. Thus, a co-owner may bring such an action, even without joining all the other co-owners as co-plaintiffs, because the suit is deemed to be instituted for the benefit of all.

Second issue: No. An action for quieting of title is imprescriptible until the claimant is ousted of his possession. The owner of a real property, as plaintiff, is entitled to the relief of quieting of title even if, at the time of the commencement of his action, he was not in actual possession of real property. Under Article 477 of the New Civil Code, the owner need not be in possession of the property.

93

G.R. No. 122904 April 15, 2005

ADORACION E. CRUZ, THELMA DEBBIE E. CRUZ, GERRY E. CRUZ and NERISSA CRUZ-TAMAYO, Petitioner, vs. THE HONORABLE COURT OF APPEALS, SUMMIT FINANCING CORP., VICTOR S. STA. ANA, MAXIMO C. CONTRERAS, RAMON G. MANALASTAS, and VICENTE TORRES, Respondents.

FACTS:

Petitioners and Arnel Cruz were co-owners of a parcel of land situated in Taytay, Rizal. Yet the property was registered only in the name of Arnel Cruz. According to petitioners, the property was among the properties they and Arnel Cruz inherited upon the death of Delfin Cruz, husband of Adoracion Cruz.

Petitioners and Arnel Cruz executed a Deed of Partial Partition, distributing to each of them their shares consisting of several lots previously held by them in common. Among the properties adjudicated to defendant Cruz was the parcel of land covered at the time by TCT No. 495225 which is the subject of this case.

Subsequently, the same parties to the Deed of Partial Partition agreed in writing to share equally in the proceeds of the sale of the properties although they had been

94

subdivided and individually titled in the names of the former co-owners pursuant to the Deed of Partial Partition.

On June 4, 1980, a Real Estate Mortgage was constituted on the disputed property then covered by TCT No. 495225 to secure the loan obtained by Arnel Cruz thru Nelson Tamayo from respondent Summit. Since the loan had remained outstanding on maturity, Summit instituted extrajudicial foreclosure proceedings, and at the foreclosure sale it was declared the highest bidder.

A Certificate of Sale was issued to respondent Summit, which more than a year later, consolidated its ownership of the foreclosed property. In their complaint before the RTC, petitioners asserted that they co-owned the properties with Arnel Cruz and argued that the mortgage was void since they did not consent to it.

The trial court rendered a judgment in favor of the plaintiffs. The CA reversed the trial court's decision.

ISSUE:

Whether or not the real estate mortgage on the property then covered by TCT No. 495225 is valid.

RULING:

YES. Resolution of the issue in turn depends on the determination of whether the mortgaged property was the exclusive property of Arnel Cruz when it was mortgaged.

Co-ownership is terminated upon judicial or extra-judicial partition of the properties owned in common. Partition, in general, is the separation, division and assignment of a thing held in common among those to whom it may belong. Every act which is intended to put an end to indivision among co-heirs and legatees or devisees is deemed to be a partition, although it should purport to be a sale, an exchange, a compromise, or any other transaction.

In the aforesaid deed, the shares of petitioners and Arnel Cruz's in the mass of co-owned properties were concretely determined and distributed to each of them. Since a partition legally made confers upon each heir the

95

exclusive ownership of the property adjudicated to him, it follows that Arnel Cruz acquired absolute ownership over the specific parcels of land assigned to him in the Deed of Partial Partition, including the property subject of this case. As the absolute owner thereof then, Arnel Cruz had the right to enjoy and dispose of the property, as well as the right to constitute a real estate mortgage over the same without securing the consent of petitioners.

G.R. No. 121069 February 7, 2003

BENJAMIN CORONEL AND EMILIA MEKING VDA. DE CORONEL, petitioners, vs. FLORENTINO CONSTANTINO, AUREA BUENSUCESO, AND THE HONORABLE COURT OF APPEALS, respondents.

FACTS:

The property involves two adjacent parcels of land situated in Sta. Monica, Hagonoy, Bulacan. The property is originally owned by Honoria Aguinaldo. One-half (1/2) of it was inherited by Emilia Meking Vda. de Coronel together with her sons Benjamin, Catalino and Ceferino, all surnamed Coronel. The other half was inherited by Florentino Constantino and Aurea Buensuceso.

In 1991, Constantino and Buensuceso filed a complaint for declaration of ownership, quieting of title and damages with prayer for writ of mandatory and/or prohibitory injunction with the Regional Trial Court against the

96

Coronel’s. They allege that Jess C. Santos and Priscilla Bernardo purchased the property belonging to Coronel’s by virtue of a deed of sale signed by Emilia alone. Santos and Bernardo sold the same to Constantino and Buensuceso by virtue of a compromise agreement in a Civil Case. As such, Constantino and Buensuceso claim that they are the owners of the subject property.

The RTC decided in favor of Constantino and Buensuceso. It was appealed but the CA affirmed the decision of the lower court.

ISSUE:

Whether or not the contract of sale executed by a parent-co-owner, in her own behalf, is enforceable with respect to the shares of her co-heirs-children.

RULING:

NO. In the "Kasulatan ng Bilihang Patuluyan" which is a private document, not having been duly notarized, it clearly shows that only the share of Emilia in the subject property was sold because Benjamin did not sign the document and the shares of Ceferino and Catalino were not subject of the sale. Therefore, it is unenforceable against the three of them. Emilia executed the instrument in her own behalf and not in representation of her three children.

Article 493 of the Civil Code provides that "Each co-owner shall have the full ownership of his part and of the fruits and benefits pertaining thereto, and he may therefore alienate, assign or mortgage it, and even substitute another person in its enjoyment, except when personal rights are involved. But the effect of the alienation or the mortgage, with respect to the co-owners, shall be limited to the portion which may be allotted to him in the division upon the termination of the co-ownership." Consequently, the sale of the subject property made by Emilia in favor of Santos and Bernardo is limited to the portion which may be allotted to her upon the termination of her co-ownership over the subject property with her children.

The sale is valid insofar as the share of Emilia is concerned. Hence, Santos and Bernardo, who purchased

97

the share of Emilia, became co-owners of the subject property together with Benjamin and the heirs of Ceferino and Catalino. As such, Santos and Bernardo could validly dispose of that portion of the subject property pertaining to Emilia in favor of Constantino and Buensuceso. However, the particular portions properly pertaining to each of the co-owners are not yet defined and determined as no partition in the proper forum or extrajudicial settlement among the parties has been effected among the parties.

G.R. No. 120864 October 8, 2003

MANUEL T. DE GUIA, petitioner, vs. COURT OF APPEALS (Former Sixth Division) and JOSE B. ABEJO, represented by his Attorney-in-Fact, Hermenegilda Abejo-Rivera, respondents.

FACTS:

Abejo alleged that he is the owner of the ½ undivided portion of a property used as a fishpond ("FISHPOND") situated in Meycauayan, Bulacan. He alleged ownership over approximately 39,611 square meters out of the fishpond’s total area of 79,220 square meters. Abejo further

98

averred that De Guia continues to possess and use the fishpond without any contract and without paying rent to Abejo’s damage and prejudice. Abejo also complained that De Guia refuses to surrender ownership and possession of the fishpond despite repeated demands to do so after De Guia’s sublease contract over the fishpond had expired.

De Guia, a lawyer by profession, alleged that the complaint does not state a cause of action and has prescribed. He claimed that the fishpond was originally owned by Maxima Termulo who died intestate with Primitiva Lejano as her only heir. According to him, Abejo is not the owner of the entire fishpond but the heirs of Primitiva Lejano who authorized him to possess the entire fishpond. He assailed ABEJO’s ownership of the ½ undivided portion of the FISHPOND as void and claimed ownership over an undivided half portion of the FISHPOND for himself. DE GUIA sought payment of damages and reimbursement for the improvements he introduced as a builder in good faith.

The trial court ruled in favor of Abejo. The case was raised to the CA which affirmed the trial court’s decision.

ISSUE:

Whether or not a co-owner can file an action of recovery of possession against a co-owner.

RULING:

Yes. Under Article 484 of the Civil Code, "there is co-ownership whenever the ownership of an undivided thing or right belongs to different persons."

Any co-owner may file an action under Article 487 not only against a third person, but also against another co-owner who takes exclusive possession and asserts exclusive ownership of the property. In the latter case, however, the only purpose of the action is to obtain recognition of the co-ownership. The plaintiff cannot seek exclusion of the defendant from the property because, as co-owner, he has a right of possession. The plaintiff cannot recover any material or determinate part of the property.

99

Abejo and De Guia are owners of the whole and over the whole, they exercise the right of dominion. However, they are at the same time individual owners of a ½ portion, which is truly abstract because until there is partition, such portion remains indeterminate or unidentified. As co-owners, Abejo and De Guia may jointly exercise the right of dominion over the entire fishpond until they partition the fishpond by identifying or segregating their respective portions.

Since a co-ownership subsists between Abejo and De Guia, judicial or extra-judicial partition is the proper recourse. An action to demand partition is imprescriptible and not subject to laches. Each co-owner may demand at any time the partition of the common property unless a co-owner has repudiated the co-ownership under certain conditions. Neither Abejo nor De Guia has repudiated the co-ownership under the conditions set by law.

G.R. No. 205867 February 23, 2015

MARIFLOR T. HORTIZUELA, represented by JOVIER TAGUFA, Petitioner, vs. GREGORIA TAGUFA, ROBERTO TAGUFA and ROGELIO LUMABAN, Respondents.

100

FACTS:

The property involved in this case is a parcel of land containing an area of 539 square meters. By virtue of the special power of attorney executed by Mariflor Tagufa Hortizuela, Jovier Tagufa instituted the instant case against the defendants praying for the peaceful surrender of the above-described property unto them and further ordering defendant Gregoria Tagufa to reconvey in plaintiff’s favor the same property which was titled under her name via fraud.

Before it was titled in the name of Defendant Tagufa, said property was originally owned by plaintiff’s parents, Spouses Epifanio Tagufa and Godofreda Jimenez. Although untitled, the spouses mortgaged the property with the Development Bank of the Philippines (DBP). For failure to redeem the property, DBP foreclosed the same and sold it to Atty. Romulo Marquez who, in turn, sold it back to Runsted Tagufa, husband of defendant Gregoria Tagufa, on April 4, 2002 using the fund sent by plaintiff Hortizuela who was in America and with the agreement that Runsted will reconvey the said property to her sister when demanded.

However, plaintiff discovered that the same unregistered property was titled in the name of Gregoria Tagufa. Investigating further, plaintiff discovered that Gregoria Tagufa was able to title the said property by virtue of a free patent application before the Department of Environment and Natural Resources (DENR) and the execution of a Deed of Extrajudicial Settlement of the Estate of the late Spouses Leandro Tagufa and Remedios Talosig dated May 9, 2003. Plaintiff now seeks to recover possession of the said property which is presently occupied by Gregoria Tagufa and her co-defendants and have the same be reconveyed unto them.

MCTC dismissed the complaint for lack of merit. Not in conformity, Hortizuela appealed to the RTC. RTC reversed the MCTC ruling and granted the appeal of the plaintiff, ordering the defendant Gregorio Tagufa to reconvey to the plaintiff Mariflor Tagufa Hortizuela the land described in the complaint; it also ordered the defendants to vacate the

101

same land and to surrender the peaceful possession thereof to the plaintiff. CA reversed the decision.

ISSUE:

Whether or not an action for reconveyance and recovery of possession constitutes an indirect collateral attack on the validity of the subject Certificate of Title which is proscribed by law.

RULING:

The Supreme Court granted the petition and reversed the decision of the CA. The Court is not unmindful of the principle of indefeasibility of a Torrens title and Section 48 of P.D. No. 1528 where it is provided that a certificate of title shall not be subject to collateral attack. A Torrens title cannot be altered, modified or cancelled except in a direct proceeding in accordance with law. When the Court says direct attack, it means that the object of an action is to annul or set aside such judgment, or enjoin its enforcement. On the other hand, the attack is indirect or collateral when, in an action to obtain a different relief, an attack on the judgment or proceeding is nevertheless made as an incident thereof.

The complaint of Hortizuela was not a collateral attack on the title warranting dismissal. As a matter of fact, an action for reconveyance is a recognized remedy, an action in personam, available to a person whose property has been wrongfully registered under the Torrens system in another’s name. In an action for reconveyance, the decree is not sought to be set aside. It does not seek to set aside the decree but, respecting it as incontrovertible and no longer open to review, seeks to transfer or reconvey the land from the registered owner to the rightful owner. Reconveyance is always available as long as the property has not passed to an innocent third person for value.

The fact that petitioner was able to secure a title in her name did not operate to vest ownership upon her of the subject land. Registration of a piece of land under the Torrens System does not create or vest title, because it is not a mode of acquiring ownership. A certificate of title is merely an evidence of ownership or title over the particular property described therein. It cannot be used to protect a

102

usurper from the true owner; nor can it be used as a shield for the commission of fraud; neither does it permit one to enrich himself at the expense of others. Its issuance in favor of a particular person does not foreclose the possibility that the real property may be co-owned with persons not named in the certificate, or that it may be held in trust for another person by the registered owner.

G.R. No. 154415. July 28, 2005

GASPAR CALACALA, BALTAZAR CALACALA, MELCHOR CALACALA, SOLOMON CALACALA, FELICIDAD CALACALA, PETRONILA CALACALA and SALOME CALACALA, Petitioners, vs. REPUBLIC OF THE PHILIPPINES, represented by the Solicitor General, and SHERIFF JUAN C. MARQUEZ,Respondents.

FACTS:

The predecessors-in-interest, then Spouses Camilo and Conchita Calacala, of herein petitioners, used their property to secure a bond for the temporary liberty of an accused in a criminal case. For failure of such accused to face trial, the bond was forfeited in favor of the government and the same was ultimately levied upon and an annotation was made on the title and registered with the ROD.

An auction was held with the Republic winning the bid, thus a Sheriff’s Deed of Sale was issued in favor of the Republic and the same was registered with the ROD in 1982. A one year redemption period was set for the owners to redeem the property but to no avail and such non-redemption event went beyond their delivery to their respective graves, in 1988 and 1994.

The Calacalas, as legal heirs of the deceased spouses, petitioners filed a complaint for Quieting of Title and Cancellation of Encumbrance on TCT No. T-21204 against respondents Republic and Sheriff Juan C. Marquez. In their complaint, petitioners prayed for the cancellation of entries on the title subject of this case or the declaration of said entries as null and void.

103

The trial court granted the Republic’s motion to dismiss and accordingly dismissed petitioners’ complaint. A Motion for Reconsideration was filed and was likewise dismissed.

Thus, a petition for review on certiorari under Rule 45 of the Rules of Court, was lodged by petitioners to the SC asking to annul and set aside the resolutions of the Regional Trial Court which respectively dismissed petitioners’ complaint and denied their motion for reconsideration.

ISSUE:

Whether or not the trial court’s dismissal of petitioners’ complaint for Quieting of Title proper

RULING:

YES, the Dismissal is proper. Citing Article 476 and 477 of the New Civil Code, the Court said that the remedy of quieting a title may be availed of only when, by reason of any instrument, record, claim, encumbrance or proceeding, which appears valid but is, in fact, invalid, ineffective, voidable or unenforceable, a cloud is thereby casts on the complainant’s title to real property or any interest therein and that the party who may bring an action to quiet title must have legal or equitable title to, or interest in the real property which is the subject-matter of the action. He need not be in possession of said property.

Thus, the Court ruled that for an action to quiet title to prosper, two (2) indispensable requisites must concur, namely: (1) the plaintiff or complainant has a legal or an equitable title to or interest in the real property subject of the action; and (2) the deed, claim, encumbrance or proceeding claimed to be casting cloud on his title must be shown to be in fact invalid or inoperative despite its prima facie appearance of validity or legal efficacy.

Absent the two requisites in the petition of the Calacalas’, the Court upheld the decision of the RTC.

104

G.R. No. 148943 August 15, 2002

AGNES GAPACAN, EUGENIA GAPACAN-KIAKI and MARILYN GAPACAN, petitioners, vs. MARIA GAPACAN OMIPET, respondent.

FACTS:

Antonio Gapacan and Maria Gapacan Omipet are children of Paicat Gapacan. Paicat left an unregistered land without will upon his death. In the adult years of the sibling’s life, Antonio Gapacan left home to find life outside their homestead while Maria remained to cultivate the land and took care of their aging father until his death during the World War II.

Antonio, now married, retired and with his wife and children, returned to his place of birth and there established his family by farming the land owned by their parents. However, instead of just farming, Antonio, in 1954, executed an Affidavit of Transfer of Real Property showing that the property of their father had been transferred to him by his sister Maria, making him in effect the legal owner of the property in question. The Affidavit of Transfer of Real Property was allegedly thumbmarked by Maria's husband, Pedro Omipet, in her behalf. Thus, by virtue of the Affidavit of Transfer of Real Property, Antonio had the property in question declared in his name for taxation purposes in

105

1954. Since then, Agnes Gapacan and her daughters Eugenia and Marilyn had been occupying and cultivating the three (3) parcels of rice land and a parcel devoted to camote subject matter of the present controversy.

As such, when Maria took time to cultivate said land, she was faced by opposition from Antonio’s family which landed in the courts in a case of forcibly entry, depriving Maria her better share on the property.

Given the situation, Maria moved for a quieting of title, alleging in her complaint that the disputed land was part of her inheritance from her deceased parents which she in fact had declared in her name for taxation purposes in 1948 although the area was only 1,188 square meters for which Tax Declaration No. A-0808 was issued in her name. She further contended that she merely lent the parcels of rice land to petitioners when Antonio Gapacan returned to Abatan after his retirement.

The trial court dismissed the complaint. On appeal, CA rendered a decision declaring the property to be the common property of both the petitioner and respondent.

ISSUE:

Whether or not the heirs of Antonio Gapacan has a better right than Maria Gapacan Omipet given their actual possession of the land and the tax declarations issued on former’s favor?

RULING:

Article 476 of the Civil Code provides that an action to quiet title may be brought when there exists a cloud on the title to real property or any interest therein. In the case of Bautista v. Exconde, we held that a property owner whose property rights were being disturbed may ask a competent court for a proper determination of the respective rights of the party-claimants, not only to place things in their proper place, that is, to require the one who has no right to refrain from acts injurious to the peaceful enjoyment of the property not only of the rightful owner but also for the benefit of both with the view of dissipating any cloud of doubt over the property.

106

It goes without saying therefore that the appellate court in resolving the present controversy is well within its authority to adjudicate on the respective rights of the parties, that is, to pass upon the ownership of the subject property; hence to declare the same as common property of the party-litigants. Besides, private respondent Maria Gapacan Omipet instituted the present action for the purpose of asking the court to pass judgment upon the issue of ownership of the disputed property with the hope that she would be declared its rightful owner.

Thus, as the Court noted, while the Heirs of Antonio proved actual possession, such possession was a possession in bad faith, and while Maria has a tax declaration to show her claim to be valid, tax declarations in themselves do not vest absolute ownership of the property upon the declarant, nor do declarations of ownership for taxation purposes constitute adequate evidence of ownership or of the right to possess realty.

It is therefore proper for the CA to declare co-ownership on the juridical concept that co-ownership is unity of the object or property and plurality of subjects x x x x Each co-owner, jointly with the other co-owners, is the owner of the whole property, but at the same time of the undivided aliquot part x x x x Each co-owner has the right to sell, assign or dispose of his share, unless personal rights are involved x x x and therefore, he may lose such rights to others, as by prescription thereof by a co-owner x x x x"

G.R. No. 144208 September 11, 2007

EFREN TANDOG, FELIX TANDOG, FELIPE TANDOG, JOSEFINO TANDOG, HELEN TANDOG, CATALINA TANDOG, ROMEO TANDOG, DOMINGO TANDOG, CATALINA SANTOS, MARIA BAUTISTA CATANYAG, ARTEMIO CATANYAG, ANGELES CATANYAG, APOLONIA CATANYAG, ADORACION CATANYAG, ARCELY CATANYAG, and AMPARO CATANYAG, all represented by EFREN TANDOG, petitioners, vs. RENATO MACAPAGAL, SPOUSES ALFONSO and MARINA CALDERON, and the LANDS MANAGEMENT BUREAU, respondents.

FACTS:

107

The subject of the controversy is a land consisting of 147,991 square meters situated at Sitio Inarawan, Barangay Inuman, San Isidro, Antipolo City.

Petitioners claim that they and their predecessors-in-interest have been in actual, open, continuous, exclusive, and notorious possession of the land since time immemorial. They trace their rights to Casimiro Policarpio, unmarried, who died in 1945. He was survived by his nephews and nieces, now deceased, except Maria Bautista Catanyag. She and Casimiros’ grandnieces and grandnephews (herein petitioners) have continued possessing and cultivating the land. When petitioners decided to apply for the judicial registration of the property, they found that portions of the land have been occupied by spouses Alfonso and Marina Calderon and Renato Macapagal, respondents.

Respondent Marina Calderon, in her answer, specifically denied petitioners allegations in their complaint. She alleged that she and her husband bought their property in 1958 and, since then, have been in possession of the same. They planted trees and crops thereon. Also, they have been paying the corresponding realty taxes. She does not know petitioners who are all strangers in the place.

After petitioners had presented their evidence, spouses Calderon filed a demurrer to evidence. The trial court granted their motion and dismissed the complaint. On appeal by petitioners, the Court of Appeals rendered a decision affirming the order of the trial court dismissing their complaint.

ISSUE:

Whether or not the allegations of spouses Calderon that they purchased their property and Macapagals claim that he applied for a Free Patent are judicial admissions which the petitioners consider as cloud upon their interest in the disputed property.

RULING:

108

As a general rule, a cloud which may be removed by suit to quiet title is not created by mere verbal or parol assertion of ownership of or an interest in property. This rule is subject to qualification, where there is a written or factual basis for the asserted right. Thus, a claim of right based on acquisitive prescription or adverse possession has been held to constitute a removable cloud on title.

While petitioners alleged that respondents claim of adverse possession is a cloud on their (petitioners) interest in the land, however, such allegation has not been proved. The alleged falsified documents relied upon by respondents to justify their possession were merely marked as exhibits but were never formally offered in evidence by petitioners. We have consistently ruled that documents which may have been marked as exhibits during the hearing, but which were not formally offered in evidence, cannot be considered as evidence, nor can they be given any evidentiary value.

It is important that petitioners must first establish their legal or equitable title to, or interest in the real property which is the subject matter of the action. Petitioners failed to do so. Parenthetically, they did not present any evidence to prove that Casimiro Policarpio existed and that he is their predecessor-in-interest.

Records show that petitioners failed to establish by evidence any or all the above requisites.

G.R. No. 157852 December 15, 2010

HEIRS OF DOMINGO VALIENTES, Petitioners, vs. Hon. REINERIO (Abraham) B. RAMAS, Acting Presiding Judge,

109

RTC, Branch 29, 9th Judicial Region, San Miguel, Zamboanga del Sur and Vilma V. Minor, Respondents.

FACTS:

Petitioners claim that they are the heirs of Valientes who, before his death, was the owner of a parcel of land in Zamboanga del Sur. In 1939, Domingo Valientes mortgaged the subject property to secure his loan to the spouses Leon Belen and Brigida Sescon.

In the 1950s, the Valientes family purportedly attempted, but failed, to retrieve the subject property from the spouses Belen. Through an allegedly forged document captioned VENTA DEFINITIVA purporting to be a deed of sale of the subject property between Domingo Valientes and the spouses Belen, the latter obtained Transfer Certificate of Title (TCT) No. T-5,427 in their name.

On February 28, 1970, Maria Valientes Bucoy and Vicente Valientes, legitimate children of the late Domingo Valientes, had their Affidavit of Adverse Claim duly entered in the Memorandum of Encumbrances at the back of TCT No. T-5,427. Upon the death of the spouses Belen, their surviving heirs Brigida Sescon Belen and Maria Lina Belen executed an extra-judicial settlement with partition and sale in favor of private respondent Vilma Valencia-Minor, the present possessor of the subject property.

On June 20, 1979, Minor then filed with the Court of First Instance a petition for cancellation of memorandum of encumbrance appearing in TCT No. T-5,427. On July 31, 2000, the RTC granted Minor’s prayer to allow the Register of Deeds to have the title to the subject property transferred to her name.

In the meantime, on August 20, 1998, petitioners filed a complaint for the cancellation of TCT NO. T-5,427. The Complaint was docketed as Civil Case No. 98-021.

Private respondent Minor filed an Omnibus Motion to Dismiss Civil Case No. 98-021 on the grounds of forum shopping and litis pendentia. On August 3, 2000, the RTC

110

issued an order in open court ruling that forum shopping does not apply. On September 22, 2000, private respondent Minor filed a Motion for Reconsideration of the August 3, 2000 Order.

On May 7, 2001, the RTC issued an Order granting the Motion for Reconsideration by dismissing Civil Case No. 98-021 on the ground of forum shopping. Petitioners filed a Motion for Reconsideration on May 30, 2001, but the same was denied by the RTC in its Order dated September 18, 2001.

On November 12, 2001, petitioners filed with the Court of Appeals a Petition for Certiorari assailing the RTC Orders dated May 7, 2001 and September 18, 2001.

Petitioners raised the sole issue of whether the trial court was correct in finding that Civil Case No. 98-021 constitutes forum shopping, litis pendentia or res judicata with SPL Case No. 186. The Petition was docketed as CA-G.R. SP No. 68501.

The Court of Appeals rendered its assailed Decision on said petition on August 16, 2002. Despite agreeing with petitioners that there was no forum shopping, litis pendentia or res judicata in the filing of Civil Case No. 98-021, the Court of Appeals, asserting that it has the discretion to review matters not otherwise assigned as errors on appeal if it finds that their consideration is necessary at arriving at a complete and just resolution of the case, held that Civil Case No. 98-021 cannot prosper on the grounds of prescription and laches. Hence, the Petition for Certiorari.

ISSUE:

Whether or not prescription or laches has already set in to bar the filing of the case at hand.

HELD:

YES. The case cannot prosper because an action for reconveyance is a legal remedy granted to a landowner

111

whose property has been wrongfully or erroneously registered in another’s name, which must be filed within ten years from the issuance of the title since such issuance operates as a constructive notice (Declaro vs. Court of Appeals, 346 SCRA 57).

Where a party has neglected to assert his rights over a property in question for an unreasonably long period, he is estopped from questioning the validity of another person’s title to the property. Long inaction and passivity in asserting one’s rights over a disputed property precludes him from recovering said property (Po Lam vs. Court vs. Court of Appeals, 347 SCRA 86).

In conclusion, petitioners’ cause of action has already prescribed and now heavily infirmed with laches.

The cause of action of petitioners in Civil Case No. 98-021, wherein they claim that private respondent Minor’s predecessor-in-interest acquired the subject property by forgery, can indeed be considered as that of enforcing an implied trust. In particular, Article 1456 of the Civil Code provides:

Art. 1456. If property is acquired through mistake or fraud, the person obtaining it is, by force of law, considered a trustee of an implied trust for the benefit of the person from whom the property comes.

However, the Court made a clear distinction in Olviga: when the plaintiff in such action is not in possession of the subject property, the action prescribes in ten years from the date of registration of the deed or the date of the issuance of the certificate of title over the property. When the plaintiff is in possession of the subject property, the action, being in effect that of quieting of title to the property, does not prescribe. In the case at bar, petitioners (who are the plaintiffs in Civil Case No. 98-021) are not in possession of the subject property.

If the Civil Case No. 98-021 were to be considered as that of enforcing an implied trust, it should have therefore been filed within ten years from the issuance of TCT No. T-

112

5,427 on December 22, 1969. It was, however, filed on August 20, 1998, which was way beyond the prescriptive period.

Under the Torrens System as enshrined in P.D. No. 1529, the decree of registration and the certificate of title issued become incontrovertible upon the expiration of one year from the date of entry of the decree of registration, without prejudice to an action for damages against the applicant or any person responsible for the fraud.

SC have allowed actions for reconveyance based on implied trusts even beyond such one-year period, for such actions respect the decree of registration as incontrovertible.

Yet, the right to seek reconveyance based on an implied or constructive trust is not absolute nor is it imprescriptible. An action for reconveyance based on an implied or constructive trust must perforce prescribe in ten years from the issuance of the Torrens title over the property.

It took petitioners 28 before filing this case. This period is unreasonably long for a party seeking to enforce its right to file the appropriate case. Thus, petitioners’ claim that they had not slept on their rights is patently unconvincing.

113

G.R. No. 141964 June 30, 2006

SPOUSES EDESITO and CONSORCIA RAGASA, Petitioners, vs.

SPOUSES GERARDO and RODRIGA ROA and the EX-OFFICIO SHERIFF OF QUEZON CITY, Respondents.

FACTS:

On May 10, 1989, Edesito and Consorcia Ragasa entered into a contract with Oakland Development Resources Corporation for the purchase in installments of a piece of property. Immediately thereafter, they took possession of the property covered by TCT No. 27946 of the Registry of Deeds for Quezon City. From May of 1989 up to the present date, plaintiffs were in continuous and notorious possession of the property.

Sps. Ragasa were able to fully pay for the agreed purchase price of the property and a Deed of Absolute Sale was executed and the original owner’s copy of TCT No. 27946 was turned over to them. Upon learning that Oakland Development Resources Corporation was no longer functional as a corporate entity, Consorcia decided to cause the transfer of registration herself.

She was thus surprised to learn from the Registry of Deeds for Quezon City that the property in question was sold by Ex-Officio Sheriff of Quezon City to defendants Sps. Roa as the highest bidder for the price and consideration of P511,000.00 as shown in the Sheriff’s Final Deed of Sale.

114

Sps. Ragasa filed a case to RTC and instead of filing an answer, respondents moved for the dismissal of the complaint on the grounds of prescription and laches. RTC granted the motion. Characterizing the suit as an action "upon an injury to the rights of the plaintiff" which, according to Article 1146 of the Civil Code, must be filed within four years, the RTC held that petitioners’ action was barred by prescription for having been filed more than four years after the registration of the execution sale.

Petitioners proceeded to the SC with the petition for review on certiorari raising only a pure question of law and seeking a reversal of the trial court’s order.

ISSUE:

Whether or not prescription or laches has already set in to bar the filing of the case at hand.

HELD

NO. SC held that the premise of RTC was erroneous. The allegations in petitioners’ complaint reveal that the action was essentially one for quieting of title to real property under Article 476 of the Civil Code which states that:

“Whenever there is a cloud on title to real property or any interest therein, by reason of any instrument, record, claim, encumbrance or proceeding which is apparently valid or effective but is in truth and in fact invalid, ineffective, voidable, or unenforceable, and may be prejudicial to said title, an action may be brought to remove such cloud or to quiet the title.”

An action may also be brought to prevent a cloud being cast upon title to real property or any interest therein.

To make out an action to quiet title, allegations should show that (1) the plaintiff has "title to real property or any interest therein" and (2) the defendant claims an interest therein adverse to the plaintiff’s arising from an

115

"instrument, record, claim, encumbrance, or proceeding which is apparently valid or effective but is in truth and in fact invalid, ineffective, voidable, or unenforceable."

Thus, the averments in petitioners’ complaint that (1) they acquired ownership of a piece of land by tradition or delivery as a consequence of sale and (2) private respondents subsequently purchased the same piece of land at an allegedly void execution sale were sufficient to make out an action to quiet title under Article 476.

It is also an established rule that actions to quiet title to property in the possession of the plaintiff are imprescriptible. Accordingly, petitioners’ action was not subject to prescription.

G.R. No. 178609 August 04, 2010

MANUEL P. NEY AND ROMULO P. NEY V. SPOUSES CELSO P. QUIJANO AND MINA N. QUIJANO

FACTS:

Petitioners Manuel P. Ney and Romulo P. Ney are the registered owners of a residential lot located at 1648 Main Street, Paco Manila. A three (3) door apartment was constructed on the subject lot - one for Manuel, the other for Romulo; and the last one for their sister Mina N. Quijano and her husband Celso Quijano.

On October 8, 1999, Quijano filed with the RTC of Manila a suit for reconveyance, partition and damages against petitioners. They averred that they are co-owners of the subject property having paid part of its purchase price; that Celso's name was inadvertently omitted as one of the buyers in the execution of the deed of sale. Consequently, TCT No. 122489 covering the subject property was issued only in the names of Manuel and Romulo.

116

To obtain a separate certificate of title, they requested from petitioners the segregation of the portion allotted to them, but the latter refused. They later discovered that the entire property was mortgaged with Metropolitan Bank & Trust Company, prompting them to execute and register their adverse claim with the Register of Deeds; and to file the instant complaint,

Petitioners averred that Celso Quijano was not a vendee of the subject lot; thus, his name did not appear on the title. They asserted that respondents cannot validly maintain an action against them because the latter possessed the property by mere tolerance; and even assuming that respondents had a valid cause of action, the same had already been barred by prescription and/or laches. Petitioners, therefore, prayed for the dismissal of the complaint.

RTC sustained petitioners' assertion that respondents possessed part of the property through mere tolerance; and that their cause of action, if any, already prescribed. The RTC thus ruled that respondents can no longer demand the segregation or reconveyance of the claimed portion of the property. CA reversed the decision of RTC. The CA considered respondents' complaint as one for quieting of title which is imprescriptible; and granted to respondents the reliefs that they prayed for.

ISSUE:

Whether or not respondents are entitled for reconveyance and if the latter’s cause of action has been prescribed.

RULING:

Yes, respondents are entitled for their claim and the complaint is one for quieting title which is imprescriptible.

These allegations make out a case for reconveyance. That reconveyance was one of the reliefs sought was made abundantly clear by respondents in their prayer. Undoubtedly, respondents did not only seek the partition of the property and the delivery of the title, but also the

117

reconveyance of their share which was inadvertently included in petitioners' TCT.

An action for reconveyance is one that seeks to transfer property, wrongfully registered by another, to its rightful and legal owner. Indeed, reconveyance is an action distinct from an action for quieting of title, which is filed whenever there is a cloud on title to real property or any interest therein, by reason of any instrument, record, claim, encumbrance or proceeding which is apparently valid or effective but is in truth and in fact, invalid, ineffective, voidable, or unenforceable, and may be prejudicial to said title for purposes of removing such cloud or to quiet title. However, the SC finds nothing erroneous in the CA's ruling treating respondents' action for reconveyance as an action to quiet title.

An action for reconveyance based on an implied trust prescribes in 10 years. The reference point of the 10-year prescriptive period is the date of registration of the deed or the issuance of the title. The prescriptive period applies only if there is an actual need to reconvey the property as when the plaintiff is not in possession of the property. However, if the plaintiff, as the real owner of the property also remains in possession of the property, the prescriptive period to recover title and possession of the property does not run against him. In such a case, an action for reconveyance, if nonetheless filed, would be in the nature of a suit for quieting of title, an action that is imprescriptible.

G.R. No. 157812 November 22, 2005

RODOLFO SANTOS, Petitioner, vs. RONALD C. MANALILI as Heir or Representative of Deceased Defendants NOLI BELEN C. MANALILI and REYNALDO MANALILI & BOARD OF LIQUIDATORS, Respondents.

FACTS:

118

Reynaldo Manalili, predecessor-in-interest of respondent Ronald C. Manalili, filed with the BOL an application to purchase a 4,608 square-meter parcel of land. The application was granted and Manalili paid the downpayment of 10% of the purchase price as required and thereafter declared the land for taxation purposes.

After the lapse of 9 years, Rodolfo Santos wrote an undated letter to the BOL protesting Manalili’s application. A formal investigation was then conducted by BOL and eventually came out with a report that Santos "was not actually occupying the lot and that he only hired certain Abalahin and Lumaad to plant bananas and coconut trees and maintain a vegetable garden thereon presumably to establish a bona-fide occupancy over the lot." Accordingly, BOL recommended the dismissal of Santos' protest and the approval of the sale of Manalili.

Santos filed a complaint for reconveyance and damages. Trial court adjudged against Santos ordering him to immediately vacate the land and deliver the possession to Manalili.

CA upheld the findings of the BOL land examiner that Santos’ protest was unfounded and was only meant to disturb the sale of the subject land to respondent Manalili. To the appellate court, the BOL’s findings were duly supported by evidence, as in fact the sale of the land to Manalili was approved by no less than the Office of the President. Hence, they filed a petition for review on certiorari on the SC.

Santos submits that he has clearly established a better right of possession over the subject property. Per his testimony, the land in dispute was originally occupied by one Col. Agsalud in 1956 up to 1959. Later, Col. Agsalud transferred his rights in favor of one Ernesto Abalahin who continuously occupied the land and from whom Santos acquired the property sometime in February 1969. He then introduced various improvements thereon and continuously occupied the same up to the present.

ISSUE:

Who has better right of possession over the lot in question?

119

RULING:

The Manalilis had a better right of possession thereto. Preponderant evidence of respondent have sufficiently established that as early as 1970, Reynaldo Manalili, respondents’ predecessor-in-interest, had already filed an Affidavit of Occupancy with the BOL, the government agency tasked to administer it; that the Manalilis administered the land before they left for Manila in 1972; that after they moved to Manila they appointed an administrator to oversee the land and the improvements and crops they have planted thereon, such as bananas and coconut trees; and that the Manalilis have been paying the real estate taxes for the subject land even before the sale thereof to them.

Possession may be exercised in one’s own name or in that of another. It is not necessary that the owner or holder of the thing exercise personally the rights of possession. Rights of possession may be exercised through agents. So, the circumstances that after the sale, the Manalilis resided in Manila and Pangasinan are of no moment.

Santos' claim of having bought the land from a certain Ernesto Abalahin who, in turn, bought it from one Col. Agsalud, allegedly a guerrilla veteran who occupied the lot from 1956 to 1959, is without basis. No proof has been presented as to the alleged title or the transfer of any rights. The supposed Deed of Absolute Sale between petitioner and Ernesto Abalahin does not even sufficiently identify the lot which was the subject of the sale and, worse, that same deed is not notarized and is unregistered. A sale of a piece of land appearing in a private deed cannot be considered binding on third persons if it is not embodied in a public instrument and recorded in the Registry of Deeds.

Santos' evidence do not also support his allegation that the BOL’s sale of the land to Reynaldo Manalili was fraudulent. It is well-settled that fraud must be established by clear and convincing evidence. Santos failed in this venture.

It is a settled rule in this jurisdiction that factual findings of an administrative agency deserve utmost

120

respect, more so, when, as here, they are supported by substantial evidence, albeit such evidence may not be overwhelming or merely preponderant.

G.R. No. 164823 August 31, 2005

MARIA CARLOS, represented by TERESITA CARLOS VICTORIA, Petitioners, vs. REPUBLIC OF THE PHILIPPINES, Respondent.

FACTS:

Maria Carlos, represented by her daughter, Teresita Carlos Victoria, filed an application for registration and confirmation of title over a parcel of land with an area of 3,975 square meters located at Pusawan, Ususan, Taguig, Metro Manila. Petitioner alleged, among others, that she is the owner of said parcel of land which she openly, exclusively and notoriously possessed and occupied since July 12, 1945 or earlier. Petitioner further claimed that she has been in possession of the subject land in the concept of an owner; that her possession has been peaceful, public, uninterrupted and continuous since 1948 or earlier; and tacking her possession with that of her predecessors-in-interest, petitioner has been in possession of the land for more than 50 years.

Upon the demise of Maria Carlos, Victoria took possession of the property with the consent of her brothers and sisters. However, they have agreed to undertake the titling of the property and promised to deliver the certificate of title to Ususan Development Corporation which bought the property from Maria Carlos. Victoria admitted that her mother had sold the land to Ususan Development Corporation in 1996 but failed to deliver the title.

The Republic of the Philippines, represented by the Director of Lands, filed an opposition to petitioner’s application. The trial court granted her application. On appeal, the Court of Appeals reversed and set aside the decision of the trial court.

CA noted that at the time she filed her application for registration of title, the applicant was no longer in possession and occupation of the land in question since the

121

applicant’s mother and predecessor-in-interest sold the subject land to Ususan Development Corporation.

ISSUE:

WON Victoria still has the possession in the concept of an owner when she filed an application for registration.

RULING:

SC affirmed the ruling of the appellate court. Applicants for confirmation of imperfect title must prove the following: (a) that the land forms part of the disposable and alienable agricultural lands of the public domain; and (b) that they have been in open, continuous, exclusive, and notorious possession and occupation of the same under a bona fide claim of ownership either since time immemorial or since June 12, 1945.

The applicant must show that he is in actual possession of the property at the time of the application. Actual possession of a land consists in the manifestation of acts of dominion over it of such a nature as a party would naturally exercise over his own property.

It is clear in the case at bar that the applicant, Maria Carlos, no longer had possession of the property at the time of the application for the issuance of a certificate of title. Victoria admitted during the hearing that her mother had sold the property to Ususan Development Corporation in 1996. They also presented as evidence the deed of absolute sale executed by and between Maria Carlos and Ususan Development Corporation. This contradicts petitioner’s claim that she was in possession of the property at the time that she applied for confirmation of title.

Even if it were true that it was petitioner who had actual possession of the land at that time, such possession was no longer in the concept of an owner. Possession may be had in one of two ways: possession in the concept of an owner and possession of a holder. A possessor in the concept of an owner may be the owner himself or one who claims to be so. On the other hand, one who possesses as a

122

mere holder acknowledges in another a superior right which he believes to be ownership, whether his belief be right or wrong.16 Petitioner herein acknowledges the sale of the property to Ususan Development Corporation in 1996 and in fact promised to deliver the certificate of title to the corporation upon its obtention. Hence, it cannot be said that her possession since 1996 was under a bona fide claim of ownership. Under the law, only he who possesses the property under a bona fide claim of ownership is entitled to confirmation of title.

G.R. No. 181812 June 8, 2011

FELICIANO GAITERO and NELIA GAITERO, Petitioners, vs. GENEROSO ALMERIA and TERESITA ALMERIA, Respondents.

FACTS:

A land registration court issued an original certificate of title to Rosario O. Tomagan (Tomagan) covering a 10,741 square-meter land, designated as Lot 9960. Tomagan subdivided the lot awarded to her into four: Lot 9960-A3; Lot 9960-B; Lot 9960-C4; and Lot 9960-D. Tomagan waived her rights over Lots 9960-A and 9960-C in favor of petitioner Feliciano Gaitero (Gaitero) and Lot 9960-B in favor of Barangay Ysulat, Tobias Fornier. She retained Lot 9960-D.

Lot 9960-A that went to Gaitero adjoined Lot 9964 which belonged to respondent spouses Almerias and was covered by OCT P-14556. In June 2000, the Almerias commissioned a relocation survey of their lot and were astonished to find that Gaitero, who owned adjoining Lot 9960-A, intruded into their lot by as much as 737 sq m (the disputed area).

Apparently to settle the dispute, the Almerias waived their rights over a 158 sq m portion of the disputed area in Gaitero’s favor but maintained their claim over the

123

remaining 579 sq m. Subsequently, however, Gaitero filed an affidavit of adverse claim on the Almerias’ title over the remaining 579 sq m.

Gaitero claimed that he was the registered owner of Lot 9960-A, that he inherited the same from his mother, Maria Obay, who in turn inherited it from her father, Bonifacio Obay; that before the cadastral survey, Lot 9960-A was erroneously lumped with Lot 9960 in Tomagan’s name; that, acknowledging the mistake, Tomagan subdivided Lot 9960 into four lots and waived her rights over Lots 9960-A and 9960-C in Gaitero’s favor; that the Almerias claimed a portion of Lot 9960-A by virtue of a relocation survey and fenced it close to Gaitero’s house, obstructing the latter’s passageway; and that while the Almerias returned 158 sq m of the disputed portion, they refused to return to him the remaining 579 sq m.

Almerias alleged that they bought Lot 9964 in 1985 by virtue of an Extra-Judicial Settlement of Estate and Sale; that it was Gaitero who unlawfully encroached on the 737 sq m portion of Lot 9964; and that, while they had waived a portion of the disputed area, Gaitero’s incessant claim to the remaining 579 sq m prompted them to cancel their previous waiver of the 158 sq m. The Almerias prayed for the dismissal of the complaint and the award of damages in their favor.

MTC rendered a decision dismissing the complaint and ordering Gaitero to pay the Almerias damages and held that the Almerias were entitled to the possession of the disputed area considering that it is included in the technical description of their registered title. Further, the MCTC held that Gaitero acknowledged the true boundaries of 9960-A when Lot 9960 was subdivided in 1993. Indeed, the subdivision plan clearly shows that the disputed area is excluded from 9960-A.

RTC reversed the decision of the MCTC due to laches. Although it recognizes the Almerias as rightful owners, laches prevented them from asserting their right over the same since it took them 15 years before they do so.

124

CA reversed the decision of the RTC and reinstated that of the MCTC. The CA held that the Almerias owned the disputed area since, between a registered title and a verbal claim of ownership, the former must prevail. The CA did not consider the Almerias in laches since no one had lodge a claim of ownership against their title to the disputed property.

ISSUE:

Whether or not the CA erred in holding that the Almerias are entitled to th e possession of the disputed area as against Gaitero?

RULING:

Possession is an essential attribute of ownership. Necessarily, whoever owns the property has the right to possess it. Here, between the Almerias’ registered title of ownership and Gaitero’s verbal claim to the same, the former’s title is far superior.

As the MCTC, the RTC, and the CA found, the disputed area forms part of the Almerias’ registered title. Upon examination, this fact is also confirmed by the subdivision plan which partitioned Tomagan’s original Lot 9960. Since Gaitero was unable to prove that fraud attended the titling of the disputed area, the Almerias’ right over the same became indefeasible and incontrovertible a year from registration.

Gaitero's cannot collaterally attack registered titles based on his alleged continuous possession and claim of ownership of the disputed area. Gaitero’s action before the MCTC is one for recovery of possession of the disputed area. . A registered title cannot be impugned, altered, changed, modified, enlarged, or diminished, except in a direct proceeding permitted by law. Otherwise, reliance on registered titles would be lost. Gaitero’s action is prohibited by law and should be dismissed.

Gaitero’s theory of laches cannot vest on him the ownership of the disputed area. To begin with, laches is a consideration in equity and therefore, anyone who invokes it must come to court with clean hands, for he who has done

125

inequity shall not have equity. Here, Gaitero’s claim of laches against the Almerias can be hurled against him. When the lot that the Almerias acquired (Lot 9964) was registered in 1979, Gaitero had constructive, if not actual, notice that the cadastral survey included the disputed area as part of the land that Leon Asenjo claimed. Yet, neither Gaitero nor his mother complained or objected to such inclusion.

When Gaitero saw the subdivision plan covering Tomagan’s original Lot 9960 in 1993, it showed that the disputed area fell outside the boundaries of Lot 9960-A which he claimed. Still, Gaitero did nothing to correct the alleged mistake. He is by his inaction clearly estopped from claiming ownership of the disputed area. He cannot avail himself of the law of equity.

SC dismissed the petition and affirmed the decision of the CA.

G.R. No. 152423 December 15, 2010

SPOUSES MARCOS R. ESMAQUEL and VICTORIA SORDEVILLA, Petitioners, vs. MARIA COPRADA, Respondent.

FACTS:

126

Sps. Marcos Esmaquel and Victoria Sordevilla (Victoria) filed an ejectment case against respondent Maria V. Coprada. In 1945, Coprada was able to persuade the spouses to allow her and her family to use and occupy the land for their residence, under the condition that they will vacate the premises should petitioners need to use the same. Respondent and her family were allowed to construct their residential house. Since then, the petitioners never made an attempt to drive them away out of pity, knowing that respondent and her eight children have no other place to live in. Also, respondent and her family have been occupying the subject premises free of rent, including payment of realty taxes.

When Coprada's present circumstances have completely improved, the spouses verbally demanded that they vacate the subject land but the former refused. Thus, petitioners were forced to send a demand letter dated August 22, 1996, giving respondent until November 30, 1996 to vacate the subject premises. Respondent still ignored said demand; hence, the spouses lodged an ejectment case against Coprada.

Coprada admitted that petitioners are the registered owners of the subject land. However, she averred that in 1945, it was Victoria Sordevilla's mother and not the petitioners who gave permission to her late husband Brigido Coprada to use the subject lot.

Respondent alleged that sometime in the early 1960's, petitioner Victoria offered the said lot for sale for P2,000.00 to respondent, who readily agreed. The purchase price was paid in installments and was fully paid in 1962. Due to their close relationship, the agreement was never reduced to writing.

Coprada further maintained that since the execution of the oral sale of the subject lot, she has been the one paying the realty taxes due on the property. After the sale, respondent built on the subject land a semi-concrete structure. Respondent stated that petitioners' claim is barred by laches. Even granting, without admitting, that respondent's claim of ownership over the property is improper because petitioners are the registered owners

127

thereof, respondent argued that she is a builder in good faith, because she was able to build the structure on the subject lot with the prior permission of the owner.

MCTC rendered judgment dismissing the complaint. It held that laches had already set in which prevented petitioners from questioning the validity of the purported sale between Victoria and Maria.

On appeal, the RTC reversed the MCTC’s judgment. The RTC ruled that respondent's occupation of the subject property was by virtue of petitioners' tolerance and permission. Hence, respondent is bound by an implied promise that she will vacate the property upon demand. respondent also failed to prove the alleged oral sale and that petitioners have adequately proven that they are entitled to the possession of the subject land as registered owners thereof.

The CA granted respondent's petition, reversed the Decision of the RTC and affirmed in toto the Decision of the MCTC. Hence, the instant petition with the SC.

ISSUE:

Whether or not petitioners have a valid ground to evict respondent from the subject property.

RULING:

Court held that respondent failed to present evidence to substantiate her allegation that a portion of the land was sold to her in 1962. In fact, when petitioners sent a letter to the respondent, demanding her to vacate the subject property, the respondent, in reply to the said letter, never mentioned that she purchased the subject land in 1962. If the sale really took place, the respondent should have immediately and categorically claimed that in her letter response. As against the respondent's unproven claim that she acquired a portion of the property from the petitioners by virtue of an oral sale, the Torrens title of petitioners must prevail. Moreover, the age-old rule is that the person who has a Torrens title over a land is entitled to possession thereof.

128

The validity of petitioners' certificate of title cannot be attacked by respondent in this case for ejectment. A certificate of title shall not be subject to collateral attack. It cannot be altered, modified or canceled, except in a direct proceeding for that purpose in accordance with law. Whether or not the respondent has the right to claim ownership over the property is beyond the power of the trial court to determine in an action for unlawful detainer.

It is settled that a Torrens Certificate of title is indefeasible and binding upon the whole world unless and until it has been nullified by a court of competent jurisdiction. As the registered owner, petitioner had a right to the possession of the property, which is one of the attributes of ownership.

The doctrine that a registered owner may lose his right to recover its possession by reason of laches is not applicable here. Laches is the failure or neglect, for an unreasonable and unexplained length of time, to do that which, by exercising due diligence, could or should have been done earlier.

Respondent first acquired possession of the subject lot by mere tolerance. From 1945 until the filing of the complaint for ejectment in 1997, the nature of that possession has never changed. Petitioners allowed the respondent to possess the property with the knowledge that the respondent will vacate the same upon demand. Hence, until such demand to vacate was communicated by the petitioners to the respondent, petitioners are not required to do any act to recover the subject land, precisely because they knew of the nature of the respondent's possession, i.e., possession by mere tolerance. Thus, it cannot be said that petitioners are guilty of failure or neglect to assert a right within a reasonable time.

In contrast, respondent, who is claiming that a portion of the property was sold to her in 1962, has herself failed within a long period of time to have that portion transferred in her name. As registered owners of the lots in question, the private respondents have a right to eject any person

129

illegally occupying their property. This right is imprescriptible.

Since respondent's occupation of the subject property was by mere tolerance, she has no right to retain its possession under Article 448 of the Civil Code. She is aware that her tolerated possession may be terminated any time and she cannot be considered as builder in good faith.

G.R. No. 157044 October 5, 2005

RODOLFO V. ROSALES, (represented by his heirs, Rodolfo, Jr., Romeo Allan, Lillian Rhodora, Roy Victor, Roger Lyle and Alexander Nicolai, all surnamed Rosales) and LILY ROSQUETA-ROSALES, Petitioners vs. MIGUEL CASTELLTORT, JUDITH CASTELLTORT, and LINA LOPEZ-VILLEGAS, assisted by her Attorney-in-Fact, Rene Villegas, Respondents.

FACTS:

Spouses-petitioners Rodolfo V. Rosales and Lily Rosqueta-Rosales are the registered owners of a parcel of land with an area of approximately 315 square meters designated as Lot17. Petitioners discovered that a house was being constructed on their lot, without their knowledge and consent, by respondent Miguel Castelltort.

Castelltort and his wife Judith had purchased a lot, Lot 16 of the same Subdivision Plan, from respondent Lina Lopez-Villegas (Lina) through her son-attorney-in-fact Rene Villegas (Villegas) but that after a survey thereof by geodetic engineer Augusto Rivera, he pointed to Lot 17 as the Lot 16 the Castelltorts purchased.

Negotiations for the settlement of the case thus began, with Villegas offering a larger lot near petitioners’ lot in the same subdivision as a replacement thereof. In the alternative, Villegas proposed to pay the purchase price of petitioners’ lot with legal interest. Both proposals were rejected.

130

To the complaint, the Castelltorts claimed in their Answer with Counterclaim that they were builders in good faith.

Lina alleged that the Castelltorts acted in good faith in constructing the house on petitioners’ lot as they in fact consulted her before commencing any construction thereon, they having relied on the technical description of the lot sold to them, Lot 16, which was verified by her officially designated geodetic engineer.

RTC ruled out good faith. CA granted the appeal and set aside the RTC decision.

ISSUE:

Whether appellant Miguel is a builder in good faith.

RULING:

Miguel relied on the title which the intervenor showed to him which, significantly, has no annotation that would otherwise show a prior adverse claim. Thus, as far as appellant Miguel is concerned, his title over the subject lot, as well as the title of the intervenor thereto, is clean and untainted by an adverse claim or other irregularities.

Miguel’s failure to secure a building permit from the Municipal Engineer’s Office on their construction on Lot 17 does not impinge on his good faith. Miguel, in good faith, built the house on appellees’ land without knowledge of an adverse claim or any other irregularities that might cast a doubt as to the veracity of the assurance given to him by the intervenor. Miguel cannot be faulted for having relied on the expertise of the land surveyor who is more equipped and experienced in the field of land surveying.

The appellants’ house erroneously encroached on the property of the appellees due to a mistake in the placement of stone monuments as indicated in the survey plan, which error is directly attributable to the fault of the geodetic engineer who conducted the same. This fact alone negates bad faith on the part of appellant Miguel.

131

A builder in good faith is one who builds with the belief that the land he is building on is his or that by some title one has the right to build thereon, and is ignorant of any defect or flaw in his title.

Article 527 of the Civil Code provides that good faith is always presumed, and upon him who alleges bad faith on the part of a possessor rests the burden of proof. The certified true copy bore no annotation indicating any prior adverse claim on Lot 16. The records indicate that at the time Castelltort began constructing his house on petitioners’ lot, he believed that it was the Lot 16 he bought and delivered to him by Villegas.

In the case at bar, Castelltort’s good faith ceased on August 21, 1995 when petitioners personally apprised him of their title over the questioned lot. As held by the CA, should petitioners then opt to appropriate the house, they should only be made to pay for that part of the improvement built by Castelltort on the questioned property at the time good faith still existed on his part or until August 21, 1995.

G.R. No. 158929 August 3, 2010

ROSARIO P. TAN, Petitioner, vs. ARTEMIO G. RAMIREZ, MOISES G. RAMIREZ, RODRIGO G. RAMIREZ, DOMINGO G. RAMIREZ, and MODESTA RAMIREZ ANDRADE, Respondents.

FACTS:

Rosario Tan filed a case for recovery of ownership of a parcel of land against respondents Ramirez, et. al. Rosario alleges that her great-grandfather, Catalino, acquired said property under a Tax Declaration in 1915. The respondents trace their ownership of said land through a compromise agreement and deed of absolute sale from Gavino who has been cultivating the said property. Gavino is the husband of one of the daughters of Catalino.

The appellate court ruled that the respondents are the true owners of the property, there being acquisitive

132

prescription. The deed of absolute sale and compromise agreement constitute just title, thus, their 24 years of possession was more than enough to meet the 10-year possession required by law.

ISSUE:

Whether or not the compromise agreement and deed of sale were enough bases for good faith and just title, thus allowing ordinary acquisitive prescription to take place.

RULING:

NO. A compromise agreement is aimed to put an end to litigation; it does not create or transmit ownership rights over a property. A deed of sale could not also be used as a basis for showing good faith in cases of acquisitive prescription. Good faith, or the want of it, can be ascertained only from the acts of the one claiming it, as it is a condition of mind that can only be judged by actual or fancied token or signs.

The respondents in the case at bar were aware of the pending litigation over the property, as evidenced by the compromise agreement. Thus, they cannot be considered to be in good faith in acquiring said property by prescription. The respondents did not acquire the property through ordinary acquisitive prescription since they were not in good faith. Likewise, they did not acquire such by extraordinary acquisitive prescription for holding it only for 24 years, short of the 30-year period mandated by law.

G.R. No. 172217 September 18, 2009

SPOUSES LYDIA FLORES-CRUZ and REYNALDO I. CRUZ, Petitioners, vs. SPOUSES LEONARDO and ILUMINADA GOLI-CRUZ, SPOUSES RICO and FELIZA DE LA CRUZ, SPOUSES BOY and LANI DE LA CRUZ, ZENAIDA A. JACINTO and ROGELIO DE LOS SANTOS, Respondents.

FACTS:

Petitioner spouses Lydia Flores-Cruz and Reynaldo I. Cruz purchased a 5,209-sq. m. lot situated in Pulong Yantok, Angat, Bulacan5 from Lydia’s siblings, namely, Teresita, Ramon and Daniel (all surnamed Flores).

133

After the death of Lydia's father, petitioners found out that respondent spouses Leonardo and Iluminada Goli-Cruz et al. were occupying a section of the land. Initially, petitioner Lydia talked to respondents and offered to sell them the portions they were occupying but the talks failed as they could not agree on the price.

Respondents countered that their possession of the land ranged from 10 to 20 years. According to respondents, the property was alienable public land.

Petitioners filed a complaint for recovery of possession of the land in the RTC. RTC rendered a decision in favor of petitioners.

CA ruled that RTC had no jurisdiction over the action for recovery of possession because petitioners had been dispossessed of the property for less than a year.

ISSUE:

The issue for resolution is whether the RTC had jurisdiction over this case.

RULING:

That the nature of the action – on which depends the question of whether a suit is within the jurisdiction of the court – is determined solely by the allegations in the complaint and the law at the time the action was commenced.

SC agree with the CA that considering that petitioners claimed that respondents were possessors of the property by mere tolerance only and the complaint had been initiated less than a year from the demand to vacate, the proper remedy was an action for unlawful detainer which should have been filed in the MTC.

Based on the allegations in petitioners’ complaint, it is apparent that such is a complaint for unlawful detainer based on possession by tolerance of the owner. It is a

134

settled rule that in order to justify such an action, the owner’s permission or tolerance must be present at the beginning of the possession. Such jurisdictional facts are present here.

There is another reason why petitioners’ complaint was not a proper action for recovery of possession cognizable by the RTC. It is no longer true that all cases of recovery of possession or accion publiciana lie with the RTC regardless of the value of the property.

When the case was filed in 2001, Congress had already approved Republic Act No. 769122 which expanded the MTC’s jurisdiction to include other actions involving title to or possession of real property (accion publiciana and reinvindicatoria) where the assessed value of the property does not exceed P20,000 (or P50,000, for actions filed in Metro Manila).

Since petitioners’ complaint made out a case for unlawful detainer which should have been filed in the MTC and it contained no allegation on the assessed value of the subject property, the RTC seriously erred in proceeding with the case. The proceedings before a court without jurisdiction, including its decision, are null and void.34 It follows that the CA was correct in dismissing the case.

G.R. No. 182673 October 5, 2009

AQUALAB PHILIPPINES, INC., Petitioner, vs. HEIRS OF MARCELINO PAGOBO

135

FACTS:

Subject of the complaint initiated by respondents are Lots 6727-Q and 6727-Y. Lot 6727-Q and Lot 6727-Y used to form part of Lot 6727 owned by respondents’ great grandfather, Juan Pagobo containing an area of 127,436 square meters. Lot 6727 was once covered by Juan Pagobo’s homestead application. Shortly after homestead patent was issued, the lots were subsequently sold to Tarcela de Espina; to Rene Espina; to Anthony Gaw Kache; and finally to Aqualab. TCT was issued.

Respondents held that they are absolute and legal owners and rightful possessors of Lot [no.] 6727-Q and Lot no. 6727-Y. These are ancestral lands which are part of a bigger parcel of land. Ownership and Possession by plaintiff’s predecessors-in-interest, and plaintiffs herein, respectively, over the said land, have been peaceful, continuous [sic] open, public and adverse, since the year 1936 or even earlier. Their peaceful possession was disturbed only in 1991.

RTC granted Aqualab’s motion and dismissed respondents’ complaint. In granting Aqualab’s motion to dismiss, the trial court ruled that prescription has set in. Moreover, the trial court held that Aqualab is an innocent purchaser for value and, thus, its rights are protected by law. Finally, it concluded that legal redemption or reconveyance was no longer available to respondents.

CA reversed the decision of the RTC declaring the sale of subject as null and void.

The CA resolved the following issues: (1) the propriety of the dismissal of the complaint by the RTC; and, (2) whether respondents have the right to redeem subject lots. The CA ruled that the trial court erred in dismissing the complaint as the sale of subject lots to Tarcela de Espina was void, thus making the subsequent conveyances ineffective and no titles were validly transferred. Moreover, it ruled that Aqualab is not an innocent purchaser for value, and held that respondents, as heirs of the homestead grantee, never lost their valid title to the subject lots.

136

ISSUE:

Whether the action of respondents is barred by prescription;

Whether Aqualab is an innocent purchaser for value.

RULING:

Respondents aver that they are the absolute and lawful owners of subject properties over which they have had actual possession since 1936 or earlier until sometime in 1991, when Aqualab disturbed such possession. While the records show that respondents did not have in their names the certificate of titles over subject lots, the factual assertion of open, peaceful, public, and adverse possession is hypothetically admitted by Aqualab.

Moreover, respondents allege that the conveyances of subject lots were fraudulently made in violation of the restrictions on alienation of homesteads under CA 141, and that said conveyances were made without their knowledge and, thus, asserting their right to redeem the subject properties in line with the policy of CA 141 that the homestead should remain with the grantee and his family. The alleged fraudulent conveyances were likewise hypothetically admitted by Aqualab.

Respondents have duly averred continuous possession until 1991 when such was allegedly disturbed by Aqualab. Being in possession of the subject lots—hypothetically admitted by Aqualab—respondents’ right to reconveyance or annulment of title has not prescribed or is not time-barred.

Verily, an action for annulment of title or reconveyance based on fraud is imprescriptible where the plaintiff is in possession of the property subject of the acts.

A buyer of real property that is in the possession of a person other than the seller must be wary, and a buyer who does not investigate the rights of the one in possession can hardly be regarded as a buyer in good faith.

137

Having hypothetically admitted respondents’ possession of subject lots, Aqualab cannot be considered, in the context of its motion to dismiss, to be an innocent purchaser for value or a purchaser in good faith. Moreover, the defense of indefeasibility of a Torrens title does not extend to a transferee who takes it with notice of a flaw in the title of his transferor.

G.R. No. 178906 February 18, 2009

ELVIRA T. ARANGOTE, petitioner, vs. SPS. MARTIN MAGLUNOB and LOURDES S. MAGLUNOB, and ROMEO SALIDO, Respondents.

FACTS:

Elvira T. Arangote, herein petitioner married to Ray Mars E. Arangote, is the registered owner of the subject property. Respondents Martin (Martin II) and Romeo are first cousins and the grandnephews of Esperanza Maglunob-Dailisan (Esperanza), from whom petitioner acquired the subject property.

The Petition stems from a complaint filed by petitioner and her husband against the respondents for Quieting of Title, Declaration of Ownership and Possession, Damages with Preliminary Injunction, and Issuance of Temporary Restraining Order before the MCTC. The complaint alleged that Esperanza inherited the subject property from her uncle Victorino Sorrosa. Esperanza executed a Last Will and Testament bequeathing the subject property to petitioner and her husband, but it was never probated.

On 9 June 1986, Esperanza executed another document, an Affidavit, in which she renounced, relinquished, waived and quitclaimed all her rights, share, interest and participation whatsoever in the subject property in favor of petitioner and her husband. In 1989, petitioner and her husband constructed a house on the subject property.

Respondents averred that they co-owned the subject property with Esperanza. The subject property was co-owned by Esperanza, respondent Martin II (together with his wife Lourdes), and respondent Romeo, each holding a one-third pro-indiviso share therein. Thus, Esperanza could

138

not validly waive her rights and interest over the entire subject property in favor of the petitioner.

Respondents also asserted in their Counterclaim that petitioner and her husband, by means of fraud, undue influence and deceit were able to make Esperanza, who was already old and illiterate, affix her thumbmark to the Affidavit dated 9 June 1986, wherein she renounced all her rights and interest over the subject property in favor of petitioner and her husband. Respondents thus prayed that the OCT issued in petitioner’s name be declared null and void insofar as their two-thirds shares are concerned.

The MCTC rendered a decision declaring petitioner and her husband as the true and lawful owners of the subject property. RTC reversed the MCTC decision. The Court of Appeals likewise rendered a Decision denying the Petition for Review of petitioner and her husband and affirming the RTC Decision.

ISSUE:

Whether or not the petitioners are builders in good faith.

RULING:

Court held that the findings of fact of the MCTC as regards the origin of the subject property are in conflict with the findings of fact of both the RTC and the Court of Appeals. SC affirms the findings of both the RTC and the Court of Appeals as regards the origin of the subject property and the fact that respondents, with their grand aunt Esperanza, were co-heirs and co-owners of the subject property. It is clear from the records that the subject property was not Esperanza’s exclusive share, but also that of the other heirs of her father, Martin I. Esperanza expressly affixed her thumbmark to the Deed of Extrajudicial Settlement of July 1981 not only for herself, but also on behalf of the other heirs of Martin I.

Further, it bears emphasis that the Partition Agreement was executed by and among the son, grandsons, granddaughters and cousins of Victorino. Esperanza was neither the granddaughter nor the cousin of Victorino, as

139

she was only Victorino’s grandniece. The cousin of Victorino is Martin I, Esperanza’s father. In effect, therefore, the subject property allotted to Esperanza in the Partition Agreement was not her exclusive share, as she holds the same for and on behalf of the other heirs of Martin I, who was already deceased at the time the Partition Agreement was made.

Esperanza’s Affidavit is, in fact, a Donation. Esperanza’s real intent in executing the said Affidavit was to donate her share in the subject property to petitioner and her husband.

Art. 749 of the Civil Code provides, "In order that the donation of an immovable may be valid, it must be made in a public document, specifying therein the property donated and the value of the charges which the donee must satisfy."

Three requisites for the validity of a simple donation of a real property: (1) it must be made in a public instrument; (2) it must be accepted, which acceptance may be made either in the same Deed of Donation or in a separate public instrument; and (3) if the acceptance is made in a separate instrument, the donor must be notified in an authentic form, and the same must be noted in both instruments.

In the present case, the said Affidavit, which is tantamount to a Deed of Donation, met the first requisite, as it was notarized; thus, it became a public instrument. Nevertheless, it failed to meet the aforesaid second and third requisites.

Possession in good faith ceases from the moment defects in the title are made known to the possessor by extraneous evidence or by a suit for recovery of the property by the true owner. Every possessor in good faith becomes a possessor in bad faith from the moment he becomes aware that what he believed to be true is not so.

In the present case, when respondents came to know that an OCT over the subject property was issued and registered in petitioner’s name, respondents brought a complaint before the Lupon of Barangay Maloco, Ibajay,

140

Aklan, challenging the title of petitioner to the subject property on the basis that said property constitutes the inheritance of respondent, together with their grandaunt Esperanza, so Esperanza had no authority to relinquish the entire subject property to petitioner. From that moment, the good faith of the petitioner had ceased.

In this case, the subject property waived and quitclaimed by Esperanza to the petitioner and her husband in the Affidavit was only covered by a tax declaration in the name of Esperanza. Petitioner did not even bother to look into the origin of the subject property and to probe into the right of Esperanza to relinquish the same. Thus, when petitioner and her husband built a house thereon in 1989 they cannot be considered to have acted in good faith as they were fully aware that when Esperanza executed an Affidavit relinquishing in their favor the subject property the only proof of Esperanza’s ownership over the same was a mere tax declaration. Settled is the rule that a tax declaration does not prove ownership. It is merely an indicium of a claim of ownership. Payment of taxes is not proof of ownership; it is, at best, an indicium of possession in the concept of ownership.

Hence, the petitioner is not entitled to the rights under Article 448 and 546 as the petitioner is not a builder and possessor in good faith. SC affirmed the RTC decision and declared the respondents as the lawful owners and possessors of the subject property.

G.R. No. 162886 August 11, 2008

HEIRS OF THE DECEASED SPOUSES VICENTE S. ARCILLA and JOSEFA ASUNCION ARCILLA, namely: Aida Arcilla Alandan, Rene A. Arcilla, Oscar A. Arcilla, Sarah A. Arcilla, and Nora A. Arcilla, now deceased and substituted by her son Sharmy Arcilla, represented by their attorney-in-fact, SARAH A. ARCILLA, petitioners, vs. MA. LOURDES A. TEODORO, respondent.

FACTS:

Ma. Lourdes A. Teodoro (respondent) initially filed with the RTC of Virac, Catanduanes an application for land registration of two parcels of land located at Barangay San Pedro, Virac, Catanduanes. Respondent alleged that, with the exception of the commercial building constructed

141

thereon, she purchased the subject lots from her father, Pacifico Arcilla (Pacifico), as shown by a Deed of Sale and that, prior thereto, Pacifico acquired the said lots by virtue of the partition of the estate of his father, Jose Arcilla evidenced by a document entitled Extrajudicial Settlement of Estate. Respondent also presented as evidence an Affidavit of Quit-Claim in favor of Pacifico, executed by herein petitioners as Heirs of Vicente Arcilla (Vicente), brother of Pacifico.

In their opposition, petitioners contended that they are the owners pro-indiviso of the subject lots including the building and other improvements constructed thereon by virtue of inheritance from their deceased parents, spouses Vicente and Josefa Arcilla; contrary to the claim of respondent, the lots in question were owned by their father, Vicente, having purchased the same from a certain Manuel Sarmiento sometime in 1917; Vicente's ownership is evidenced by several tax declarations attached to the record; petitioners and their predecessors-in-interest had been in possession of the subject lots since 1906.

MTC rendered a decision declaring Lourdes A. Teodoro as the valid owner of the subject property. RTC dismissed the appeal of petitioners for lack of merit. CA dismissed the same.

ISSUE:

Whether or not the petitioners have right of possession and ownership to subject property.

RULING:

NO. The Court agrees with the CA in its finding that petitioners failed to present any substantial evidence, such as a deed of sale, to prove their claim that their predecessor, Vicente Arcilla, bought the disputed property from Sarmiento. Petitioners were only able to present tax declarations in Vicente's name to prove their allegation that Vicente became the owner of the subject property. The tax declarations presented in evidence by petitioners are not supported by any other substantial proofs.

142

Tax declarations do not prove ownership but are at best an indicium of claims of ownership. Payment of taxes is not proof of ownership, any more than indicating possession in the concept of an owner. Neither a tax receipt nor a declaration of ownership for taxation purposes is evidence of ownership or of the right to possess realty when not supported by other effective proofs.

In addition, the Court agrees with the CA when it held that if Vicente, in fact, owned the disputed properties, his widow, Josefa, would not have agreed to include said lots among those partitioned in the Extrajudicial Settlement of the Estate of Jose.

On the other hand, respondent's claim of ownership is not only backed up by tax declarations but also by other pieces of evidence such as the subject Extrajudicial Settlement, Affidavit of Quitclaim, and Deed of Sale.

G.R. No. 146550 March 17, 2006

FELIPA DELFIN, GINA MAALAT, SHIRLEY TAMAYO, RECIO DAÑOS, and ROBERTO DELFIN, Petitioners, vs. PRESENTACION D. BILLONES, ROSARIO D. DEMONARCA (accompanied by husband Pedro and Demonarca), WENEFREDO DEGALA (representing Pedro Degala),

143

RAMON DELA CRUZ (representing his deceased wife Maria Daradar dela Cruz), TERESITA DALIVA DEVIENTE (daughter of Esperanza Daradar Daliva), and JOLLY DATAR (representing his deceased mother Trinidad D. Datar) and the COURT OF APPEALS, Respondents.

FACTS:

A Deed of Absolute Sale over Lot No. 213 of the Cadastral Survey of Panitan, Capiz, was executed by Teresa Daños, Esperanza Daradar, Estrella Daradar and Maria Daradar, with the marital consent of Cipriano Degala, husband of Teresa Daños, in favor of the spouses Rodolfo Delfin and Felipa Belo (spouses Delfin). The document, so it appears, bore the signatures of Esperanza and Estrella, as well as the thumb marks of Teresa, Maria, and Cipriano, and was acknowledged before a notary public. The spouses Delfin registered the sale with the ROD and a new title was issued under their name.

Respondents, claiming to be the heirs of the former owners of Lots No. 213 and No. 3414, filed an action for annulment, reconveyance, recovery of ownership and possession and damages.8 According to them, it was only in 1989 when they discovered that Teresa Daños, sick and in dire need of money, was constrained to mortgage the one-half (1/2) portion of Lot No. 3414 to the spouses Delfin for P300.00 sometime in 1965.

Taking advantage of her condition, the spouses Delfin made her sign a document purporting to be a mortgage, but which turned out to be an extrajudicial partition with deed of absolute sale. As to Lot No. 213, respondents averred that the Deed of Sale covering the property was fictitious and the signatures and thumb marks contained therein were all forged because three (3) of the signatories therein died before the alleged sale in 1960, namely: Estrella Daradar, who died in 1934, and Esperanza Daradar and Cipriano Degala, who both died in 1946.

Petitioners alleged that respondents’ action was already barred by prescription and laches. Further, they argued that the spouses Delfin, as well as the subsequent owners of the subject properties, are innocent purchasers for value and in good faith, whose titles to the lots at the

144

time of the purchase were all clean and free from liens and encumbrances.

Giving credence to the claims of petitioners, the trial court ruled that respondents’ claim of ownership over the subject properties was not established by a preponderance of evidence. Compared to respondents’ verbal claims of ownership, the spouses Delfin were able to prove that they bought the properties from the original owners. The trial court held that the deeds of sale being duly executed notarial and public documents; they enjoy the presumption of regularity which can only be contradicted by clear and convincing evidence. In addition, respondents’ claims based on fraud were barred by prescription, having been filed more than four (4) years from the time the instruments were registered with the Register of Deeds, and they are estopped from annulling the documents by reason of laches, the action having been filed 15 years after the deeds were registered. The trial court also denied respondents’ claims for damages.

The CA reversed the decision of the trial court. The Court of Appeals ruled that while an action for reconveyance based on implied or constructive trust prescribes in ten (10) years from the date of the issuance of the certificate of title over the property, such prescriptive period does not apply if the person claiming to be the owner of the property is in possession thereof, such as respondents in this case.

ISSUE:

Whether or not the petitioners have a right to possession on the subject property.

RULING:

YES. When one’s property is registered in another’s name without the former’s consent, an implied trust is created by law in favor of the true owner. Implied trusts are those which, without being expressed, are deducible from the nature of the transaction by operation of law as matters of equity, independently of the particular intention of the parties. Meanwhile, constructive trusts are created in order to satisfy the demands of justice and prevent unjust enrichment. They arise against one who, by fraud, duress or

145

abuse of confidence, obtains or holds the legal right to property which he ought not, in equity and good conscience, to hold.

An action for reconveyance based upon an implied or constructive trust prescribes in ten (10) years from the registration of the deed or from the issuance of the title, registration being constructive notice to all persons. However, an action for reconveyance based on fraud is imprescriptible where the plaintiff is in possession of the property subject of the acts.

Court held that respondents failed to prove that fraud attended the sale of Lots No. 213 and No. 3414. A duly executed contract carries with it the presumption of validity. The party who impugns its regularity has the burden of proving its simulation. A notarized document is executed to lend truth to the statements contained therein and to the authenticity of the signatures. Notarized documents enjoy the presumption of regularity which can be overturned only by clear and convincing evidence. Hence, parties who have the burden of proof must produce such quantum of evidence, with plaintiffs having to rely on the strength of their own evidence, not on the weakness of the defendant’s.

146

G. R. No. 147074 July 15, 2005

Spouses RODRIGO PADERES and SONIA PADERES , Petitioners, vs. The Hon. COURT OF APPEALS,1 Hon. CARLOTA P. VALENZUELA, in her capacity as the Liquidator of Banco Filipino Savings and Mortgage Bank,2 Respondents.

FACTS:

Manila International Construction Corporation (MICC) executed a real estate mortgage over 21 registered parcels of land including the improvements thereon in favor of Banco Filipino Savings and Mortgage Bank (Banco Filipino) in order to secure a loan. The mortgage was registered with the Registry of Deeds of Pasay City and annotated on the corresponding transfer certificates of title (TCTs).

The 21 mortgaged properties included two lots covered by TCT Nos. 610626 and 61078. Subsequently MICC sold the lot covered by TCT No. 61078, together with the house thereon, to the petitioners in the first case, the Paderes spouses. MICC sold the house built on the lot covered by TCT No. 61062 to the petitioners in the second case, the Bergado spouses. Neither sale was registered, however.

For failure of MICC to settle its obligations, Banco Filipino filed a verified Petition for the extrajudicial foreclosure of MICC’s mortgage; at the auction sale of the foreclosed properties. Banco Filipino was declared the highest bidder.

147

A writ of possession was granted by RTC, copies of which addressed to MICC "and/or All persons claiming rights under them" to voluntarily vacate the premises within 7 days from receipt thereof, were served on petitioners. Instead of vacating the two lots, however, petitioners filed separate petitions before the Court of Appeals, docketed as C.A. G.R. Numbers 42470 and 42471 which were later consolidated, assailing the validity of the Writ of Possession. On appeal, the CA dismissed the consolidated petitions for lack of merit and uphold the validity of the writ of possession.

Petitioners’ Motion for Reconsideration of the appellate court’s decision having been denied by Resolution of February 16, 2001, they jointly come before this Court arguing that: (1) having purchased their respective properties in good faith from MICC, they are third parties whose right thereto are superior to that of Banco Filipino; (2) they are still entitled to redeem the properties and in fact a binding agreement between them and the bank had been reached; (3) their respective houses should not have been included in the auction sale of the mortgaged properties; (4) on the contrary, as builders in good faith, they are entitled to the benefits of Article 448 of the Civil Code; and (5) the writ of possession issued by the RTC in 1996 had already lost its validity and efficacy.

ISSUE:

Whether or not the writ of possession is effective against petitioners.

RULING:

YES. That petitioners purchased their properties from MICC in good faith is of no moment. The purchases took place after MICC’s mortgage to Banco Filipino had been registered. As such, a real right or lien in favor of Banco Filipino had already been established, subsisting over the properties until the discharge of the principal obligation, whoever the possessor(s) of the land might be.

Sale or transfer cannot affect or release the mortgage. A purchaser is necessarily bound to acknowledge and respect the encumbrance to which is subjected the purchased thing and which is at the disposal of the creditor

148

"in order that he, under the terms of the contract, may recover the amount of his credit therefrom." For, a recorded real estate mortgage is a right in rem, a lien on the property whoever its owner may be. Because the personality of the owner is disregarded; the mortgage subsists notwithstanding changes of ownership; the last transferee is just as much of a debtor as the first one; and this, independent of whether the transferee knows or not the person of the mortgagee. So it is, that a mortgage lien is inseparable from the property mortgaged. All subsequent purchasers thereof must respect the mortgage, whether the transfer to them is with or without the consent of the mortgagee. For, the mortgage, until discharge, follows the property.

Petitioners are also not entitled to redeem the foreclosed properties. The debtor in extra-judicial foreclosures under Act No. 3135, or his successor-in-interest, has, one year from the date of registration of the Certificate of Sale with the Registry of Deeds, a right to redeem the foreclosed mortgage; petitioners failed to do so.

G.R. No. L-30817 September 29, 1972

DOMINADOR DIZON, doing business under the firm name "Pawnshop of Dominador Dizon", petitioner, vs. LOURDES G. SUNTAY, respondent.

FACTS:

A diamond ring was turned over to a certain Clarita R. Sison, for sale on commission, along with other pieces of jewelry of respondent Suntay. It was then pledged to petitioner. Since what was done was violative of the terms of the agency, there was an attempt on her part to recover possession thereof from petitioner, who refused. She had to file an action then for its recovery. She was successful, as noted above, both in the lower court and thereafter in the Court of Appeals. She prevailed as she had in her favor the protection accorded by Article 559 of the Civil Code.

149

Suntay is the owner of a three-carat diamond ring valued at P5,500.00. She and Clarita R. Sison entered into a transaction wherein the ring was delivered to Clarita R. Sison for sale on commission. Upon receiving the ring, Clarita R. Sison executed and delivered to Suntay the receipt ... . Suntay had already previously known Clarita R. Sison as the latter is a close friend of the respondent's cousin and they had frequently met each other at the place of the respondent said cousin.

After the lapse of a considerable time without Clarita R. Sison having returned to Suntay the latter's ring, the plaintiff made demands on Clarita R. Sison for the return of her ring but the latter could not comply with the demands because, without the knowledge of the respondent, three days after the ring above-mentioned was received by Clarita R. Sison from the respondent, said ring was pledged by Melia Sison, niece of the husband of Clarita R. Sison, evidently in connivance with the latter, with the defendant's pawnshop for P2,600.00.

Sison finally delivered to the former the pawnshop ticket ... which is the receipt of the pledge with the defendant's pawnshop of the respondent's ring. Suntay filed a case of estafa and filed an action with the RTC for the recovery of the said ring. The lower court issued the writ of replevin and later on rendered judgment declaring that Suntay had the right to the possession of the ring in question.

CA denied the appeal of Dizon.

RULING:

Article 559 of the Civil Code provides "The possession of movable property acquired in good faith is equivalent to a title. Nevertheless, one who has lost any movable or has been unlawfully deprived thereof may recover it from the person in possession of the same. If the possessor of a movable lost of which the owner has been unlawfully deprived, has acquired it in good faith at a public sale, the owner cannot obtain its return without reimbursing the price paid therefor."

If the possessor of a movable lost of which the owner has been unlawfully deprived, has acquired it in good faith

150

at a public sale, the owner cannot obtain its return without reimbursing the price paid therefor. The only exception the law allows is when there is acquisition in good faith of the possessor at a public sale, in which case the owner cannot obtain its return without reimbursing the price.

The common law principle that were one of two innocent persons must suffer by a fraud perpetrated by another, the law imposes the loss upon the party who, by his misplaced confidence, has enabled the fraud to be committed, cannot be applied in a case which is covered by an express provision of the new Civil Code, specifically Article 559. Between a common law principle and a statutory provision, the latter must prevail in this jurisdiction.

Petitioner invoked the principle of estoppel. Estoppel has its roots in equity. Good faith is its basis. It is a response to the demands of moral right and natural justice. For estoppel to exist though, it is indispensable that there be a declaration, act or omission by the party who is sought to be bound.

In the present case, not only has the ownership and the origin of the jewels misappropriated been unquestionably proven but also that the accused, acting fraudulently and in bad faith, disposed of them and pledged them contrary to agreement, with no right of ownership, and to the prejudice of the injured party, who was thereby illegally deprived of said jewels; therefore, in accordance with the provisions of article 464, the owner has an absolute right to recover the jewels from the possession of whosoever holds them.

Petitioner ought to have been on his guard before accepting the pledge in question. Estoppel certainly cannot be justly invoked.

G.R. No. L-20264 January 30, 1971

CONSUELO S. DE GARCIA and ANASTACIO GARCIA, petitioners, vs. HON. COURT OF APPEALS, ANGELINA D. GUEVARA and JUAN B. GUEVARA, respondents.

151

FACTS:

Mrs. Guevara owned a pretty diamond ring with white gold mounting, 2.05 diamond-solitaire, and 4 brills. Sometime in February 1952, the ring was stolen from her house. Luckily, on October 1953 (barely a year after), she found it at a restaurant, La Bulakena, on the finger of the restaurant owner, Consuelo De Garcia.

Guevara asked De Garcia where she bought it and explained to her how she had lost it. When the ring was handed to her by De Garcia, it fitted her perfectly. The next time around, she brought her husband and Rebullida, the person whom she bought the ring from, to verify the identity of the ring. Rebullida examined the ring with the aid of high power lens and his 30 years of experience. He concluded that it was the very ring that he had sold to the Guevaras. After that, Guevara sent a written request for the ring, but De Garcia did not deliver it. When the sheriff tries to serve a writ of seizure, De Garica likewise refused to deliver the ring.

According to De Garcia, she bought the ring from her kumare who got it from another Miss who in turn got it from the owner, a certain Aling Petring. Aling Petring however, was nowhere to be found. She boarded three months at the first buyer’s house but left a week after her landlady bought the ring. The first buyer did not even know Aling Petring’s last name nor her forwarding address. De Garcia claims to be a holder in good faith and for value. She says her possession is equivalent to title.

ISSUE:

Who has a better right of possession.

RULING:

Respondent Angelina D. Guevara, having been unlawfully deprived of the diamond ring in question, was entitled to recover it from petitioner Consuelo S. de Garcia who was found in possession of the same.

The controlling provision is Article 559 of the Civil Code. It reads thus: "The possession of movable property acquired in good faith is equivalent to a title. Nevertheless, one who has lost any movable or has been unlawfully

152

deprived thereof may recover it from the person in possession of the same. If the possessor of a movable lost of which the owner has been unlawfully deprived, has acquired it in good faith at a public sale, the owner cannot obtain its return without reimbursing the price paid therefor."

The only exception the law allows is when there is acquisition in good faith of the possessor at a public sale, in which case the owner cannot obtain its return without reimbursing the price.

153

G.R. Nos. 154391-92 September 30, 2004

Spouses ISMAEL and TERESITA MACASAET, petitioners, vs. Spouses VICENTE and ROSARIO MACASAET, respondents.

FACTS:

The present case involves a dispute between parents and children. The children were invited by the parents to occupy the latter’s two lots, out of parental love and a desire to foster family solidarity. Unfortunately, an unresolved conflict terminated this situation. Out of pique, the parents asked them to vacate the premises. Thus, the children lost their right to remain on the property. They have the right, however, to be indemnified for the useful improvements that they constructed thereon in good faith and with the consent of the parents. In short, Article 448 of the Civil Code applies.

Petitioners Ismael and Teresita Macasaet and Respondents Vicente and Rosario Macasaet are first-degree relatives. Ismael is the son of respondents, and Teresita is his wife. the parents filed with the Municipal Trial Court in Cities (MTCC) of Lipa City an ejectment suit against the children. Respondents alleged that they were the owners of two (2) parcels of land; that by way of a verbal lease agreement, Ismael and Teresita occupied these lots in March 1992 and used them as their residence and the situs of their construction business; and that despite repeated demands, petitioners failed to pay the agreed rental of P500 per week.

Ismael and Teresita denied the existence of any verbal lease agreement. They claimed that respondents had invited them to construct their residence and business on the subject lots in order that they could all live near one other, employ Marivic (the sister of Ismael), and help in resolving the problems of the family. They added that it was the policy of respondents to allot the land they owned as an advance grant of inheritance in favor of their children.

154

The MTCC ruled in favor of respondents and ordered petitioners to vacate the premises. On appeal, the RTC upheld the findings of MTCC and allowed respondents to appropriate the building and other improvements introduced by petitioners after payment of the indemnity provided for by Article 448 in relation to Articles 546 and 548 of the Civil Code.

The CA sustained the finding of the two lower courts that Ismael and Teresita had been occupying the subject lots only by the tolerance of Vicente and Rosario. Thus, possession of the subject lots by petitioners became illegal upon their receipt of respondents’ letter to vacate it.

In ascertaining the right of petitioners to be reimbursed for the improvements they had introduced on respondents’ properties, the appellate court applied the Civil Code’s provisions on lease. The CA modified the RTC Decision by declaring that Article 448 of the Civil Code was inapplicable. The CA opined that under Article 1678 of the same Code, Ismael and Teresita had the right to be reimbursed for one half of the value of the improvements made.

ISSUE:

Whether or not Article 1678 of the Civil Code should apply to the case on the matters of improvements, or is it Article 447 of the Civil Code in relation to the Article 453 and 454 thereof that should apply, if ever to apply the Civil Code.

RULING:

In the present case, petitioners failed to justify their right to retain possession of the subject lots, which respondents own. Since possession is one of the attributes of ownership, respondents clearly are entitled to physical or material possession.

This Court has consistently held that those who occupy the land of another at the latter’s tolerance or permission, without any contract between them, are necessarily bound by an implied promise that the occupants will vacate the property upon demand. A summary action for ejectment is

155

the proper remedy to enforce this implied obligation. The unlawful deprivation or withholding of possession is to be counted from the date of the demand to vacate.

Court ruled out the finding of possession by mere tolerance. Petitioners were able to establish that respondents had invited them to occupy the subject lots in order that they could all live near one other and help in resolving family problems. The occupancy of the subject lots by petitioners was not merely "something not wholly approved of" by respondents. Neither did it arise from what Tolentino refers to as "neighborliness or familiarity." In point of fact, their possession was upon the invitation of and with the complete approval of respondents, who desired that their children would occupy the premises. It arose from familial love and a desire for family solidarity, which are basic Filipino traits.

When persistent conflict and animosity overtook the love and solidarity between the parents and the children, the purpose of the agreement ceased. Thus, petitioners no longer had any cause for continued possession of the lots. Their right to use the properties became untenable. It ceased upon their receipt of the notice to vacate. And because they refused to heed the demand, ejectment was the proper remedy against them. Their possession, which was originally lawful, became unlawful when the reason therefor -- love and solidarity -- ceased to exist between them.

The right of petitioners to inherit from their parents is merely inchoate and is vested only upon the latters’ demise. Indisputably, rights of succession are transmitted only from the moment of death of the decedent.

Article 447 is not applicable, because it relates to the rules that apply when the owner of the property uses the materials of another. It does not refer to the instance when a possessor builds on the property of another, which is the factual milieu here. Article 1678 of the Civil Code is likewise inapplicable.

On the other hand, when a person builds in good faith on the land of another, the applicable provision is Article 448. The established facts of this case show that

156

respondents fully consented to the improvements introduced by petitioners. In fact, because the children occupied the lots upon their invitation, the parents certainly knew and approved of the construction of the improvements introduced thereon. Thus, petitioners may be deemed to have been in good faith when they built the structures on those lots.

Respondents have the right to appropriate -- as their own -- the building and other improvements on the subject lots, but only after (1) refunding the expenses of petitioners or (2) paying the increase in value acquired by the properties by reason thereof. They have the option to oblige petitioners to pay the price of the land, unless its value is considerably more than that of the structures -- in which case, petitioners shall pay reasonable rent.

G.R. No. 141325 July 31, 2006

PELBEL MANUFACTURING CORPORATION, Substituted by Pelagia Beltran, and Virginia Malolos, petitioners, vs. HON. COURT OF APPEALS, and THE REPUBLIC OF THE PHILIPPINES, respondents.

FACTS:

Pelbel Manufacturing Corporation, Aladdin Trinidad and Virginia Malolos are the original applicants sought to register two parcels of land situated in San Juan, Taytay, Rizal. Initial hearing was scheduled; there being no formal opposition on record, an order of general default was issued and applicants were allowed to present evidence ex-parte. Laguna Lake Development Authority filed a manifestation stating that the subject lots are situated below the elevation of 12.50 meters, thus forming part of the bed of the Laguna Lake.

Opposition was filed by the Office of the Solicitor General alleging that neither the applicants nor their

157

predecessors-in-interest have been in open, continuous, exclusive and notorious possession and occupation of the land since June 12, 1945 or prior thereto; that the applicants' claim of ownership in fee simple on the basis of Spanish Title or grant can no longer be availed of for failure to file the appropriate application for registration within six (6) months from February 16, 1976 as required by P.D. No. 892; and that applicant Pelbel Manufacturing Corporation is disqualified, being a private corporation, to hold lands of the public domain except by lease pursuant to Section 11, Article XIV of the 1973 Constitution.

Pelbel filed a motion to substitute Pelagia P. Beltran in its place as applicant with respect to 17,500 square meters of the lot applied for and was granted by the lower court. The lower court rendered the assailed decision adjudicating the parcels of land applied for in favor of the following: Pelagia Beltran – 17,500 square meters; Aladdin Trinidad – 2,500 square meters; Virginia Malolos – 10,251 square meters.

A Motion to Amend Order of General Default and Set Aside Decision was filed by Laguna Lake Development Authority on the ground that LLDA had already established by preliminary investigations that the lots are below elevation of 12.50 meters, hence are of public dominion. The lower court directed the Office of the Solicitor General to file comment on the motion. The OSG filed its Comment supporting the LLDA's position that lakes and their beds such as the lots sought to be registered are considered public domain. OSG further argued that applicant Pelbel, being a corporation, is disqualified from acquiring lands of the public domain and that applicants are not entitled to registration for lack of the requisite number of years of possession before June 12, 1945.

The trial court rendered a decision in favor of the petition. On appeal, CA reversed the decision of the lower court. It dismissed the applications for land registration of petitioners Pelagia Beltran, Aladdin F. Trinidad and Virginia Malolos.

ISSUES:

Whether or not the subject lots are public land.

158

If they are not public land, whether applicants-petitioners have registrable title to the land.

RULING:

The controlling act in this case is the Public Land Act. Under the Public Land Act, there is a presumption that the land applied for belongs to the state, and that the occupants and possessors can only claim an interest in the land by virtue of their imperfect title or continuous, open, and notorious possession thereof for a period prescribed by law. The basic doctrine is that all lands not otherwise appearing to be clearly within private ownership are presumed to belong to the State.

Any applicant for judicial confirmation of an imperfect title has the burden of proving, by incontrovertible evidence, that the (a) land applied for is alienable and disposable public land; and, (b) the applicant, by himself or through his predecessors-in-interest had occupied and possessed the land, in the concept of owner, openly, continuously, exclusively, and adversely since June 12, 1945, or earlier.

Court held that petitioners failed to show that the parcels of land subject of their application are alienable and disposable.

The following are of public dominion: (1) Rivers and their natural beds; (2) Continuous or intermittent waters of springs and brooks running in their natural beds and the beds themselves; (3) Waters rising continuously or intermittently on lands of public dominion; (4) Lakes and lagoons formed by Nature on public lands, and their beds.

Petitioners invoke the case of Bautista v. Court of Appeals, claiming that the inundation was merely due to the rains, and that the water elevation should be determined from the highest ordinary depth during dry season. It is contended that the measurement of Laguna Lake Development Authority Geodetic Engineer Merida of 12.19 meters as the highest observed elevation of the subject lots was made in November, which is still rainy season. We disagree for while November is not part of the summer season, it is not part of the rainy season either. It still is

159

part of the dry season during which the waters are at their "highest ordinary depth."

In Ledesma v. Municipality of Iloilo, this Court held that "simple possession of a certificate of title, under the Torrens System, does not make the possessor the true owner of all the property described therein. If a person obtains a title, under the Torrens System, which includes by mistake or oversight land which cannot be registered under the Torrens System, he does not, by virtue of said certificate alone, become the owner of the lands illegally included." It is basic principle that prescription does not run against the government.

Possession is open when it is visible and apparent to a common observer. Continuous possession consists of uninterrupted acts of nonpermissive possession of property by the current occupants and their predecessors. To be notorious, possession must be so conspicuous that it is generally known and talked of by the public or at least by the people in the vicinity of the premises. Mere possession of land and the making of vague assertions to the public that a possessor is claiming the land are not sufficient to satisfy the requirement of open and notorious possession. Bernardo failed to show that his alleged possession and occupation were of the nature and duration required by law.

160